You are on page 1of 124

Algebra (Ring Theory)

Undergraduate Course
Academic Year: 2019-2020

Prof. Dr. Akeel Ramadan Mehdi Al-Yassiri


Department of Mathematics, College of Education, Al-Qadisiyah University
Al-Diwaniyah City/ Al-Qadisiyah/ Iraq.
E-mail: akeel.mehdi@qu.edu.iq .

Contents
1 Definition and Elementary Properties of Rings 3
1.1 Basic Definitions and Examples . . . . . . . . . . . . . . . . . . . . . . . . . . . . . . . . . . . . . . . 3
1.2 Subrings . . . . . . . . . . . . . . . . . . . . . . . . . . . . . . . . . . . . . . . . . . . . . . . . . . . . 11
1.3 Integral Domains . . . . . . . . . . . . . . . . . . . . . . . . . . . . . . . . . . . . . . . . . . . . . . . 14
1.4 Direct Sums of Rings . . . . . . . . . . . . . . . . . . . . . . . . . . . . . . . . . . . . . . . . . . . . . 18
1.5 Characteristic of rings . . . . . . . . . . . . . . . . . . . . . . . . . . . . . . . . . . . . . . . . . . . . . 20

2 Ideals and Quotient Rings 22


2.1 Ideals . . . . . . . . . . . . . . . . . . . . . . . . . . . . . . . . . . . . . . . . . . . . . . . . . . . . . 22
2.2 Some Operations on Ideals . . . . . . . . . . . . . . . . . . . . . . . . . . . . . . . . . . . . . . . . . . 26
2.3 Quotient Rings . . . . . . . . . . . . . . . . . . . . . . . . . . . . . . . . . . . . . . . . . . . . . . . . 33

3 Ring Homomorphisms and Isomorphisms 36


3.1 Ring Homomorphisms . . . . . . . . . . . . . . . . . . . . . . . . . . . . . . . . . . . . . . . . . . . . 36
3.2 Ring Isomorphism Theorems . . . . . . . . . . . . . . . . . . . . . . . . . . . . . . . . . . . . . . . . . 44

4 Division Rings, Fields and Prime Fields 50


4.1 Division Rings and Fields . . . . . . . . . . . . . . . . . . . . . . . . . . . . . . . . . . . . . . . . . . . 50
4.2 Subfields and Prime Fields . . . . . . . . . . . . . . . . . . . . . . . . . . . . . . . . . . . . . . . . . . 54

5 Certain Special Ideals 57


5.1 Maximal and Minimal ideals . . . . . . . . . . . . . . . . . . . . . . . . . . . . . . . . . . . . . . . . . 57
5.2 Krull-Zorn’s Theorem and Some Applications . . . . . . . . . . . . . . . . . . . . . . . . . . . . . . . . 64
5.3 Local Rings and Jacobson Radical of Rings . . . . . . . . . . . . . . . . . . . . . . . . . . . . . . . . . 65
5.4 Nil Ideals . . . . . . . . . . . . . . . . . . . . . . . . . . . . . . . . . . . . . . . . . . . . . . . . . . . 70
5.5 Prime Ideals . . . . . . . . . . . . . . . . . . . . . . . . . . . . . . . . . . . . . . . . . . . . . . . . . . 72
5.6 Primary Ideals . . . . . . . . . . . . . . . . . . . . . . . . . . . . . . . . . . . . . . . . . . . . . . . . . 76
5.7 Radical of Ideals and Semiprime Ideals . . . . . . . . . . . . . . . . . . . . . . . . . . . . . . . . . . . 78

6 Polynomial Rings 82
6.1 Definitions and Some Properties . . . . . . . . . . . . . . . . . . . . . . . . . . . . . . . . . . . . . . . 82
6.2 Division Algorithm, Remainder Theorem and Factorization Theorem . . . . . . . . . . . . . . . . . . . . 88
6.3 Irreducible Polynomials . . . . . . . . . . . . . . . . . . . . . . . . . . . . . . . . . . . . . . . . . . . . 96

7 Extension Fields 99
7.1 Extension Fields and Kronecker Theorem . . . . . . . . . . . . . . . . . . . . . . . . . . . . . . . . . . 99
7.2 Splitting Extension Fields . . . . . . . . . . . . . . . . . . . . . . . . . . . . . . . . . . . . . . . . . . . 104

8 Boolean Rings 107


8.1 Definition, Examples and Some Properties of Boolean Rings . . . . . . . . . . . . . . . . . . . . . . . . 107

1
9 Basic Concepts in Module Theory 110
9.1 Definition and Examples of Modules . . . . . . . . . . . . . . . . . . . . . . . . . . . . . . . . . . . . . 110
9.2 Submodules . . . . . . . . . . . . . . . . . . . . . . . . . . . . . . . . . . . . . . . . . . . . . . . . . . 114
9.3 Quotient Modules . . . . . . . . . . . . . . . . . . . . . . . . . . . . . . . . . . . . . . . . . . . . . . . 117
9.4 Homomorphisms of Modules and Isomorphism Theorems . . . . . . . . . . . . . . . . . . . . . . . . . 118
9.5 Simple Modules . . . . . . . . . . . . . . . . . . . . . . . . . . . . . . . . . . . . . . . . . . . . . . . . 122

2
Ring Theory Prof. Dr. Akeel Ramadan Mehdi Al-Yassiri

1 Definition and Elementary Properties of Rings


1.1 Basic Definitions and Examples
Definition 1.1. A ring is an ordered triple (R, +, .) consisting of a non-empty set R
and two binary operations on R called addition (+) and multiplication (.), satisfying
the following properties:
(R1) (R, +) is an abelian group, that is,
(A1) a + (b + c) = (a + b) + c for all a, b, c in R.
(A2) a + b = b + a for all a, b in R.
(A3) There is an element 0 ∈ R satisfying a + 0 = a for all a in R.
(A4) For every a ∈ R there is an element b ∈ R such that a + b = 0.
(R2) R is associative under multiplication: that is, (a.b).c = a.(b.c), for all a, b, c ∈ R.
(R3) Multiplication is distributive (on both sides) over addition; that is, a.(b + c) =
a.b + a.c and (a + b).c = a.c + b.c , for all a, b, c ∈ R. (The two distributive laws are
respectively called the left distributive law and the right distributive law.)

Remarks 1.2. (1) We usually refer simply to the ring as R, rather than (R, +, .).
(2) We usually write ab instead of a.b.
(3) The identity of the additive abelian group is called zero element of the ring R
and is unique. We denote the zero element of a ring by 0.
(4) The additive inverse of an element a of the additive abelian group (R, +) shall,
as usual, be denoted by −a. Thus, in a ring R, a + (−a) = 0 for all a ∈ R.
(5) If a, b ∈ R, we denote a + (−b) by a − b.
(6) If a ring R contains only one element, i.e., R = {0}, then it is called a trivial
ring or zero ring.
(7) If a ∈ R, then we write an = a.a...a +
{z... + a}, for any n ∈ Z .
| {z } and na = a| + a +
n times n times

Remark 1.3. If (R, +, .) is a ring, then:


(1) (R, +) is an abelian group.
(2) (R, .) is semigroup.
(3) The additive identity is unique.
(4) The additive inverse of any element in R is unique.
(5) The cancellation law for addition holds. That is if a, b, c ∈ R with a + b = a + c,
then b = c.

Definition 1.4. A commutative ring is a ring (R, +, .) for which ab = ba, for all
a, b ∈ R. If a ring is not commutative it is called noncommutative.

Department of Mathematics/ College of Education/ Al-Qadisiyah University 3 of 124


Ring Theory Prof. Dr. Akeel Ramadan Mehdi Al-Yassiri

Definition 1.5. A ring with identity e (also called a ring with unity) is a ring R which
contains an element e ∈ R (with e 6= 0) satisfying ea = ae = a , for all a ∈ R.
Generally, the unity or identity element of a ring R is denoted by 1 or 1R .

Definition 1.6. A ring which has finite many elements is called finite ring.

Example 1.7. Z, Q, R and C are commutative rings with identity, with the usual
operations of addition and multiplication, where Z (respect: Q, R, C) is the set of all
integer (respect: rational, real, complex) numbers

Example 1.8. Let n ≥ 1 be an integer. Then the set Zn = {0, 1, ..., n − 1} under ad-
dition +n and multiplication .n modulo n is a commutative ring with unity 1, known
as the ring of integers modulo n. The multiplication modulo n is defined on Zn as
following: ab mod n (or a.n b) is the integer r ∈ Zn such that ab = qn + r in Z for
some q ∈ Z.

Example 1.9. Show that the triple (Z6 , +6 , .6 ) is a commutative ring with identity.
Proof. (A) First we will prove that (Z6 , +6 ) is an abelian group.
Consider the following addition table:

+6 0 1 2 3 4 5
0 0 1 2 3 4 5
1 1 2 3 4 5 0
2 2 3 4 5 0 1
3 3 4 5 0 1 2
4 4 5 0 1 2 3
5 5 0 1 2 3 4
- From table we have Z6 is closed under addition +6 .
- Let s,t, r ∈ Z6 . Thus (s +6 t) +6 r = (s + t) + r = s + (t + r) = s +6 (t + r) =
s +6 (t +6 r). Hence +6 is an associative operation.
- Let m ∈ Z6 . Then m +6 0 = m + 0 = m = 0 +6 m ⇒ 0 is the additive identity in Z6 .
- Let m ∈ Z6 . Put r = 6 − m ⇒ r ∈ Z6 and m +6 r = m +6 6 − m = m + 6 − m = 0.
Then every element m in Z6 has additive inverse 6 − m in Z6 .
- Let a, b ∈ Z6 . Then a +6 b = a + b = b + a = b +6 a. Hence +6 is commutative.
Therefore, (Z6 , +6 ) is an abelian group.
(B) Now we will prove that (Z6 , .6 ) is a semigroup.
Consider the following addition table:

Department of Mathematics/ College of Education/ Al-Qadisiyah University 4 of 124


Ring Theory Prof. Dr. Akeel Ramadan Mehdi Al-Yassiri

.6 0 1 2 3 4 5
0 0 0 0 0 0 0
1 0 1 2 3 4 5
2 0 2 4 0 2 4
3 0 3 0 3 0 3
4 0 4 2 0 4 2
5 0 5 4 3 2 1
- From table we have Z6 is closed under multiplication .6 .
- Let s,t, r ∈ Z6 . Thus (s.6t).6 r = (s.t).r = s.(t.r) = s.6 (t.r) =
s.6 (t.6 r). Hence .6 is an associative operation.
Thus (Z6 , .6 ) is a semigroup.
(C) Let r, s,t ∈ Z6 . Thus r.6 (s +6 t) = r.(s + t) = r.s + r.t = r.s +6 r.t =
(r.6 s) +6 (r.6t) and (s +6 t).6 r = (s + t).r = s.r + t.r = s.r +6 t.r =
(s.6 r) +6 (t.6 r). Hence (Z6 , +6 , .6 ) is a ring.
(D) Let r, s ∈ Z6 . Then r.6 s = r.s = s.r = s.6 r. Hence .6 is commutative.
(E) For any r ∈ Z6 , we have 1.6 r = 1.r = r = r.6 1.
Therefore, (Z6 , +6 , .6 ) is a commutative ring with identity 1.

Example 1.10. Let R = {0, 2, 4} ⊆ Z6 . Then (R, +6 , .6 ) is a commutative ring with


identity 4.

 
a11 a12 a13 . . . a1n
a21 a22 a23 . . . a2n 
Example 1.11. Let R be a ring and let Mn (R) = { . . . . . . . . . . . . . . . . . . . . |ai j ∈ R}

an1 an2 an3 . . . ann


be the set of all n × n matrices with entries in R. Then Mn (R) is a ring with respect
to addition and multiplication of matrices is called the ring of n × n matrices over R.
 identity, thenthe ring Mn (R) has identity, namely the n × n identity matrix
If R has
1 0 0 ... 0
0 1 0 . . . 0
In = 
. . . . . . . . . . . . ..

0 0 0 ... 1
 If n ≥ 2 and R  is not  then Mn (R) is noncommutative.
 a trivial ring,   For if A =
a 0 0 b 0 ab 0 0
and B = , then AB = and BA = . Thus AB 6= BA when
0 0 0 0 0 0 0 0
ab 6= 0.

Department of Mathematics/ College of Education/ Al-Qadisiyah University 5 of 124


Ring Theory Prof. Dr. Akeel Ramadan Mehdi Al-Yassiri

Example 1.12. Let R be a ring and let RX be the set of all functions from a nonempty
set X to R. Define addition and multiplication on RX by ( f + g)(x) = f (x) + g(x) and
( f .g)(x) = f (x).g(x) for all f , g ∈ RX . Then (RX , +, .) is a ring. If R is commutative,
then (RX , +, .) is also a commutative ring.

Example 1.13. The set 2Z = {2x | x ∈ Z} of even integers under ordinary addition
and multiplication is a commutative ring without unity. More generally, if n ≥ 2,
then the set nZ = {xn | x ∈ Z} under ordinary addition and multiplication is a com-
mutative ring without unity


Example 1.14. Let S = {m + n 2 | m, n ∈ Z} ⊂ R, and let √+, . be √
the usual addition
and multiplication
√ of
√ real numbers. That is,√ for any m + n 2, r +t 2 ∈ S, we define
(m + n√2) + (r +√t 2) = (m + r) + (n + t) 2 and √
(m + n 2).(r + t 2) = (mr + 2nt) + (mt + nr) 2.
Then (S, +, .) is a commutative ring with identity.
Note first that (S, +) is an abelian subgroup of the group (R, +) (H.W.). Hence S
is an abelian group. √ √
Also, S√is closed under multiplication,
√ since
√ (m + n 2).(r +t 2) = (mr + 2nt) +
(mt + nr) 2 ∈ S, for any (m + n 2), (r +t 2) ∈ S. Hence multiplication in R gives
a binary operation on S.
The associative and distributive properties of . in S hold because they hold in the
larger set R. Therefore,
√ (S, +, .) is√a commutative
√ ring. √ √
Since 1 + 0 √2 ∈ S and (1 + 0 2).(m + n 2) = m + n 2 for any m + n 2 ∈ S,
we have 1 + 0 2 is the identity of the ring S and hence (S, +, .) is a commutative
ring with identity.

Exercise 1. Let ∗, ◦ be two binary operations on Z defined by: a ∗ b = a + 2b and


a ◦ b = 2ab, for all a, b ∈ Z. Prove that the multiplication ◦ is distributive (on both
sides) over addition ∗.


Exercise 2. Let p be a prime integer number. Prove that the set S = {m+n p | m, n ∈
Z} with the usual addition and multiplication of real numbers is a commutative ring
with identity.

Exercise 3. Let X be a nonempty set and P(X) the power set of X, that is, P(X) =
{A | A ⊆ X}. Define A4B = (A − B) (B − A) for all subsets A and B of X. Prove
S

that (P(X), 4, ) is a commutative ring with identity X.


T

Department of Mathematics/ College of Education/ Al-Qadisiyah University 6 of 124


Ring Theory Prof. Dr. Akeel Ramadan Mehdi Al-Yassiri

Exercise 4. Let R = {0, a, b, c} and let +, . be two operations defined on R as in the


following tables:
+ 0 a b c . 0 a b c
0 0 a b c 0 0 0 0 0
a a 0 c b a 0 a b c
b b c 0 a b 0 a b c
c c b a 0 c 0 0 0 0
Prove that (R, +, .) is a noncommutative ring without identity.
Exercise 5. Let ∗, ◦ be two binary operations on R defined by: a ∗ b = a + b + 1 and
a ◦ b = a + b + ab, for all a, b ∈ R. Prove that (R, ∗, ◦) is a commutative ring with
identity.

Exercise 6. Let R be the set of all functions from R to R. Define addition and
multiplication on R by ( f + g)(x) = f (x) + g(x) and ( f ◦ g)(x) = f (g(x)) for all
f , g ∈ R. Show that whether (R, +, ◦) is a ring or not and why?.

Definition 1.15. Let R be a ring with identity 1. A non zero-element a in a ring R


is called a unit if it has a multiplicative inverse, i.e., if there exists b ∈ R such that
ab = ba = 1. We denote the multiplicative inverse of a by a−1 .

Theorem 1.16. Let R be a ring with identity 1.


(1) The multiplicative identity is unique.
(2) Let a ∈ R. If a has a multiplicative inverse in R, then it is unique.
Proof. (1) Let R be a ring with identity 1. Assume that 10 is another identity of R.
Since 1 is identity, 1.10 = 10 . Since 10 is identity, 1.10 = 1. Thus 1 = 10 .
(2) Let b be another inverse of a in R. Then b = b.1 = b.(a.a−1 ) = (b.a).a−1 =
1.a−1 = a−1 .

Theorem 1.17. Let R be a ring with 1 and let R∗ be the set of all multiplicative
inverse elements in R. Then (R∗ , .) is a group. It is called the group of invertible
elements.

Department of Mathematics/ College of Education/ Al-Qadisiyah University 7 of 124


Ring Theory Prof. Dr. Akeel Ramadan Mehdi Al-Yassiri

Proof. Since 1 ∈ R∗ , we have R∗ is a non-empty set.


Let a, b ∈ R∗ . Then (ab).(b−1 a−1 ) = a.1.a−1 = 1 and
(b−1 a−1 ).(ab) = b−1 .1.b = 1. Thus ab is invertible in R and hence ab ∈ R∗ .
For any a ∈ R∗ , we have 1.a = a.1 = a. Then 1 is the identity in R∗ .
Also, since a ∈ R∗ , we have a.a−1 = a−1 a = 1 and hence a−1 ∈ R∗ .
Since (R, +, .) is a ring, the multiplication (.) is an associative and hence (R∗ , .) is
a group.

Theorem 1.18. Let R be a ring. Then for all a, b, c ∈ R,


(1) a0 = 0 = 0a.
(2) a(−b) = −(ab) = (−a)b.
(3) a(b − c) = ab − ac and (a − b)c = ac − bc.
(4) (−a)(−b) = ab.
Proof. (1) a0 = a(0 + 0) = a0 + a0.
Thus a0 + (−(a0)) = a0. So, 0 = a0.
Similarly, 0 = 0a. (H.W)
(2) 0 = a0 = a(b + (−b)) = ab + a(−b). Thus, −(ab) = a(−b).
Similarly, −(ab) = (−a)b. (H.W)
(3) a(b − c) = a(b + (−c)) = ab + a(−c) = ab − ac.
Similarly, (a − b)c = ac − bc. (H.W)
(4) (−a)(−b) = −(a(−b)) (by (2))
= −(−(ab)) (by (2))
= ab.

Corollary 1.19. Let R be a ring with identity 1. If R 6= 0, then 1 6= 0.


Proof. Suppose that R 6= 0, there is 0 6= a ∈ R. Assume that 1 = 0, thus a = a.1 =
a.0 = 0 (by Theorem 1.18(1)). This contradiction implies that 1 6= 0.

Theorem 1.20. For all positive integers m and n and for all a, b in a ring R, the
following hold:
(1) am an = am+n .
(2) (am )n = amn .
(3) ma + na = (m + n)a.
(4) m(na) = (mn)a.
(5) (ma)(nb) = (mn)(ab).
Proof. Exercise.

Department of Mathematics/ College of Education/ Al-Qadisiyah University 8 of 124


Ring Theory Prof. Dr. Akeel Ramadan Mehdi Al-Yassiri

Definition 1.21. An element a in a ring R is called idempotent if a = a2 .

Definition 1.22. An element a in a ring R is called nilpotent if there exists a positive


integer n such that an = 0.

Examples 1.23. (1) Clearly, 0 and 1 (if R has unity) are idempotent elements and 0
is always nilpotent.
(2) Every non zero idempotent element is not nilpotent.
(2) 0 is the only idempotent element in a ring (2Z, +, .) and it is the only nilpotent
element.
(3) 0, 3 are idempotent elements in a ring ({0, 3}, +6 , .6 ).
(4) Every element in a ring (P(X), 4, ) is idempotent.
T
   2
0 1 0 1
(5) The element in a 2 × 2 matrix ring is nilpotent because =
0 0 0 0
 
0 0
.
0 0

Exercise 7. Show that a ring R has no nonzero nilpotent element if and only if 0 is
the only solution of x2 = 0 in R.

Theorem 1.24. If every element in a ring R is idempotent, then:


(1) a + a = 0.
(2) R is a commutative ring.
Proof. (1) Let a ∈ R. Then a + a = (a + a)2 = (a + a)(a + a) =
a2 + a2 + a2 + a2 = a + a + a + a and hence a + a = 0.
(2) Let a, b ∈ R. Then
a + b = (a + b)2 = (a + b)(a + b) = a2 + ab + ba + b2 = a + ab + ba + b.
Hence ab + ba = 0 = ab + ab (by (1) above).
By cancellation law of addition, we have ab = ba. Therefore, R is a commutative
ring.

Theorem 1.25. Let (R, +, .) be a ring with identity 1. Then (ab)2 = a2 b2 for any
a, b ∈ R if and only if R is a commutative ring.

Department of Mathematics/ College of Education/ Al-Qadisiyah University 9 of 124


Ring Theory Prof. Dr. Akeel Ramadan Mehdi Al-Yassiri

Proof. (⇒) Let a, b ∈ R. Then


(a(b + 1))2 = a2 (b + 1)2 (by hypothesis)⇒ (ab + a)2 = a2 (b2 + 2b + 1)
⇒ (ab)2 + aba + a2 b + a2 = a2 b2 + 2a2 b + a2 ⇒ aba = a2 b
By replacing a by a + 1, we have
(a + 1)b(a + 1) = (a + 1)2 b ⇒ (ab + b)(a + 1) = (a2 + 2a + 1)b
⇒ aba + ab + ba + b = a2 b + 2ab + b
⇒ aba + ab + ba + b = aba + 2ab + b.
By cancellation law of addition, we have ab = ba and hence R is a commutative ring.
(⇐) Exercise.

Exercise 8. Prove that a ring (R, +, .) is commutative if and only if (a + b)2 = a2 +


2(a.b) + b2 , for all a, b ∈ R.

Department of Mathematics/ College of Education/ Al-Qadisiyah University 10 of 124


Ring Theory Prof. Dr. Akeel Ramadan Mehdi Al-Yassiri

1.2 Subrings
Definition 1.26. Let (R, +, .) be a ring, and S a nonempty subset of R. If (S, +, .) is
also a ring under the same operations as R, then S is called a subring of R.

Examples 1.27. (1) Every ring R has two trivial subrings, 0 and R.
(2) (Z, +, .) is a subring of the ring (Q, +, .).
(3) (Q, +, .) is a subring of the ring (R, +, .).
(4) (nZ, +, .) is a subring of the ring (Z, +, .), for all n ∈ Z+ .

Theorem 1.28. A non-empty subset S of a ring (R, +, .) is a subring if and only if


for all a, b ∈ S we have a − b ∈ S and a.b ∈ S.
Proof. (⇒) Follows from the definition of a subring.
(⇐). The condition a − b ∈ S for all a, b ∈ S implies (S, +) is an additive subgroup
of the group (R, +).
Because the operation (.) is an associative in R, it also associative in S. Hence the
condition a.b ∈ S for all a, b ∈ S implies (S, .) is a semigroup.
Because the two distributive laws hold in R, they also hold in S. Hence, S is a subring
of (R, +, .).

Example 1.29.Let R = M2×2 (Z) be the ring of all 2 × 2 matrices over a ring Z.
a 0
Let S = { | a, b ∈ Z}. Show that whether (S, +, .) is a subring of R or not and
0 b
why?.
 
0 0
Proof. Since 0 ∈ Z, we have ∈ S and hence S is a non-empty subset of R.
0 0
   
a 0 c 0
Let A, B ∈ S. Thus A = and B = , where a, b, c, d ∈ Z. So, we have:
0 b 0 d
     
a 0 c 0 a−c 0
A−B = − = ∈ S (because a − c, b − d ∈ Z).
0 b 0 d 0 b−d
     
a 0 c 0 ac 0
Also, A.B = . = ∈ S (because ac, bd ∈ Z).
0 b 0 d 0 bd
By Theorem 1.28, (S, +, .) is a subring of R.


Example 1.30. Let S = {a+b 2 | a, b ∈ Z}. Show that whether (S, +, .) is a subring
of (R, +, .) or not and why?.

Department of Mathematics/ College of Education/ Al-Qadisiyah University 11 of 124


Ring Theory Prof. Dr. Akeel Ramadan Mehdi Al-Yassiri


Proof. Since
√ 0 = 0√+ 0 2 ∈ S, we have S is a non-empty subset of R.
Let a + b 2,√c + d 2 ∈ S.
√ √
Thus (a + b √2) − (c +√d 2) = (a − c) + (b − d) 2 √ ∈ S (because a − c, b − d ∈ Z).
Also, (a + b 2).(c + d 2) = (ac + 2bd) + (ad + bc) 2 ∈ S (because ac + 2bd, ad +
bc ∈ Z).
By Theorem 1.28, (S, +, .) is a subring of (R, +, .).

Example 1.31. Let S = {0, 3, 6, 9}. Show that (S, +12 , .12 ) is a subring of the ring
(Z12 , +12 , .12 ).
Proof. Clearly, S is a non-empty subset of Z12 .
Consider the following two tables:

−12 0 3 6 9 .12 0 3 6 9
0 0 9 6 3 0 0 0 0 0
3 3 0 9 6 3 0 9 6 3
6 6 3 0 9 6 0 6 0 6
9 9 6 3 0 9 0 3 6 9

From the tables, we have a −12 b, a.12 b ∈ S, for all a, b ∈ S. By Theorem 1.28,
(S, +12 , .12 ) is a subring of the ring (Z12 , +12 , .12 ).

Proposition 1.32. The intersection of any two subrings of a ring R is a subring of R.


Proof. Let S1 and S2 be two subrings of a ring R.
Thus 0 ∈ S1 , 0 ∈ S2 and hence 0 ∈ S1 S2 .
T

Since S1 ⊆ R and S2 ⊆ R, we have S1 S2 ⊆ R and hence S1 S2 is a non-empty


T T

subset of R.
Let a, b ∈ S1 S2 , thus a, b ∈ S1 and a, b ∈ S2 .
T

Since S1 and S2 are subrings of R, we have a − b, a.b ∈ S1 and a − b, a.b ∈ S2 .


Then a − b, a.b ∈ S1 S2 and hence S1 S2 is a subring of R (by Theorem 1.28).
T T

Definition 1.33. Let R be a ring. The center of a ring R is denoted by cent(R) and
defined as follows: cent(R) = {x ∈ R | xr = rx, for all r ∈ R}.

Proposition 1.34. Let R be a ring. Then the center of a ring R is a subring of R.

Department of Mathematics/ College of Education/ Al-Qadisiyah University 12 of 124


Ring Theory Prof. Dr. Akeel Ramadan Mehdi Al-Yassiri

Proof. Since 0.r = r.0 = 0 for any r ∈ R, we have 0 ∈ cent(R) and hence cent(R) is a
non-empty subset of R.
Let x, y ∈ cent(R), thus xr = rx and yr = ry, for all r ∈ R.
For any r ∈ R, we have:
(x − y)r = xr − yr = rx − ry = r(x − y) and
(xy)r = x(yr) = x(ry) = (xr)y = (rx)y = r(xy).
Thus x − y, xy ∈cent(R) and hence cent(R) is a subring of R (by Theorem 1.28).

Exercise 9. Prove that the intersection of any family of subrings of a ring R is a


subring of R.

Exercise 10. Show that whether the union of any two subrings of a ring R is a subring
of R or not and why?.

Exercise 11.  R = M2×2 (Z) be the ring of all 2 × 2 matrices over a ring Z.
Let
a 0
Let S = { | a, b ∈ Z}. Show that whether (S, +, .) is a subring of R or not and
b c
why?.

Exercise 12. Prove that every subring of a commutative ring is commutative.

Exercise 13. Give an example of non-commutative ring contains a commutative su-


bring with proof.

Exercise 14. Give an example of a ring with identity (R, +, .) such that R contains a
subring without identity with proof.

Exercise 15. Give an example of a ring without identity (R, +, .) such that R contains
a subring has an identity with proof.

Exercise 16. Give an example of a ring (R, +, .) with identity 1R and a subring
(S, +, .) with identity 1S such that 1R 6= 1S with proof.

Department of Mathematics/ College of Education/ Al-Qadisiyah University 13 of 124


Ring Theory Prof. Dr. Akeel Ramadan Mehdi Al-Yassiri

1.3 Integral Domains

Definition 1.35. A non-zero element x in a ring R is called a left zero divisor if there
exists a nonzero element y ∈ R such that xy = 0.
A non-zero element x in a ring R is called a right zero divisor if there exists a nonzero
element y ∈ R such that yx = 0.
A non-zero element x in a ring R is called a zero divisor if it is a left and right zero
divisor.

Lemma 1.36. Let x ∈ Zn . Then the following statements are equivalent:


(1) x is a zero divisor.
(2) x 6= 0 and gcd(x, n) 6= 1.

Theorem 1.37. The ring (Z p , + p , . p ) has no zero divisor if and only if p is a prime
integer number.
Proof. (⇒) Suppose that the ring (Z p , + p , . p ) has no zero divisor.
Assume that p is not prime, thus there are n, m ∈ Z+ with 1 < n, m < p and p = n.m.
Thus n. p m = nm = p = 0 with n 6= 0 and m 6= 0.
Hence n and m are zero divisors in (Z p , + p , . p ) and this is a contradiction.
Therefore, p is prime.
(⇐) Suppose that p is a prime number. By Definition 1.35, 0 is a non zero divisor.
Let 0 6= x ∈ Z p . Since gcd(x, p) = 1, we have from Lemma 1.36 that x is a non zero
divisor. Hence the ring (Z p , + p , . p ) has no zero divisor.

Example 1.38. Find all zero divisors in a ring (Z4 , +4 , .4 ).


Solution: 0 is a non zero divisor (by Definition 1.35).
Since gcd(1, 4) = 1 and gcd(3, 4) = 1, we have from Lemma 1.36 that 1 and 3 are
non zero divisors.
Since gcd(2, 4) = 2 6= 1, , we have from Lemma 1.36 that 2 is a zero divisor and
2.4 2 = 0. Thus the zero divisor in a ring (Z4 , +4 , .4 ) is only 2.

Example 1.39. Find all zero divisors in a ring (Z8 , +8 , .8 ).


Solution: 0 is a non zero divisor (by Definition 1.35).
Since gcd(1, 8) = 1, gcd(3, 8) = 1, gcd(5, 8) = 1 and gcd(7, 8) = 1, we have from
Lemma 1.36 that 1, 3, 5, 7 are non zero divisors.
Since gcd(2, 8) = 2 6= 1, gcd(4, 8) = 4 6= 1 and gcd(6, 8) = 2 6= 1, we have from

Department of Mathematics/ College of Education/ Al-Qadisiyah University 14 of 124


Ring Theory Prof. Dr. Akeel Ramadan Mehdi Al-Yassiri

Lemma 1.36 that 2, 4, 6 are zero divisors in (Z8 , +8 , .8 ) and 2.8 4 = 6.8 4 = 0.
Thus the zero divisors in a ring (Z8 , +8 , .8 ) are {2, 4, 6}.

Example 1.40. Find all zero divisors in a ring (Z24 , +24 , .24 ).
Solution: From Lemma 1.36, we have that the zero divisors in a ring (Z24 , +24 , .24 )
are: 2, 3, 4, 6, 8, 9, 10, 12, 14, 15, 16, 18, 20, 21, 22
   
0 0 1 0
Example 1.41. Let R be a ring with identity 1. Then and are zero divi-
0 1 0 0
   
0 0 1 0
sors in the ring M2×2 (R) of all 2×2 matrices over a ring R, (because . =
0 1 0 0
     
0 0 1 0 0 0
= . ).
0 0 0 0 0 1

Proposition 1.42. Let R be a non zero ring with identity 1. Then every unit element
(element has a multiplicative inverse) a in R is a non zero divisor.
Proof. Let a be a unit element in R, thus there is b ∈ R with ba = 1.
Assume that a is a zero divisor, there is 0 6= c ∈ R with ac = 0.
Then c = 1.c = (ba)c = b(ac) = b.0 = 0 and this is a contradiction.
Thus every unit element in R is a non zero divisor.

Corollary 1.43. Let R be a non zero ring with identity 1, let R∗ be the set of all unit
elements in R and let R+ be the set of zero divisor elements in R. Then R∗ R+ = φ .
T

Proof. Assume that R∗ R+ 6= φ . There is a ∈ R∗ R+ and hence a is unit element


T T

and zero divisor and this contradicts Proposition 1.42.

Definition 1.44. Let R be a ring. We say that R satisfies the cancellation laws for
multiplication if for any a, b, c ∈ R such that a 6= 0 and ab = ac or ba = ca, then
b = c.

Example 1.45. The ring (Z4 , +4 , .4 ) does not satisfy the cancellation laws for mul-
tiplication, since 2.4 2 = 2.4 0 but 2 6= 0.

Theorem 1.46. A ring R is without zero divisors if and only if R satisfies the can-
cellation laws for multiplication.

Department of Mathematics/ College of Education/ Al-Qadisiyah University 15 of 124


Ring Theory Prof. Dr. Akeel Ramadan Mehdi Al-Yassiri

Proof. (⇒) Let a, b, c ∈ R such that a 6= 0 and ab = ac or ba = ca.


Thus ab − ac = 0 or ba − ca = 0 and hence a(b − c) = 0 or (b − c)a = 0.
As a is non-zero and R is without zero divisors so b − c = 0.
Hence R satisfies the cancellation laws for multiplication.
(⇐) Let a, b ∈ R. If a 6= 0 and a.b = 0, then a.b = a.0 and by our hypothesis b = 0.
On the other hand, if b 6= 0 and a.b = 0, then a.b = 0.b. By hypothesis, a = 0.
Thus R is a ring without zero divisors.

Corollary 1.47. Let R be a ring with identity which has no zero divisors. Then the
only solutions of the equation x2 = x are x = 0 and x = 1.
Proof. Let a be a solution of the equation x2 = x with a 6= 0, thus a2 = a = a.1.
By Theorem 1.46, R satisfies the cancellation laws for multiplication and
hence a = 1.

Definition 1.48. An integral domain is a commutative ring with identity which does
not have zero divisors.

Corollary 1.49. If R is an integral domain, then:


(1) R satisfies the cancellation laws for multiplication.
(2) 0 and 1 are the only idempotent elements in R.
Proof. (1) By Theorem 1.46.
(2) By Corollary 1.47.

Example 1.50. The ring (Z p , + p , . p ) is an integral domain, for any prime number p
(by Theorem 1.37).

Example 1.51. The rings (Z6 , +6 , .6 ) and 2Z are not integral domains.

Examples 1.52. The rings Z, Q, R, C are integral domains.

Exercise 17. Prove that if R is a ring without zero divisors, then every subring of R
is without zero divisors.

Department of Mathematics/ College of Education/ Al-Qadisiyah University 16 of 124


Ring Theory Prof. Dr. Akeel Ramadan Mehdi Al-Yassiri

Exercise 18. Show that whether if R is an integral domain, then every subring of R
is an integral domain or not and why?.

Exercise 19. Let R be an integral domain. Prove that every subring of R with identity
is an integral domain.

Exercise 20. Let R be an integral domain. Prove that the only solutions of the equa-
tion x2 = 1 are either x = 1 or x = −1.

Exercise 21. If the set X contains more than one element, prove that every nonempty
proper subset of X is a zero divisor in the ring (P(X), 4, ).
T

Exercise 22. Prove that in an integral domain the zero element is the only nilpotent
element.

Department of Mathematics/ College of Education/ Al-Qadisiyah University 17 of 124


Ring Theory Prof. Dr. Akeel Ramadan Mehdi Al-Yassiri

1.4 Direct Sums of Rings


Example 1.53. Let R and S be two rings with identity, and let 1R and 1S denote
the multiplicative identities of R and S, respectively. Let R × S = {(r, s) | r ∈ R and
s ∈ S}. Define addition and multiplication on the set R × S by:
(r1 , s1 ) + (r2 , s2 ) = (r1 + r2 , s1 + s2 ) and (r1 , s1 ).(r2 , s2 ) = (r1 r2 , s1 s2 ), for all
(r1 , s1 ), (r2 , s2 ) ∈ R × S (That is, the operations are defined "componentwise".)
Then (R × S, +, .) is a ring with identity (1R , 1S ), called the direct sum (or direct
product) ring of R and S.
Proof. (A) We will prove that (R × S, +) is an abelian group.
Let a, b, c ∈ R × S. Then we have a = (r1 , s1 ), b = (r2 , s2 ) and c = (r3 , s3 ), for some
r1 , r2 , r3 ∈ R and s1 , s2 , s3 ∈ S. Thus
- (a + b) + c = ((r1 , s1 ) + (r2 , s2 )) + (r3 , s3 ) = (r1 + r2 , s1 + s2 ) + (r3 , s3 ) = ((r1 +
r2 ) + r3 , (s1 + s2 ) + s3 ) = (r1 + (r2 + r3 ), s1 + (s2 + s3 )) = (r1 , s1 ) + (r2 + r3 , s2 + s3 )
= (r1 , s1 ) + ((r2 , s2 ) + (r3 , s3 )) = a + (b + c).
- Let 0R and 0S be the zero elements of R and S, and let a ∈ R × S. We have a = (r, s)
for some r ∈ R, s ∈ S.
Now a + (0R , 0S ) = (r, s) + (0R , 0S ) = (r + 0R , s + 0S ) = (r, s) = a, and similarly
(0R , 0S ) + a = a. Thus (0R , 0S ) is a zero element for R × S.
- Let a ∈ R × S. There exist r ∈ R and s ∈ S with a = (r, s). Let b = (−r, −s),
thus a + b = (r, s) + (−r, −s) = (r + (−r)), (s + (−s)) = (0R , 0S ) Similarly b + a =
(0R , 0S ), and since (0R , 0S ) is the zero element of R×S this shows that b is an additive
inverse of a.
- Let a, b ∈ R × S. Then we have a = (r1 , s1 ), b = (r2 , s2 ), for some r1 , r2 ∈ R and
s1 , s2 ∈ S.
Thus a + b = (r1 , s1 ) + (r2 , s2 ) = (r1 + r2 , s1 + s2 ) = (r2 + r1 , s2 + s1 ) = (r2 , s2 ) +
(r1 , s1 ) = b + a. Therefore, (R × S, +) is an abelian group.
The other conditions are left exercise.

Example 1.54. More generally, if {Ri }i∈I is a family of rings, then ∏i∈I Ri is a ring
with coordinatewise operations, called the direct product ring of {Ri }i∈I .

Example 1.55. Consider the ring R = Z × Z of ordered pairs of integers with coor-
dinatewise operations, i.e. (a, b) + (c, d) = (a + c, b + d) and (a, b) · (c, d) = (ac, bd).
Then the diagonal ring D = {(a, a) | a ∈ Z} is a subring of R: it contains the additive
identity (0, 0), the multiplicative identity (1, 1) and is closed under multiplication
and addition.

Department of Mathematics/ College of Education/ Al-Qadisiyah University 18 of 124


Ring Theory Prof. Dr. Akeel Ramadan Mehdi Al-Yassiri

Proposition 1.56. Let R be a non zero ring. Then the ring R × R has a zero divisor.
Proof. Since R 6= 0, there is a non zero element x ∈ R. Let a = (x, 0) and b = (0, x),
thus a 6= (0, 0) and b 6= (0, 0). Since ab = ba = 0, we have that a and b are zero
divisors in the ring R × R.

Exercise 23. Let R and S be two commutative rings with identity. Prove that the ring
(R × S, +, .) is commutative with identity.

Exercise 24. Let R and S be two integral domains. Show that whether the ring
(R × S, +, .) is an integral domain or not and why?.

Department of Mathematics/ College of Education/ Al-Qadisiyah University 19 of 124


Ring Theory Prof. Dr. Akeel Ramadan Mehdi Al-Yassiri

1.5 Characteristic of rings


Definition 1.57. The characteristic of a ring R is the least positive integer n such that
nx = 0 for all x ∈ R. If no such integer exists, we say that R has characteristic 0. The
characteristic of R is denoted by char(R).

Example 1.58. Let R = {0, 2, 4, 6, 8} be a subring of the ring (Z10 , +10 , .10 ). Find
the characteristic of R with proof.
Solution: Since R = {0, 2, 4, 6, 8} is a subring of Z10 , then R is a ring under addition
and multiplication modulo 10.
Choose the positive integer n = 1. Take any 0 6= x ∈ R, we got that 1x 6= 0
Choose the positive integer n = 2. Take any 0 6= x ∈ R, we got that 2x 6= 0
Choose the positive integer n = 3. Take any 0 6= x ∈ R, we got that 3x 6= 0
Choose the positive integer n = 4. Take any 0 6= x ∈ R, we got that 4x 6= 0
Choose the positive integer n = 5. Take any x ∈ R, we got that 5x = 0
So, n = 5 is the least positive integer such that nx = 0 for all x ∈ R. Then we conclude
that char(R) = 5.

Example 1.59. The rings of integer numbers Z, rational numbers Q and real num-
bers R are having zero characteristic.

Example 1.60. Let X be a nonempty set contains more one element and P(X) the
power set of X. The ring (P(X), 4, ) is of characteristic two.
T

Proof. Let A ∈ R = P(X). Then 2A = A 4 A = (A − A) (A − A) = φ . Thus 2 is the


S

least positive integer such that 2A = 0 for all A ∈ R and hence 2 is the characteristic
of R.

Theorem 1.61. Let (R, +, .) be a ring with identity 1R . Then R has characteristic
n > 0 if and only if n is the least positive integer such that n1R = 0.
Proof. (⇒) Suppose that R has characteristic n > 0. Thus n is the least positive
integer such that nx = 0, for all x ∈ R and hence n1R = 0.
Assume that there is a positive integer m such that 0 < m < n with m1R = 0.
Then mx = m(1R .x) = (m1R ).(1x) (by Theorem 1.20(5))
= (m1R ).x = 0.x = 0 and this contradicts that n is the least positive integer such that
nx = 0, for all x ∈ R.
Thus n is the least positive integer such that n1 = 0.
(⇐) Suppose that n is the least positive integer such that n1 = 0.

Department of Mathematics/ College of Education/ Al-Qadisiyah University 20 of 124


Ring Theory Prof. Dr. Akeel Ramadan Mehdi Al-Yassiri

Let a ∈ R. Then
na = (n.1)(1R .a) = (n.1R )(1.a) (by Theorem 1.20)
= (n.1R ).a = 0.a = 0.
Assume that there is m ∈ Z+ with m < n and ma = 0, for all a ∈ R.
Thus m.1 = 0 and m < n and this is a contradiction with the hypothesis.
Thus R has characteristic n > 0.

Example 1.62. The characteristic of the ring (Zn , +n , .n ) is n.


Proof. Clearly, we have n.1 = n = 0.
Let m ∈ Z+ with m < n. Then m.1 = m 6= 0.
Thus n is the least positive integer such that n.1 = 0.
By Theorem 1.61, n is the characteristic of the ring (Zn , +n , .n ).

Corollary 1.63. Let R be an integral domain with characteristic n > 0. Then the
additive order of any non-zero element a in R is n.
Proof. Let R be an integral domain with characteristic n > 0 and let 0 6= a ∈ R with
additive order m.Thus m ≤ n.
Since m.a = 0, we have m.a = (m.1R ).a = 0.
Since R is an integral domain and a 6= 0, we have m.1R = 0.
Since n is the characteristic of R (by hypothesis), it follows from Theorem 1.61 that
n is the least positive integer such that n.1R = 0. Thus n = m.

Corollary 1.64. The characteristic of any integral domain is either zero or prime
number.
Proof. Let R be an integral domain of characteristic n > 0.
Assume that n is not prime number, thus n = ab for some a, b ∈ Z + with 1 < a, b < n.
It follows that 0 = n1R (by Theorem 1.61)
= (ab)1R = (a1R )(b1R ).
Since R has no zero divisor, a1R = 0 or b1R = 0.
Then a = n or b = n by Theorem 1.61 and this is a contradiction.
Hence, n is a prime number.

Exercise 25. For every n > 1, show that there exists at least one ring of characteristic
n.

Department of Mathematics/ College of Education/ Al-Qadisiyah University 21 of 124


Ring Theory Prof. Dr. Akeel Ramadan Mehdi Al-Yassiri

2 Ideals and Quotient Rings


2.1 Ideals
Definition 2.1. A nonempty subset I of a ring R is said to be a left ideal of R if
1) a − b ∈ I, for any a, b ∈ I.
2) ra ∈ I, for any a ∈ I and r ∈ R.

Definition 2.2. A nonempty subset I of a ring R is said to be a right ideal of R if


1) a − b ∈ I, for any a, b ∈ I.
2) ar ∈ I, for any a ∈ I and r ∈ R.

Definition 2.3. A nonempty subset I of a ring R is said to be an ideal (or two-sided


ideal) of R if
1) a − b ∈ I, for any a, b ∈ I.
2) ra ∈ I and ar ∈ I, for any a ∈ I and r ∈ R.
That is, a nonempty subset I of a ring R is an ideal of R if I is both a left and a right
ideal of R.

Definition 2.4. An ideal I of R is said to be a proper ideal if I 6= R.

Remarks 2.5. (1) {0} and R are ideals of any ring R, called the trivial ideals.
(2) If R is a commutative ring, all the right (resp. left) ideals of R are ideals.
(3) Every left (resp. right) ideal of a ring is a subring.
The converse is not in general, for example: (Z, +, .) is a subring of (Q, +, .) but
(Z, +, .) is not a left and a right ideal of (Q, +, .), since 1 ∈ Z and 1/2 ∈ Q but
(1/2).1 = 1.(1/2) = 1/2 ∈ / Z.

Proposition 2.6. Let R be a ring and let a be an element of R.


Then Ra = {ra | r ∈ R} is a left ideal of R.
Proof. Since 0 = 0.a, we have 0 ∈ Ra and hence φ 6= Ra ⊆ R.
Let x, y ∈ Ra and r ∈ R. Thus x = s1 a and y = s2 a, for some s1 , s2 ∈ R.
Then x − y = s1 a − s2 a = (s1 − s2 )a ∈ Ra and rx = r(s1 a) = (r.s1 )a ∈ Ra.
Hence Ra = {ra | r ∈ R} is a left ideal of R.

Proposition 2.7. Let R be a ring and let a be an element of R.


Then aR = {ar | r ∈ R} is a right ideal of R.

Department of Mathematics/ College of Education/ Al-Qadisiyah University 22 of 124


Ring Theory Prof. Dr. Akeel Ramadan Mehdi Al-Yassiri

Example 2.8. The set 2Z = {2r | r ∈ Z} is an ideal of the ring Z. In general, for any
n ∈ Z, we have nZ = {nr | r ∈ Z} is an ideal of the ring Z.

Example 2.9. In the ring of integers Z every subring is an ideal.


To see this, let I be a subring of Z and a ∈ I, r ∈ Z.
{z... + a} and hence ra ∈ I.
If r > 0, then ra = a| + a +
n times
If r = 0, then ra = 0 and hence ra ∈ I.
If r < 0, then ra = −a
| − a{z− ... − a} and hence ra ∈ I.
n times
Since Z is a commutative ring, we have I is an ideal of Z.

Example 2.10. I = {0, 2, 4}. Then (I, +6 , .6 ) is an ideal of the ring (Z6 , +6 , .6 ).
Proof. Clearly, I is a non-empty subset of Z6 .
Consider the following two tables:

.6 0 2 4
0 0 0 0
−6 0 2 4
1 0 2 4
0 0 4 2
2 0 4 2
2 2 0 4
3 0 0 0
4 4 2 0
4 0 2 4
5 0 4 2

From the tables, we have for all a, b ∈ I and r ∈ Z6 that a −6 b ∈ I and r.6 a ∈ I.
Hence (I, +6 , .6 ) is a left ideal of the ring (Z6 , +6 , .6 ).
Since (Z6 , +6 , .6 ) is a commutative ring, we have (I, +6 , .6 ) is an ideal of the ring
(Z6 , +6 , .6 ).

Lemma 2.11. The ideals of the ring (Zn , +n , .n ) are taken the form (< m >, +n , .n ),
where m is a positive integer divides n.

Exercise 26. Find all ideals of the following rings: 1) (Z18 , +18 , .18 ) 2) (Z19 , +19 , .19 ).

 
a 0
Example 2.12. Let R be a ring with identity 1 and let I = { | a, b ∈ R}.
b 0
Then I is a left ideal of the ring M2×2 (R), but it is not a right ideal.

Department of Mathematics/ College of Education/ Al-Qadisiyah University 23 of 124


Ring Theory Prof. Dr. Akeel Ramadan Mehdi Al-Yassiri

 
0 0
Proof. Since ∈ I, we have φ 6= I ⊆ M2×2 (R).
0 0
     
a 0 c 0 x y
Let , ∈ I and let ∈ M2×2 (R).
b 0 d 0 z w
     
a 0 c 0 a−c 0
Then − = ∈ I and
b 0 d 0 b−d 0
     
x y a 0 xa + yb 0
. = ∈ I. Hence I is a left ideal of the ring M2×2 (R).
z w b 0 za + wb 0
 
1 0
In other hand, I is not a right ideal of the ring M2×2 (R), since if we take A = ∈
1 0
       
1 1 1 0 1 1 1 1
I and B = ∈ M2×2 (R), then AB = . = ∈
/ I.
1 1 1 0 1 1 1 1

Theorem 2.13. Let R be a ring.


(a) If I is an ideal of R, then Mn (I) is an ideal of the n × n matrices ring Mn (R).
(b) If R has an identity and K is an ideal of Mn (R), then there is some ideal I of R
such that J = Mn (I).
Proof. Exercise.

Example 2.14. Find all ideals of the ring Mn (Z10 ).


Solution: By Lemma 2.11, the ideals of the ring (Z10 , +10 , .10 ) are:
({0}, +10 , .10 ), (Z10 , +10 , .10 ), ({0, 2, 4, 6, 8}, +10 , .10 ) and ({0, 5}, +10 , .10 ).
By Theorem 2.13, the ideals of the ring Mn (Z10 ) are:
0 = Mn (0), Mn (Z10 ), Mn (< 2 >) and Mn (< 5 >).

Proposition 2.15. Let R be a ring with identity 1 and let I be a left (resp. right) ideal
of a ring R. Then I is a proper left (resp. right) ideal of R if and only if 1 ∈
/ I.
Proof. (⇒) Suppose that I is a proper left ideal of R. Thus there is r ∈ R with r ∈
/ I.
Assume that 1 ∈ I, then r = r.1 ∈ I and this is a contradiction. Thus 1 ∈ / I.
(⇐) Suppose that 1 ∈ / I, thus I 6= R and hence I is a proper left ideal of R.

Corollary 2.16. Let R be a ring with identity 1 and let I be a proper left (resp. right)
ideal of a ring R. Prove that I has no invertible element.
Proof. Exercise.

Department of Mathematics/ College of Education/ Al-Qadisiyah University 24 of 124


Ring Theory Prof. Dr. Akeel Ramadan Mehdi Al-Yassiri

Definition 2.17. A non-zero ring R is said to be a simple ring if R does not contain
any nontrivial ideals (i.e., different from 0 and R).

Example 2.18. The following rings are simple:


(1) (Z5 , +5 , .5 ) (2) (Z19 , +19 , .19 ) (3) (Z p , + p , . p ), where p > 1 is a prime number
(4) (Mn (Z23 ), +, .) (5) (M6 (Z29 ), +, .) (6) (R, +, .) (7) (Q, +, .).

Example 2.19. The following rings are not simple:


(1) (Z6 , +6 , .6 ) (2) (Z26 , +26 , .26 ) (3) (Zn , +n , .n ), where n is not a prime number
(4) (Mn (Z24 ), +, .) (5) (M6 (Z32 ), +, .) (6) (Z, +, .).

Proposition 2.20. Let R be a ring with identity and let n ∈ Z+ . Then R is a simple
ring if and only if the n × n matrices ring Mn (R) is simple.
Proof. (⇒) Suppose that R is a simple ring.
Assume that the ring Mn (R) is not simple, thus there is a non-trivial ideal J of a ring
Mn (R). By Theorem 2.13(b), there is an ideal I of R such that J = Mn (I).
Since J is a non-trivial ideal, we have I 6= 0 and I 6= R and this is a contradiction.
Thus Mn (R) is a simple ring.
(⇐) Suppose that Mn (R) is a simple ring.
Assume that the ring R is not simple, thus there is a non-trivial ideal I of a ring
R. By Theorem 2.13(a), Mn (I) is a non-trivial ideal of the ring Mn (R) and this is a
contradiction. Thus R is a simple ring.

Exercise 27. Find all ideals of the following rings:


1) (Z15 , +15 , .15 ) 2) (Z36 , +36 , .36 ) 3) (M5 (Z20 ), +, .).

Exercise 28. Give an example of a noncommutative ring contains 6 ideals only with
proof.
 
a b
Exercise 29. Let R be a ring with identity 1 and let I = { | a, b ∈ R}.
0 0
Prove that I is a right ideal of the ring M2×2 (R), but it is not a left ideal.
Exercise 30. Determine the simple and not simple rings of the following with proof?
(1) (Z7 , +7 , .7 ) (2) (Z40 , +40 , .40 ) (3) (M6 (R), +, .) (4) (M6 (Z), +, .).

Department of Mathematics/ College of Education/ Al-Qadisiyah University 25 of 124


Ring Theory Prof. Dr. Akeel Ramadan Mehdi Al-Yassiri

2.2 Some Operations on Ideals


Proposition 2.21. Let {I j } j∈∧ be a family of ideals of a ring R. Then
T
j∈∧ I j is an
ideal of R.
Proof. Since 0 ∈ I j , for all j ∈ ∧, we have 0 ∈ j∈∧ I j .
T

Since I j ⊆ R, for all j ∈ ∧, we have j∈∧ I j ⊆ R and hence j∈∧ I j is a non-empty


T T

subset of R.
Let a, b ∈ j∈∧ I j and r ∈ R, thus a, b ∈ I j , for all j ∈ ∧.
T

Since I j is an ideal of R, we have a − b, ra, ar ∈ I j , for all j ∈ ∧.


Thus a − b, ra, ar ∈ j∈∧ I j and hence j∈∧ I j is an ideal of R.
T T

Remark 2.22. In general, union of two ideals of a ring R need not be an ideal of R,
for example: < 2 > and < 3 > are two ideals of Z, but < 2 > ∪ < 3 > is not an ideal
in Z (since 2, 3 ∈< 2 > ∪ < 3 > but 3 − 2 = 1 ∈<
/ 2 > ∪ < 3 >).

Proposition 2.23. Let I1 and I2 be two ideals of a ring R. Then I1 ∪ I2 is an ideal of


R if and only if either I1 ⊆ I2 or I2 ⊆ I1 .
Proof. (⇒) Suppose that I1 ∪ I2 is an ideal of a ring R.
Assume that I1 " I2 and I2 " I1 . Then there is a ∈ I1 with a ∈ / I2 and there is b ∈ I2
with b ∈ / I1 and hence a, b ∈ I1 ∪ I2 .
Since I1 ∪ I2 is an ideal of a ring R (by hypothesis), we have a + b ∈ I1 ∪ I2 .
Hence either a + b ∈ I1 or a + b ∈ I2 .
If a + b ∈ I1 , then −a + (a + b) ∈ I1 and hence b ∈ I1 (since I1 is an ideal in R) and
this is a contradiction
If a + b ∈ I2 , then −b + (a + b) ∈ I2 and hence a ∈ I2 (since I2 is an ideal in R) and
this is a contradiction. Thus either I1 ⊆ I2 or I2 ⊆ I1 .
(⇐) Suppose that either I1 ⊆ I2 or I2 ⊆ I1 .
If I1 ⊆ I2 , then I1 ∪ I2 = I2 . Since I2 is an ideal of R (by hypothesis), we have I1 ∪ I2
is an ideal of R.
If I2 ⊆ I1 , then I1 ∪ I2 = I1 . Since I1 is an ideal of R (by hypothesis), we have I1 ∪ I2
is an ideal of R.

Definition 2.24. If X is a subset of a ring R, then < X > will denote the intersection
of all ideals of R which contain X, that is:
< X >= {I | I is an ideal of R and X ⊆ I}. This is called the ideal of R generated
T

by X, while the elements of X are called generators of < X >.

Department of Mathematics/ College of Education/ Al-Qadisiyah University 26 of 124


Ring Theory Prof. Dr. Akeel Ramadan Mehdi Al-Yassiri

Proposition 2.25. Let X be a subset of a ring R. Then < X > is the smallest ideal of
R that contains a subset X.
Proof. By Proposition 2.21, < X > is an ideal of R. Let J be an ideal of R contains
X. By Definition 2.24, < X >= {I | I is an ideal of R and X ⊆ I}. Since J is an
T

ideal of R and X ⊆ J it follows that < X >⊆ J. Thus < X > is the smallest ideal of
R that contains a subset X.

Definition 2.26. (1) An ideal I of a ring R is said to be finitely generated if it gene-


rated by a finite subset X, that is I =< X >.
(2) An ideal I of a ring R is said to be principal if it generated by a subset X = {a}
contains one element only, that is I =< {a} >.

Remark 2.27. If I is a principal ideal of a ring R generated by a subset X = {a},


then we will write I =< a >.

Examples 2.28. (1) Let X = {6, 12} be a subset of the ring (Z24 , +24 , .24 ).
Then < 6, 12 >= Z 24 ∩ < 2 > ∩ < 3 > ∩ < 6 >= {0, 6, 12, 18} =< 6 >.
(2) Let X = {2, 6} be a subset of the ring (Z8 , +8 , .8 ).
Then < X >= Z 24 ∩ < 2 >=< 2 >= {0, 2, 4, 8}.

Proposition 2.29. If R is a commutative ring with identity 1 and a ∈ R, then


(a) = Ra = {ra | r ∈ R}.
Proof. Since 1 ∈ R, we have a = 1.a ∈ Ra.
By Proposition 2.6, Ra is a left ideal of a ring R.
Since R is a commutative ring, Ra is an ideal of R contains a.
By Proposition 2.25, (a) is the smallest ideal of R contains a and hence (a) ⊆ Ra.
Conversely, let x ∈ Ra, thus x = ra, for some r ∈ R.
Since (a) is an ideal of R contains a (by Proposition 2.25), it follows that x = ra ∈ (a).
Thus Ra ⊆ (a) and hence (a) = Ra = {ra | r ∈ R}.

Example 2.30. (1) The ideal (3) in the ring (Z, +, .) is (3) = 3Z = {3r | r ∈ Z}.
(2) The ideal (3) in the ring (Z6 , +6 , .6 ) is (3) = 3Z6 = {3.6 r | r ∈ Z6 } = {0, 3}.

Example 2.31. Let (R, +, .) be the ring of real numbers. Then R =< a >, for any
non-zero element a ∈ R.

Department of Mathematics/ College of Education/ Al-Qadisiyah University 27 of 124


Ring Theory Prof. Dr. Akeel Ramadan Mehdi Al-Yassiri

Proof. Let 0 6= a ∈ R, thus Ra ⊆ R.


Let b ∈ R, thus b = b.1 = b.(a−1 .a) = (b.a−1 ).a ∈ Ra and hence R ⊆ Ra.
Then R = Ra. By Proposition 2.29, (a) = Ra and hence R =< a >.

Proposition 2.32. A ring R is simple if and only if R =< a >, for any non-zero
element a ∈ R.
Proof. (⇒) Suppose that R is a simple ring.
Assume that there is 0 6= a ∈ R such that R 6=< a >.
Thus < 0 >$< a >$ R and this is a contradiction.
Hence R =< a >, for any non-zero element a ∈ R.
(⇐) Suppose that R =< a >, for any non-zero element a ∈ R.
Assume that R is not simple, thus there is an ideal I such that < 0 >$ I $ R.
Then there is an element x ∈ I with x 6= 0 and hence R =< x > (by hypothesis).
Since < x >⊆ I, we have R ⊆ I and hence I = R and this is a contradiction.
Thus R is a simple ring.

Definition 2.33. A ring R is said to be principal ideal ring (or PIR) if R is an com-
mutative ring with identity in which every ideal is principal.

Definition 2.34. A ring R is said to be principal ideal domain (or PID) if R is a


principal ideal ring and R is an integral domain.

Examples 2.35. (1) (Z6 , +6 , .6 ) is a principal ideal ring but it is not principal ideal
domain.
(2) (Z7 , +7 , .7 ) and (R, +, .) are principal ideal domains.

Lemma 2.36. (Division Algorithm): If a, b ∈ Z with b > 0, then exist unique inte-
gers q and r such that a = qb + r and 0 ≤ r < b.

Theorem 2.37. The ring of integers (Z, +, .) is a principal ideal domain.


Proof. Let I be an ideal of Z. If I = 0, then I =< 0 >, so I is a principal ideal.
If I 6=< 0 >, then there is 0 6= n ∈ I and hence −n ∈ I.
Thus I contains a positive integer. Let m be the smallest integer such that m > 0 and
m ∈ I. We will prove that I =< m >.

Department of Mathematics/ College of Education/ Al-Qadisiyah University 28 of 124


Ring Theory Prof. Dr. Akeel Ramadan Mehdi Al-Yassiri

Since m ∈ I we have < m >⊆ I.


Conversely, let a ∈ I, thus we have from Division Algorithm that a = qm + r, for
some q, r ∈ Z with 0 ≤ r < m. This gives r = a − qm, so r ∈ I.
Since m is the smallest positive element of I, this implies that r = 0.
Therefore a = qm, and so a ∈< m > and hence I ⊆< m >.
Therefore I =< m > and hence every ideal in the ring (Z, +, .) is principal.
Since (Z, +, .) is an integral domain, we have (Z, +, .) is a principal ideal domain.

Definition 2.38. Let R be a ring and let a, b ∈ R with a 6= 0. We say that a divides
b (or b is a multiple of a), denoted by a | b, if there is an element r ∈ R such that
b = ra.
If a is not divide b, then we will write a - b.

Lemma 2.39. Let R be a commutative ring with identity and let a, b ∈ R with a 6= 0.
Then a divides b if and only if < b >⊆< a >.
Proof. (⇒) Suppose that a divides b. Then there is r ∈ R such that b = ra. Since
ra ∈ Ra =< a >, we have b ∈< a > and hence < b >⊆< a >.
(⇐) Suppose that < b >⊆< a >, thus b ∈< a > and hence there is r ∈ R such that
b = ra. Then a divides b.

Definition 2.40. Let R be a commutative ring with identity and let a1 , a2 , ..., an be
nonzero elements in R. The common multiple of a1 , a2 , ..., an is defined to be an
element d ∈ R such that ai | d (ai divides d), for all i = 1, 2, ..., n.

Definition 2.41. Let R be a commutative ring with identity and let a1 , a2 , ..., an be
nonzero elements in R. The least common multiple of a1 , a2 , ..., an , denoted by
lcm(a1 , a2 , ..., an ), is defined to be an element d ∈ R such that
(1) d is a common multiple of a1 , a2 , ..., an .
(2) If x is an another common multiple of a1 , a2 , ..., an , then d | x (d divides x).

Examples 2.42. (1) Let R be a commutative ring with identity and let a1 , a2 , ..., an be
nonzero elements in R. Then a1 .a2 ...an and −(a1 .a2 ...an ) are both common multiples
of a1 , a2 , ..., an .
(2) In the ring of integer (Z, +, .), the least common multiple of 4, 6 is 12.
(3) In the ring of integer (Z, +, .), the least common multiple of 6, 8, 20 is 120.
(4) In the ring of integer (Z, +, .), the least common multiple of 3, 8, 12 is 24.

Department of Mathematics/ College of Education/ Al-Qadisiyah University 29 of 124


Ring Theory Prof. Dr. Akeel Ramadan Mehdi Al-Yassiri

Theorem 2.43. Let a1 , a2 , ..., an be nonzero elements in a principal ideal ring (R, +, .).
Then ( ni=1 < ai >, +, .) = (< a >, +, .), where a is the least common multiple of
T

a1 , a2 , ..., an .
Proof. By Proposition 2.21, ( ni=1 < ai >, +, .) is an ideal of R.
T

Since R is a principal ideal ring, we have there is a ∈ R such that ni=1 < ai >=< a >.
T

We will prove that a is the least common multiple of a1 , a2 , ..., an .


Since < a >⊆ ni=1 < ai >, we have < a >⊆< ai >, for all i = 1, 2, ..., n.
T

By Lemma 2.39, ai divides a, for all i = 1, 2, ..., n and


hence a is a common multiple of a1 , a2 , ..., an .
Let b be an another common multiple of a1 , a2 , ..., an ,
thus b = ri .ai , where ri ∈ R (i = 1, 2, ..., n).
Let x ∈< b >, there is r ∈ R such that x = r.b.
Hence x = r.b = r.(ri .ai ) = (r.ri ).ai ∈< ai >, for each i = 1, 2, ..., n.
Then < b >⊆< ai >, for each i = 1, 2, ..., n and
hence < b >⊆ ni=1 < ai >=< a >.
T

By Lemma 2.39, a divides b and hence a is the least common multiple of a1 , a2 , ..., an .

Examples 2.44. (1) In the ring of integer (Z, +, .), we have


(< 4 > ∩ < 6 >, +, .) = (< 12 >, +, .), where 12 the least common multiple of 4, 6.
(2) In the ring of integer (Z, +, .), we have
(< 6 > ∩ < 8 > ∩ < 20 >, +, .) = (< 120 >, +, .), where 120 the least common
multiple of 6, 8, 20.

Definition 2.45. Let I1 and I2 be two ideals of a ring R.


(1) The sum of I1 and I2 is defined by I1 + I2 = {a + b|a ∈ I1 and b ∈ I2 }.
(2) The product of I1 and I2 is defined by I1 .I2 = {∑ni=1 ai .bi |ai ∈ I1 , bi ∈ I2 , n ∈ Z+ }.

Proposition 2.46. Let I1 and I2 be two ideals of a ring R. Then I1 + I2 and I1 .I2 are
ideals of R.
Proof. I1 + I2 is an ideal of R (Exercise).
Let x, y ∈ I1 .I2 and r ∈ R. Then x = ∑ni=1 ai .bi and y = ∑mj=1 c j .d j , where ai , c j ∈ I1
and bi , d j ∈ I2 .
Thus x − y = ∑ni=1 ai .bi − ∑mj=1 c j .d j = ∑ni=1 ai .bi + ∑mj=1 (−c j ).d j ∈ I1 .I2 .
Also, rx = r(∑ni=1 ai .bi ) = ∑ni=1 (rai ).bi ∈ I1 .I2 and
xr = (∑ni=1 ai .bi )r = ∑ni=1 ai .(bi r) ∈ I1 .I2 .
Therefore, I1 .I2 is an ideal of R.

Department of Mathematics/ College of Education/ Al-Qadisiyah University 30 of 124


Ring Theory Prof. Dr. Akeel Ramadan Mehdi Al-Yassiri

Theorem 2.47. If A, B,C are ideals of a ring R with B ⊆ C, then


(A + B) ∩C = (A ∩C) + (B ∩C) = (A ∩C) + B.
Proof. (1) We will prove that (A + B) ∩C = (A ∩C) + B.
Let x ∈ (A + B) ∩C, thus x ∈ A + B and x ∈ C and hence x = a + b, where a ∈ A and
b ∈ B. Since B ⊆ C it follows that x, b ∈ C and hence x − b ∈ C.
Since a ∈ A and a = x − b it follows that a ∈ A ∩ C and hence a + b ∈ (A ∩ C) + B.
Thus x ∈ (A ∩C) + B and this implies that (A + B) ∩C ⊆ (A ∩C) + B.
Since A ∩C ⊆ C it follows that (A ∩C) + B ⊆ C + B.
Since A ∩C ⊆ A it follows that (A ∩C) + B ⊆ A + B and
hence (A ∩C) + B ⊆ (A + B) ∩ (C + B) = (A + B) ∩C.
Therefore, (A + B) ∩C = (A ∩C) + B.
(2) We will prove that (A ∩C) + (B ∩C) = (A ∩C) + B.
Since B ⊆ C it follows that B ∩C = B and hence (A ∩C) + (B ∩C) = (A ∩C) + B.
From (1) and (2) we get that (A + B) ∩C = (A ∩C) + (B ∩C) = (A ∩C) + B.

Remarks 2.48. (1) In Theorem 2.47, if we remove the condition B ⊆ C, then we


already have (A ∩C) + (B ∩C) ⊆ (A + B) ∩C.
Proof. Since A ⊆ A + B it follows that A ∩C ⊆ (A + B) ∩C. Also, since B ⊆ A + B it
follows that B ∩C ⊆ (A + B) ∩C and hence (A ∩C) + (B ∩C) ⊆ (A + B) ∩C.
(2) The reverse inclusion in (1) above does not necessarily hold, for example:
(Exercise).

Definition 2.49. (Direct summand) An ideal I of a ring R is said to be a direct


summand of R if there is an ideal J of R such that R = I + J and I ∩ J = 0. Then
we will write R = I ⊕ J.

Example 2.50. Find all direct summands of the ring (Z6 , +6 , .6 ).


Proof. (Z6 , +6 , .6 ), < 0 >, I1 =< 2 >= {0, 2, 4} and I2 =< 3 >= {0, 3} are all direct
summands of the ring (Z6 , +6 , .6 ).

Example 2.51. Find all direct summands of the ring (Z8 , +8 , .8 ).


Proof. (Z8 , +8 , .8 ) and < 0 > are all direct summands of the ring (Z8 , +8 , .8 ).

Example 2.52. Find all direct summands of the ring (Z12 , +12 , .12 ).

Department of Mathematics/ College of Education/ Al-Qadisiyah University 31 of 124


Ring Theory Prof. Dr. Akeel Ramadan Mehdi Al-Yassiri

Proof. (Z12 , +12 , .12 ), < 0 >, I1 =< 4 >= {0, 4, 8} and I2 =< 3 >= {0, 3, 6, 9} are
all direct summands of the ring (Z12 , +12 , .12 ).

Example 2.53. If R is a simple ring, then 0 and R are all direct summands in R.

Example 2.54. < 0 > and Z are the only direct summands of the ring of integers
(Z, +, .).
Proof. Assume that I is a direct summand of Z with I 6= 0 and I 6= Z. Thus I =< n >
with n ∈ Z and n 6= 0, n 6= 1, n 6= −1. Since I is a direct summand of Z, there is
an ideal J =< m > of Z for some m ∈ Z such that Z =< n > ⊕ < m > and hence
< n > ∩ < m >= 0. Since nm ∈< n > ∩ < m >, we have nm = 0. Since n 6= 0 and
Z is an integral domain, we have m = 0. Since Z =< n > + < m >, it follows that
Z =< n >. Since either Z =< 1 > or Z =< −1 >, we have either n = 1 or n = −1
and this is a contradiction. Thus < 0 > and Z are the only direct summands of the
ring (Z, +, .).

Exercise 31. Let I1 and I2 be two ideals of a ring R. Prove that I1 + I2 =< I1 ∪ I2 >.

Exercise 32. Let I1 , I2 , ..., In be ideals of a ring R. Prove that I1 + I2 + ... + In is an


ideal of R, where I1 + I2 + ... + In = {a1 + a2 + ... + an |ai ∈ Ii , for all i = 1, 2, ..., n}.

Exercise 33. If I1 , I2 are two ideals of a ring R such that I1 ∩ I2 =< 0 >, prove that
a.b = 0 for every a ∈ I1 , b ∈ I2 and then prove that I.J =< 0 >.

Exercise 34. Let I be an ideal of a ring R. Define [R : I] as follows:


[R : I] = {x ∈ R | rx ∈ I, for all r ∈ R}. Prove that [R : I] is an ideal of R contains I.

Exercise 35. Let R be a commutative ring with identity 1 and let a ∈ R be an idem-
potent element. Prove that R =< a > ⊕ < 1 − a >.

Exercise 36. Let A, B, and C be ideals of a commutative ring R. Prove the following:
(1) A.(B.C) = (A.B).C, (2) A.B = B.A, (3) A.B ⊆ A ∩ B,
(4) A.(B+C) = A.B+A.C, (5) A ⊆ B implies A.C ⊆ B.C, (6) A.(B∩C) ⊆ A.B∩A.C.

Department of Mathematics/ College of Education/ Al-Qadisiyah University 32 of 124


Ring Theory Prof. Dr. Akeel Ramadan Mehdi Al-Yassiri

2.3 Quotient Rings


Definition 2.55. Let I be an ideal of a ring R and let a ∈ R. We define the subset
a + I of R by a + I = {a + b|b ∈ I} and it is called an additive coset of I in R.

Lemma 2.56. Let I be an ideal of a ring R and let a, b ∈ R. Then a + I = b + I if and


only if a − b ∈ I.

Proposition 2.57. Let I be an ideal of a ring R.


Then the set R/I = {a + I|a ∈ R} of additive cosets a + I of I in R forms a ring with
respect to coset addition

(a + I) + (b + I) = (a + b) + I
and coset multiplication

(a + I).(b + I) = (a.b) + I,
for all a + I, b + I ∈ R/I.
Proof. Since (R, +) is an abelian group and (I, +) is a subgroup of (R, +), we have
(I, +) is a normal subgroup of (R, +) and hence (R/I, +) is an abelian group.
Now we will prove that (R/I, .) is a semigroup.
At first, we will see that the multiplication (.) is well defined.
0 0 0 0
That is, we need to show that if a + I = a + I and b + I = b + I, then ab + I = a b + I.
0 0 0 0
Since a+I = a +I and b+I = b +I, we have a−a ∈ I and b−b ∈ I (by Lemma 2.56).
0 0 0 0 0 0 0 0 0
Since ab − a b = ab − ab + ab − a b = a(b − b ) + (a − a )b ∈ I, we have
0 0
ab + I = a b + I (by Lemma 2.56). Hence the multiplication (.) is well defined.
Let a + I, b + I ∈ R/I. Then (a + I).(b + I) = ab + I ∈ R/I and hence (R, .) is a ma-
thematical system.
Let a + I, b + I, c + I ∈ R/I.
Then (a + I).((b + I).(c + I)) = (a + I).(b.c + I)
= a.(b.c) + I = (a.b).c + I
= (a.b + I).(c + I) = ((a + I).(b + I)).(c + I).
Thus the multiplication (.) is an associative and hence (R/I, .) is a semigroup.
Now we will prove that the multiplication (.) distributes on (+). (Exercise)
Therefore, (R/I, +, .) is a ring.

Definition 2.58. If I is an ideal of a ring R, then the ring (R/I, +, .) described in


Proposition 2.57 is called the quotient (or factor) ring of R by I.

Department of Mathematics/ College of Education/ Al-Qadisiyah University 33 of 124


Ring Theory Prof. Dr. Akeel Ramadan Mehdi Al-Yassiri

Example 2.59. The elements of the quotient ring (Z/ < n >, +, .) are:
0+ < n >, 1+ < n >, 2+ < n >, ..., (n − 1)+ < n >.

Example 2.60. The elements of the quotient ring (Z/ < 6 >, +, .) are:
0+ < 6 >, 1+ < 6 >, 2+ < 6 >, 3+ < 6 >, 4+ < 6 >, 5+ < 6 >.
We can see that: (3+ < 6 >) + (5+ < 6 >) = 8+ < 6 >= 2 + 6+ < 6 >= 2+ < 6 >
and (3+ < 6 >).(5+ < 6 >) = 15+ < 6 >= 3 + 12+ < 6 >= 3+ < 6 >.

Example 2.61. The elements of the quotient ring (2Z/ < 10 >, +, .) are:
0+ < 10 >, 2+ < 10 >, 4+ < 10 >, 6+ < 10 >, 8+ < 10 >.

Example 2.62. The elements of the quotient ring (Z12 / < 4 >, +, .) are:
0+ < 4 >, 1+ < 4 >, 2+ < 4 >, 3+ < 4 >.

Proposition 2.63. Let I be an ideal of a ring R. If R is a commutative ring with


identity 1, then the quotient ring (R/I, +, .) is commutative with identity.
Proof. Let a + I ∈ R/I. Then (a + I).(1 + I) = (a.1) + I = a + I and
(1 + I).(a + I) = (1.a) + I = a + I.
Since 1 + I ∈ R/I, we have 1 + I is the identity element of the ring (R/I, +, .).
Let a + I, b + I ∈ R/I. Then (a + I).(b + I) = ab + I = ba + I = (b + I).(a + I) and
hence the ring (R/I, +, .) is commutative.

Lemma 2.64. Let I be an ideal of a ring R. Then A is an ideal of the quotient ring
R/I if and only if there is a unique ideal K of R such that I ⊆ K and A = K/I.
Proof. Exercise.

Question 2.65. Let I be an ideal of a ring R. Show that whether if a ring R has no
zero divisors, then the quotient ring (R/I, +, .) has no zero divisors or not and why?
Answer: The statement is not true in general, for example: let (Z, +, .) be the ring
of integers and let I =< 6 >. Then the ring (Z, +, .) has no zero divisors but the
quotient ring (Z/I, +, .) has zero divisors, since 2+ < 6 >, 3+ < 6 >∈ Z/I and
2+ < 6 >6= 0, 3+ < 6 >6= 0 but (2+ < 6 >).(3+ < 6 >) = 6+ < 6 >= 0.

Exercise 37. Let I be an ideal of a ring R. Show that whether if a quotient ring
(R/I, +, .) has no zero divisors, then the ring R has no zero divisors or not and why?

Department of Mathematics/ College of Education/ Al-Qadisiyah University 34 of 124


Ring Theory Prof. Dr. Akeel Ramadan Mehdi Al-Yassiri

Exercise 38. Let I be an ideal of a ring R. Prove that if the ring R is a principal ideal
ring, then the quotient ring (R/I, +, .) is a principal ideal ring?.

Exercise 39. Let I be an ideal of a ring R. Prove that if the ring R is simple, then the
quotient ring (R/I, +, .) is simple? Is the converse true or not and why?

Exercise 40. Let I be an ideal of a ring R. Show that whether the characteristic of R
is equal to the characteristic of the quotient ring (R/I, +, .) or not and why?

Department of Mathematics/ College of Education/ Al-Qadisiyah University 35 of 124


Ring Theory Prof. Dr. Akeel Ramadan Mehdi Al-Yassiri

3 Ring Homomorphisms and Isomorphisms


3.1 Ring Homomorphisms
0 0 0 0
Definition 3.1. Let (R, +, .) and (R , + , . ) be two rings. A function f : R → R is
said to be a ring homomorphism if for all a, b ∈ R, then
0 0
f (a + b) = f (a) + f (b) and f (a.b) = f (a). f (b).

0 0 0
Definition 3.2. Let (R, +, .) and (R , + , . ) be two rings. A ring homomorphism
0
f : R → R is said to be a ring monomorphism if it is injective.

0 0 0
Definition 3.3. Let (R, +, .) and (R , + , . ) be two rings. A ring homomorphism
0
f : R → R is said to be a ring epimorphism if it is surjective.

0 0 0
Definition 3.4. Let (R, +, .) and (R , + , . ) be two rings. A ring homomorphism
0
f : R → R is said to be a ring isomorphism if f is both injective and surjective.
0 0
The rings R and R are said to be isomorphic, denoted R ∼ = R , if there is a ring
0
isomorphism ϕ : R → R .

0 0 0
Example 3.5. Let (R, +, .) and (R , + , . ) be two rings.
0 0
Define f : R → R by f (x) = 0 , for all x ∈ R. Then f is a ring homomorphism, it is
called the trivial ring homomorphism.
Proof. Let a, b ∈ R.
0 0 0 0 0
Then f (a + b) = 0 = 0 + 0 = f (a) + f (b) and
0 0 0 0 0
f (a.b) = 0 = 0 . 0 = f (a). f (b).
Hence f is a ring homomorphism.

Example 3.6. Define f : Z → 2Z by f (x) = 2x, for all x ∈ Z. Then f is not a ring
homomorphism.
Proof. Let a, b ∈ Z.
Then f (a + b) = 2(a + b) = 2a + 2b = f (a) + f (b),
but if a = b = 1, then
f (a.b) = 2(a.b) = 2(1.1) = 2 6= f (a). f (b) = f (1). f (1) = (2.1).(2.1) = 4.
Hence f is not a ring homomorphism.

Department of Mathematics/ College of Education/ Al-Qadisiyah University 36 of 124


Ring Theory Prof. Dr. Akeel Ramadan Mehdi Al-Yassiri

Example 3.7. Define f : (Z, +, .) → (Zn , +n , .n ) by f (x) = x, for all x ∈ Z. Then f


is a ring epimorphism.
Proof. Let a, b ∈ Z.
Then f (a + b) = a + b = a +n b = f (a) +n f (b),
and f (a.b) = a.b = a.n b = f (a).n f (b).
Hence f is a ring homomorphism.
Let x ∈ Zn , thus x ∈ Z and f (x) = x.
Therefore, f is a ring epimorphism.

Example 3.8. Let I be an ideal of a ring R. Define natI : R → R/I by natI (a) =
a + I, for all a ∈ R. Then natI is a ring epimorphism, it is called the natural ring
epimorphism.
Proof. Let a, b ∈ R.
Then natI (a + b) = (a + b) + I = (a + I) + (b + I) = natI (a) + natI (b),
and natI (a.b) = (a.b) + I = (a + I).(b + I) = natI (a).natI (b).
Hence natI is a ring homomorphism.
Let a + I ∈ R/I, thus a ∈ R and natI (a) = a + I.
Therefore, natI is a ring epimorphism.

Example 3.9. Let (S, +, .) be a subring of a ring (R, +, .). Then the inclusion map-
ping i : S → R defined by i(a) = a, for all a ∈ S is a ring monomorphism.
Proof. Let a, b ∈ S.
Then i(a + b) = a + b = i(a) + i(b),
and i(a.b) = a.b = i(a).i(b).
Hence i is a ring homomorphism.
Let a, b ∈ S such that i(a) = i(b), thus a = b and this implies that i is an injective.
Therefore, i is a ring monomorphism.

Example 3.10. Let R be a ring. The identity mapping IR : R → R defined by


IR (x) = x, for all x ∈ R is a ring isomorphism.
Proof. Exercise.

Example 3.11. Let (R, +, .) be a ring with identity 1R .


Define f : Z → R by f (n) = n.1R , for all n ∈ Z. Then f is a ring homomorphism.

Department of Mathematics/ College of Education/ Al-Qadisiyah University 37 of 124


Ring Theory Prof. Dr. Akeel Ramadan Mehdi Al-Yassiri

Proof. Let n, m ∈ Z.
Then f (n + m) = (n + m).1R = (n.1R ) + (m.1R ) (by Theorem 1.20(3))
= f (n) + f (m),
and f (n.m) = (n.m).1R = (n.m).(1R .1R ) = (n.1R ).(m.1R ) (by Theorem 1.20(5))
= f (n). f (m) Hence f is a ring homomorphism.

 
x 0
Example 3.12. Let R be a ring. Define f : R → M2×2 (R) by f (x) = for all
0 x
x ∈ R. Then f is a ring monomorphism.
Proof. Exercise.

0 0
Proposition 3.13. Let (R, +, .) and (S, + , . ) be rings, and f : R → S a ring homo-
morphism. Then each of the following holds:
(1) f (0R ) = 0S .
(2) For all x ∈ R, f (−x) = − f (x).
0
Proof. (1) f (0R ) = f (0R + 0R ) = f (0R ) + f (0R ).
0
By cancellation law for addition (+ ), we have f (0R ) = 0S .
(2) Let x ∈ R. Then 0S = f (0R ) = f (x + (−x)) = f (x) + f (−x).
By adding − f (x) to two sides, we have − f (x) = f (−x).

0 0
Theorem 3.14. Let (R, +, .) and (S, + , . ) be rings, and f : R → S a ring homomor-
phism. Then each of the following holds:
0 0 0 0
(1) ( f (R), + , . ) is a subring of the ring (S, + , . ), where f (R) = { f (a)|a ∈ R} and it
is called the homomorphic image of R by the mapping f and is denoted by Im( f ) or
f (R).
(2) If 1R ∈ R, then f (1R ) is the identity of the subring f (R).
(3) If 1R ∈ R and f is an epimorphism, then f (1R ) = 1S .
(4) If R is commutative, then f (R) is commutative.

Proof. (1) By Proposition 3.13(1), 0S = f (0R ) ∈ f (R) and hence f (R) 6= φ .


Since f (R) ⊆ S, we have f (R) is a non-empty subset of S.
Let x, y ∈ f (R), thus there are a, b ∈ R such that x = f (a) and y = f (b).
0 0
Then x − y = f (a) − f (b) = f (a − b) ∈ f (R) (because a − b ∈ R) and
0 0
x. y = f (a). f (b) = f (a.b) ∈ f (R) (because a.b ∈ R).
0 0 0 0
Therefore, ( f (R), + , . ) is a subring of the ring (S, + , . ).
(2) Let x ∈ f (R), thus there is a ∈ R such that x = f (a).
0 0
Then x. f (1R ) = f (a). f (1R ) = f (a.1R ) = f (a) = x.

Department of Mathematics/ College of Education/ Al-Qadisiyah University 38 of 124


Ring Theory Prof. Dr. Akeel Ramadan Mehdi Al-Yassiri

0
Similarly, f (1R ). x = x. Hence, f (1R ) is the identity of f (R).
(3) Suppose that f is an epimorphism, thus f (R) = S.
By (2) above, f (1R ) is the identity of the subring f (R) and hence f (1R ) is the identity
of the ring S.
By Theorem 1.16(1), the multiplicative identity is unique and hence f (1R ) = 1S .
(4) Suppose that R is a commutative ring.
Let x, y ∈ f (R), thus there are a, b ∈ R such that x = f (a) and y = f (b).
0 0 0 0
Then x. y = f (a). f (b) = f (a.b) = f (b.a) = f (b). f (a) = y. x.
Therefore, f (R) is commutative.

0 0
Proposition 3.15. Let (R, +, .) and (S, + , . ) be rings, and f : R → S a ring homo-
morphism. For all x ∈ R, if x has a multiplicative inverse and f is onto, then f (x) has
a multiplicative inverse, and f (x−1 ) = ( f (x))−1 .
Proof. Suppose that x has a multiplicative inverse in R and f is onto.
0
Then f (x). f (x−1 ) = f (x.x−1 ) = f (1R ) = 1S (by Theorem 3.14(3)) and
0
f (x−1 ). f (x) = f (x−1 .x) = f (1R ) = 1S (by Theorem 3.14(3)).
Hence f (x−1 ) is a multiplicative inverse of f (x)
and so f (x) has a multiplicative inverse.
Since ( f (x))−1 is a multiplicative inverse of f (x) and the multiplicative inverse is
unique (by Theorem 1.16(2)), we have f (x−1 ) = ( f (x))−1 .

0 0
Definition 3.16. Let (R, +, .) and (S, + , . ) be rings, and f : R → S a ring homomor-
phism. The kernel of f is denoted by ker( f ) and defined as
ker ( f ) = {x ∈ R| f (x) = 0S }.

and the image of f is denoted by Im( f ) and defined as


Im ( f ) = {y ∈ S|y = f (x), for some x ∈ R} = f (R).

Example 3.17. Let R be a ring. If f : R → R is the zero homomorphism, then


ker( f ) = R and Im( f ) = 0R .
Proof. ker( f ) = {x ∈ R| f (x) = 0R } = {x | x ∈ R} = R and Im( f ) = f (R) = 0R .

Example 3.18. Let f : (Z, +, .) → (Zn , +n , .n ) be the ring homomorphism defined


by f (x) = x, for all x ∈ Z. Then ker( f ) =< n > and Im( f ) = Zn .

Department of Mathematics/ College of Education/ Al-Qadisiyah University 39 of 124


Ring Theory Prof. Dr. Akeel Ramadan Mehdi Al-Yassiri

Proof. ker( f ) = {x ∈ Z| f (x) = 0}


= {x ∈ Z|x = 0}
= {x ∈ Z|x = 0 mod n}
= {x ∈ Z|x = kn, k ∈ Z} =< n >.
Since f is an epimorphism, we have Im( f ) = Zn .

Example 3.19. Let I be an ideal of a ring R and let natI : R → R/I be the natural ring
homomorphism defined by natI (a) = a + I, for all a ∈ R (as in Example 3.8). Then
ker( f ) = I.
Proof. ker( f ) = {x ∈ R| f (x) = 0}
= {x ∈ R|x + I = I}
= {x ∈ R|x ∈ I}
= I.

0 0
Proposition 3.20. Let (R, +, .) and (S, + , . ) be rings, and let f : R → S be a ring
homomorphism. Then ker( f ) is an ideal of R and Im( f ) is a subring of S.
Proof. First we will prove that ker( f ) is an ideal of R.
Since f (0R ) = 0S , we have 0R ∈ ker( f ) and hence φ 6= ker( f ) ⊆ R.
Let x, y ∈ ker( f ) and r ∈ R, thus f (x) = 0S and f (y) = 0S .
Hence f (x − y) = f (x) − f (y) = 0S − 0S = 0S and so x − y ∈ ker( f ).
Also, f (rx) = f (r). f (x) = f (r).0S = 0S and f (xr) = f (x). f (r) = 0S . f (r) = 0S .
Hence rx, xr ∈ ker( f ).
Therefore, ker( f ) is an ideal of R.
Now, we will prove that Im( f ) is a subring of S.
Since f (0R ) = 0S , we have 0S ∈ Im( f ) and hence φ 6= Im( f ) ⊆ S.
Let x, y ∈ Im( f ), thus x = f (a) and y = f (b), for some a, b ∈ R.
Then x − y = f (a) − f (b) = f (a − b) ∈ Im( f ) and
xy = f (a). f (b) = f (a.b) ∈ Im( f ).
Therefore, Im( f ) is a subring of S.

0 0
Proposition 3.21. Let (R, +, .) and (S, + , . ) be rings, and let f : R → S be a ring
homomorphism. Then ker( f ) = 0 if and only if f is an injective.
Proof. Exercise.

0 0
Proposition 3.22. Let (R, +, .) and (S, + , . ) be rings. Suppose that f : R → S is a
surjective ring homomorphism. If I is an ideal of R, then f (I) = { f (a)|a ∈ I} is an
ideal of S.

Department of Mathematics/ College of Education/ Al-Qadisiyah University 40 of 124


Ring Theory Prof. Dr. Akeel Ramadan Mehdi Al-Yassiri

Proof. Since 0R ∈ I and 0S = f (0R ) ∈ f (I), we have φ 6= f (I) ⊆ S.


Let x, y ∈ f (I), thus there are a, b ∈ I such that x = f (a) and y = f (b).
0 0
Then x − y = f (a) − f (b) = f (a − b) (since f is a ring homomorphism).
Since I is an ideal in R, we have a − b ∈ I and hence f (a − b) ∈ f (I).
0
Therefore, x − y ∈ f (I), for any x, y ∈ f (I).
Let s ∈ S, thus there is c ∈ R such that s = f (c) (since f is an epimorphism).
0 0
Then s. x = f (c). f (a) = f (c.a).
Since I is an ideal in R, we have c.a ∈ I and hence f (c.a) ∈ f (I).
0
Therefore, s. x ∈ f (I), for any x ∈ f (I) and s ∈ S.
0 0
Also, we have x. s = f (a). f (c) = f (a.c).
Since I is an ideal in R, we have a.c ∈ I and hence f (a.c) ∈ f (I).
0
Therefore, x. s ∈ f (I), for any x ∈ f (I) and s ∈ S and hence f (I) = { f (a)|a ∈ I} is
an ideal of S.

0 0
Proposition 3.23. Let (R, +, .) and (S, + , . ) be rings. Suppose that f : R → S is a
ring homomorphism. If I is an ideal of S, then f −1 (I) = {a ∈ R| f (a) ∈ I} is an ideal
of R and ker( f ) ⊆ f −1 (I).
Proof. Since f (0R ) = 0S ∈ I, we have 0R ∈ f −1 (I) and hence φ 6= f −1 (I) ⊆ R.
Let x, y ∈ f −1 (I), thus f (x), f (y) ∈ I.
0
Then f (x − y) = f (x) − f (y) (since f is a ring homomorphism).
0
Since I is an ideal in S, we have f (x) − f (y) ∈ I and hence f (x − y) ∈ I.
Therefore, x − y ∈ f −1 (I), for any x, y ∈ f −1 (I).
0
Let r ∈ R, thus f (r.x) = f (r). f (x) (since f is a ring homomorphism).
0
Since I is an ideal in S, we have f (r). f (x) ∈ I and hence f (r.x) ∈ I.
Therefore, r.x ∈ f −1 (I), for any x ∈ f −1 (I) and r ∈ R.
0
Also, we have f (x.r) = f (x). f (r) (since f is a ring homomorphism).
0
Since I is an ideal in S, we have f (x). f (r) ∈ I and hence f (x.r) ∈ I.
Therefore, x.r ∈ f −1 (I), for any x ∈ f −1 (I) and r ∈ R and hence
f −1 (I) = {a ∈ R| f (a) ∈ I} is an ideal of R.
Now we will prove that ker( f ) ⊆ f −1 (I).
Since ker( f ) = {a ∈ R| f (a) = 0S } = {a ∈ R| f (a) ∈ {0S }} = f −1 (0), we have ker( f ) =
f −1 (0) ⊆ f −1 (I).

0 0
Theorem 3.24. Let (R, +, .) and (S, + , . ) be rings and let f : R → S be a ring epi-
morphism. If R is a principal ideal ring, then S is a principal ideal ring too.
Proof. Since R is a PIR, we have R is a commutative ring with identity 1R and every
ideal in R is principal.
By Theorem 3.14(3), f (1R ) is the identity of the ring S.

Department of Mathematics/ College of Education/ Al-Qadisiyah University 41 of 124


Ring Theory Prof. Dr. Akeel Ramadan Mehdi Al-Yassiri

0 0
By Theorem 3.14(4), the ring (S, + , . ) is commutative.
Now, we will prove that every ideal in S is principal.
Let I be an ideal in S. By Proposition 3.23, f −1 (I) is an ideal of R.
Since R is a a principal ideal ring (by hypothesis), there is a ∈ R such that
f −1 (I) =< a > and hence f ( f −1 (I)) = f (< a >).
Since f is a surjective function, we have f ( f −1 (I)) = I and hence I = f (< a >).
Now, we will prove that f (< a >) =< f (a) >.
Let x ∈ f (< a >), thus there is y ∈< a > such that
x = f (y) and hence x = f (r.a) for some r ∈ R.
0
Then x = f (r.a) = f (r). f (a) ∈< f (a) >, and so f (< a >) ⊆< f (a) >.
0
Let y ∈< f (a) >, thus there is s ∈ S such that y = s. f (a).
Since f is onto (by hypothesis), there is r ∈ R such that s = f (r).
0
Then y = f (x) = f (r). f (a) = f (r.a) ∈ f (< a >) and hence < f (a) >⊆ f (< a >).
Thus f (< a >) =< f (a) > and hence I < f (a) > and so that I is a principal ideal in
S. Therefore, every ideal in S is principal and hence S is principal ideal ring.

Corollary 3.25. The ring (Zn , +n , .n ) is a principal ideal ring.


Proof. Define f : (Z, +, .) → (Zn , +n , .n ) by f (x) = x, for all x ∈ Z.
By Example 3.7, f is a ring epimorphism.
By Theorem 2.37, the ring (Z, +, .) is a principal ideal ring.
By Theorem 3.24, the ring (Zn , +n , .n ) is a principal ideal ring.

Corollary 3.26. If (R, +, .) is a principal ideal ring and I is an ideal of R, then R/I
is a principal ideal ring.
Proof. Exercise.

Exercise 41. Let (R, +, .) be a ring with identity and let a be an invertible element
in R. Define fa : R → R by fa (x) = a.x.a−1 , for all x ∈ R. Prove that fa is a ring
homomorphism.

Exercise 42. Let (R, +, .) be a ring with identity 1 and let S = R ⊕ R be the direct
product ring of R by R. Define f : R → S by f (a) = (a, 0), for all a ∈ R.
Prove that f is a ring monomorphism with f (1R ) 6= 1S .

Exercise 43. Give an example of a ring homomorphism f : R → S such that the


ring R contains an element a has a multiplicative inverse in R but f (a) has not a
multiplicative inverse in S with proof.

Department of Mathematics/ College of Education/ Al-Qadisiyah University 42 of 124


Ring Theory Prof. Dr. Akeel Ramadan Mehdi Al-Yassiri

Exercise 44. Let R be a ring with identity 1. Define f : R → M2×2 (R) by


  f (x) 
=
x 0 1 0
for all x ∈ R. Prove that f is a ring monomorphism and f (1) 6= .
0 0 0 1

Exercise 45. Let R be a ring with identity 1R and φ : Q → R be a non-zero ring


homomorphism. Prove that φ is injective.

Exercise 46. Let R and S be rings. Suppose that f : R → S is a ring homomorphism.


Show that whether if I is an ideal of R, then f (I) = { f (a)|a ∈ I} is an ideal of S or
not and why?.

Department of Mathematics/ College of Education/ Al-Qadisiyah University 43 of 124


Ring Theory Prof. Dr. Akeel Ramadan Mehdi Al-Yassiri

3.2 Ring Isomorphism Theorems


0 0
Theorem 3.27. (First Isomorphism Theorem for Rings) Let (R, +, .) and (S, + , . )
be rings. If f : R → S is a ring homomorphism, then R/ker( f ) ∼
= im( f ).
Proof. Define ϕ : R/ker( f ) → im( f ) by ϕ(x + ker( f )) = f (x), for all x ∈ R.
If x + ker( f ) = y + ker( f ), then x − y ∈ ker( f )
0
⇒ f (x − y) = 0S ⇒ f (x) − f (y) = 0S ⇒ f (x) = f (y)
⇒ ϕ(x + ker( f )) = ϕ(y + ker( f )). Thus ϕ is well-defined.
Let x + ker( f ), y + ker( f ) ∈ R/ker( f ).
Thus ϕ((x + ker( f )) + (y + ker( f )))
= ϕ((x + y) + ker( f ))
0 0
= f (x + y) = f (x) + f (y) = ϕ(x + ker( f )) + ϕ(y + ker( f )).
Also, ϕ((x + ker( f )).(y + ker( f )))
0
= ϕ(xy + ker( f )) = f (x.y) = f (x). f (y)
0
= ϕ(x + ker( f )). ϕ(y + ker( f )).
Hence ϕ is a well-defined ring homomorphism.
Let y ∈ im( f ) ⇒ ∃x ∈ M 3 f (x) = y.
Since x + ker( f ) ∈ R/ker( f ) and ϕ(x + ker( f )) = f (x) = y, we have
ϕ is a surjective mapping and hence ϕ is a ring epimorphism.
Let x + ker( f ) ∈ ker(ϕ), thus ϕ(x + ker( f )) = 0S
⇒ f (x) = 0S ⇒ x ∈ ker( f )
⇒ x + ker( f ) = ker( f ) = 0(R/ker( f )) ⇒ ker(ϕ) = 0(R/ker( f )) .
By Proposition 3.21, ϕ is a ring monomorphism.
Therefore, ϕ is a ring isomorphism and hence R/ker( f ) ∼ = im( f ).

Corollary 3.28. If f : R → S is a ring epimorphism, then R/ker( f ) ∼


= S.
Proof. By Theorem 3.27, R/ker( f ) ∼ = im( f ).
Since f is surjective, im( f ) = S and hence R/ker( f ) ∼
= S.

Corollary 3.29. (Z/ < n >, +, .) ∼


= (Zn , +n , .n ).
Proof. Define f : (Z, +, .) → (Zn , +n , .n ) by f (x) = x, for all x ∈ Z.
By Example 3.7, f is a ring epimorphism.
By Corollary 3.28, Z/ker( f ) ∼
= Zn .
By Example 3.18, ker( f ) =< n > and hence Z/ < n >∼ = Zn .

Lemma 3.30. Let I be an ideal in a ring R and S be a subring of R. Then:


(1) S + I is a subring of R,

Department of Mathematics/ College of Education/ Al-Qadisiyah University 44 of 124


Ring Theory Prof. Dr. Akeel Ramadan Mehdi Al-Yassiri

(2) I is an ideal in S + I,
(3) S ∩ A is an ideal in S.
Proof. Exercise.

Theorem 3.31. (Second Isomorphism Theorem for Rings) If I is an ideal in a ring


R and S is a subring of R, then S/(S ∩ I) ∼
= (S + I)/I.
Proof. By Lemma 3.30, we have S + I is a subring of R, I is an ideal in S + I and
S ∩ A is an ideal in S.
Define the mapping ϕ : S → (S + I)/I by ϕ(x) = x + I, for all x ∈ S.
Note that ϕ is a ring homomorphism (Exercise).
ϕ is surjective, since if (x + a) + I ∈ (S + I)/I with x ∈ S and a ∈ I, then ϕ(x) =
x + I = x + (a + I) = (x + a) + I.
By Theorem 3.27 (the First Isomorphism Theorem), we have S/ker(ϕ) ∼ = (S + I)/I.
Now, we will prove that ker(ϕ) = S ∩ I.
Let x ∈ker(ϕ). Then I = ϕ(x) = x + I and hence x ∈ I.
Since ker(ϕ) ⊆ S, we have x ∈ S.
Thus x ∈ S ∩ I and so ker(ϕ) ⊆ S ∩ I.
Conversely, suppose x ∈ S ∩ I, thus x ∈ I and hence ϕ(x) = x + I = I.
That is, x ∈ ker(ϕ) and so S ∩ I ⊆ ker(ϕ).
Therefore, ker(ϕ) = S ∩ I.

Corollary 3.32. Let I and J be two ideals of a ring R. If R = I ⊕ J, then R/I ∼


= J.
Proof. By Theorem 3.31, J/(J ∩ I) ∼= (J + I)/I.
Since R = I + J and I ∩ J = (0), we have R/I = (J + I)/I ∼= J/(J ∩ I) = J/(0) ∼= J.

Therefore, R/I = J.

Theorem 3.33. (Third Isomorphism Theorem for Rings) If I and J are ideals of
a ring R such that J ⊆ I, then I/J is an ideal of R/J and (R/J)/(I/J) ∼
= R/I.
Proof. I/J is an ideal of R/J (Exercise)
Define ϕ : R/J → R/I by ϕ(r + J) = r + I.
ϕ is a well-defined, since if we suppose that r + J = s + J then
r − s ∈ J ⊆ I and hence r + I = s + I.
Thus ϕ(r + J) = ϕ(s + J) and hence ϕ is a well-defined.
Now, we will prove that ϕ is a ring homomorphism.
Let r + J, s + J ∈ R/J, thus
ϕ ((r + J) + (s + J)) = ϕ ((r + s) + J)
= (r + s) + I = (r + I) + (s + I) = ϕ(r + J) + ϕ(s + J) and

Department of Mathematics/ College of Education/ Al-Qadisiyah University 45 of 124


Ring Theory Prof. Dr. Akeel Ramadan Mehdi Al-Yassiri

ϕ ((r + J) . (s + J)) = ϕ (rs + J) = rs + I = (r + I) . (s + I) = ϕ(r + J).ϕ(s + J).


Hence ϕ is a ring homomorphism.
It is clear that ϕ is a surjective (Exercise) and hence ϕ is an epimorphism.
By Theorem 3.27 (the First Isomorphism Theorem), we have (R/J)/ker(ϕ) ∼ = R/I.
Now, we will prove that ker(ϕ) = I/J.
Let r + J ∈ ker(ϕ), thus ϕ(r + J) = I.
Since ϕ(r + J) = r + I, we have r + I = I and hence r ∈ I.
Thus r + J ∈ I/J and so ker(ϕ) ⊆ I/J.
Let x + J ∈ I/J, thus x ∈ I and hence x + I = I.
Since ϕ(x + J) = x + I, we have ϕ(x + J) = I and hence x + J ∈ ker(ϕ).
Thus I/J ⊆ ker(ϕ) and hence ker(ϕ) = I/J.
Therefore, (R/J)/(I/J) ∼ = R/I.

Example 3.34. Since < 9 > and < 3 > are two ideals of the ring (Z, +, .) with
< 9 >⊆< 3 >, we have from Theorem 3.33 that
(< 3 > / < 9 >) is an ideal in the ring (Z/ < 9 >, +, .) and
(Z/ < 9 >)/(< 3 > / < 9 >) ∼ = Z/ < 3 >∼ = Z3 .

Example 3.35. If n divides m in the ring (Z, +, .), then


(Z/mZ) / (nZ/mZ) ∼ = (Z/nZ) ∼ = Zn .

Definition 3.36. A ring R is said to be embedded in a ring S (or S is said to be an


extension of R) iff there exists a ring monomorphism f : R → S.

Example  3.37. Consider the mapping f : (Z, +, .) → (M2 (Z), +, .) defined by:
x 0
f (x) = for all x ∈ Z.
0 x
Clearly, f is a ring monomorphism. Then (Z, +, .) is embedded in a ring (M2 (Z), +, .).

Example 3.38. Let S be a subring in a ring R. Then S is embedded in the ring R.


Since the inclusion mapping i : S → R is a ring monomorphism. Then S is embedded
in the ring R.

Lemma 3.39. Let S, R and T be rings. If f : R → S and g : S → T are two ring


homomorphisms, then g ◦ f : R → T is a ring homomorphism.
Proof. Exercise.

Department of Mathematics/ College of Education/ Al-Qadisiyah University 46 of 124


Ring Theory Prof. Dr. Akeel Ramadan Mehdi Al-Yassiri

0 0 0 00 00 00
Corollary 3.40. Let (R, +, .), (R , + , . ) and (R , + , . ) be rings. If R is embedded
0 0 00
in the ring R and the ring R is embedded in the ring R , then the ring R is embedded
00
in the ring R .
0
Proof. Since the ring R is embedded in the ring R , there exists a ring monomorphism
0
f :R→R.
0 00
Since the ring R is embedded in the ring R , there exists a ring monomorphism
0 00
g:R →R .
00
Put h : g ◦ f : R → R . By Lemma 3.39, h is a ring homomorphism.
Since f and g are injective, we have h is injective and hence h is a ring monomor-
00
phism. Therefore, R is embedded in the ring R .

0 0
Proposition 3.41. Let (S, +, .) and (R, + , . ) be two rings. Then S is embedded in
the ring R if and only if S isomorphic to a subring of R.
Proof. (⇒) Suppose that S is embedded in the ring R.
Thus there is a ring monomorphism f : S → R.
By Proposition 3.20, f (S) is a subring in R.
Define α : S → f (S) by α(a) = f (a), for all a ∈ S.
α is a ring isomorphism, since
0 0
if a, b ∈ S, then α(a + b) = f (a + b) = f (a) + f (b) = α(a) + α(b) and
0 0
α(a.b) = f (a.b) = f (a). f (b) = α(a). α(b).
Thus α is a ring homomorphism.
Also, if α(a) = α(b), then f (a) = f (b).
Since f is injective, we have a = b and hence α is a ring monomorphism.
Let y ∈ f (S), thus there is x ∈ S with y = f (x) and hence α(x) = f (x) = y.
Therefore, α is a ring epimorphism and so f is a ring isomorphism.
Thus S isomorphic to the subring f (S) of R.
(⇐) Suppose that S isomorphic to a subring of R, say H.
Thus there is a ring isomorphism α : S → H.
Let f = iH ◦ α : S → R, where iH : H → R is the inclusion mapping.
Since iH and α are ring homomorphisms, we have from Lemma 3.39 that iH ◦ α is
ring homomorphism.
Since iH and α are injective, we have iH ◦ α is injective and
hence iH ◦ α is a ring monomorphism.
Therefore, S is embedded in the ring R.

Theorem 3.42. (Dorroh Extension Theorem) Let R be a ring without identity ele-
ment. Then R can be embedded in a ring S with identity element.

Department of Mathematics/ College of Education/ Al-Qadisiyah University 47 of 124


Ring Theory Prof. Dr. Akeel Ramadan Mehdi Al-Yassiri

Proof. Let R be a ring without identity element and S = Z × R.


Define (+) and (.) on S by
(n, a) + (m, b) = (n + m, a + b) and (n, a).(m, b) = (nm, nb + ma + ab).
Then (S, +, .) is a ring with the identity element (1Z , 0R ) (Exercise).
Define a mapping f : R → Z × R by f (a) = (0, a), for all a ∈ R.
Then f is a ring homomorphism (Exercise).
Since ker f = {a ∈ R | f (a) = (0, 0R )} = {0R },
we have from Proposition 3.21 that f is a ring monomorphism.
Thus R is embedded in Z × R.

Proposition 3.43. If f : Z → Z is a ring homomorphism, then either f = 0 or f is


the identity mapping IZ .
Proof. Let f : Z → Z be a non-zero ring homomorphism.
We will prove that f = IZ .
Let n ∈ Z.
If n > 0, then f (n) = f (1 {z... + 1}) = |f (1) + f (1){z+ ... + f (1)} = n f (1).
| +1+
n times n times
If n < 0, then −n is a positive integer and hence f (−n) = (−n) f (1).
Since f is a ring homomorphism, we have from Proposition 3.13(2) that
f (−n) = − f (n) and hence − f (n) = −n f (1).
Thus, f (n) = n f (1) in this case too.
If n = 0, then from Proposition 3.13(1) we have f (0) = 0 = 0 f (1).
Therefore, f (n) = n f (1), for all n ∈ Z.
Now, since f is a non-zero ring homomorphism, we have f (m) 6= 0 for some m ∈ Z.
Then, f (m) = f (m.1) = f (m) f (1).
Since Z is an integral domain, we have from Corollary 1.49(1) that
Z satisfies the cancellation laws for multiplication and
hence by cancelling f (m) from both sides we get f (1) = 1.
Since f (n) = n f (1), for all n ∈ Z (as above), we see that f (n) = n for all n ∈ Z.
Hence f = IZ .

Lemma 3.44. If f : R → S is a ring isomorphism, then f −1 : S → R is a ring isomor-


phism.
Proof. Exercise.

Corollary 3.45. Let R be a ring. If f and g are two ring isomorphisms from R onto
Z, then f = g.

Department of Mathematics/ College of Education/ Al-Qadisiyah University 48 of 124


Ring Theory Prof. Dr. Akeel Ramadan Mehdi Al-Yassiri

Proof. By Lemma 3.44, g−1 : Z → R is an isomorphism.


Then f ◦ g−1 : Z → Z is an isomorphism.
By Proposition 3.43, f ◦ g−1 = IZ .
Therefore, f = g.

Theorem 3.46. Let R be a ring and f and g be ring epimorphisms from R onto Z
such that Ker ( f ) = Ker (g). Then f = g.
In other word: Any ring epimorphism from an arbitrary ring R onto Z is uniquely
determined by its kernel.
Proof. By Theorem 3.27, we have isomorphisms
α : R/Ker( f ) → Z and β : R/Ker(g) → Z.
Since Ker ( f ) = Ker (g), we have α and β are isomorphisms of the same ring onto
Z. Thus, by the Corollary 3.45, α = β .
Also, the natural ring epimorphisms
natKer ( f ) : R → R/Ker ( f ) and natKer (g) : R → R/Ker (g) are the same
since Ker ( f ) = Ker (g).
Therefore, f = α ◦ natKer ( f ) = β ◦ natKer (g) = g.

Exercise 47. Let R be a commutative ring with identity 1 and let a ∈ R be an idem-
potent element. Prove that R/ < a >∼
=< 1 − a >.

Exercise 48. Prove that (Z/ < 20 >)/(< 5 > / < 20 >) ∼
= Z5 .

Exercise 49. Prove that (Z20 / < 10 >)/(< 5 > / < 10 >) ∼
= {< 5 >, 1+ < 5 >
, 2+ < 5 >, 3+ < 5 >, 4+ < 5 >}.

Exercise 50. Let R be a ring with identity 1. Prove that R is embedded in the ring
(M2 (R), +, .).

Exercise 51. Describe all ring homomorphisms from Z to Z.

Department of Mathematics/ College of Education/ Al-Qadisiyah University 49 of 124


Ring Theory Prof. Dr. Akeel Ramadan Mehdi Al-Yassiri

4 Division Rings, Fields and Prime Fields


4.1 Division Rings and Fields
Definition 4.1. A ring R with identity (1 6= 0) is called a division ring or a (skew
field) if every non-zero element has a multiplicative inverse.

Remarks 4.2. (1) If u = a + bi and v = c + di are complex numbers, then


u + v = (a + bi) + (c + di) = (a + c) + (b + d)i and
u.v = (a + bi).(c + di) = (ac − bd) + (ad + bc)i.
(2) If u = a + bi, then u = a − bi and u.u = a2 + b2 .

 
u −v
Example 4.3. (Hamilton’s quaternions ring) Let H = { | u, v ∈ C} with
v u
usual addition (+) and multiplication (.) on matrices.
Then (H, +, .) is a non-commutative division ring.
 
1 0
Proof. It is clear that (H, +, .) is a subring of the ring (M2 (C), +, .) and is
0 1
the identity element of the ring (H, +, .)(Exercise).   
0 1 i 0
The ring (H, +, .) is non-commutative, since if A = and B = ,
−1 0 0 −i
       
0 1 i 0 0 −i 0 i
then A.B = . = 6= B.A = .
−1 0 0 −i −i 0 i 0
The non-zero
 elements  of H are invertible.
 
u −v 0 0
Let A = ∈ H with A 6= .
v u 0 0
 
u/(uu + vv) v/(uu + vv)
Let B = .
−v/(uu + vv) u/(uu + vv)
 
1 0
Since B ∈ H and A.B = B.A = , we have A is invertible and
0 1
 
−1 u/(uu + vv) v/(uu + vv)
A = .
−v/(uu + vv) u/(uu + vv)
Therefore, (H, +, .) is a non-commutative division ring.

Definition 4.4. A field is a commutative ring with identity (1 6= 0) in which every


non-zero element has a multiplicative inverse.

Remark 4.5. Every field is a division ring but the converse is not true in general, for
example:

Department of Mathematics/ College of Education/ Al-Qadisiyah University 50 of 124


Ring Theory Prof. Dr. Akeel Ramadan Mehdi Al-Yassiri

 
u −v
let H = { | u, v ∈ C} with usual addition (+) and multiplication (.) on
v u
matrices. Then (H, +, .) is a non-commutative division ring (by Example 4.3) and
hence it is not a field.

Examples 4.6. (1) (Q, +, .), (R, +, .) and (C, +, .) are fields (called fields of rational
numbers, real numbers and complex numbers, respectively).
(2) The ring (Z, +, .) is not a field since 2 ∈ Z has no a multiplicative inverse in Z.


Example 4.7. If F = {a + b 3 | a, b ∈ Q}, then (F, +, .) is a field.
Proof. As in Example
√ 1.14, we can prove that (F, +, .) is a commutative ring with
identity 1√ + 0 3 (Exercise).√
Let a + b 3 ∈ F with a + b 3 6= 0, thus either√a 6= 0 or b 6= 0.

Hence (a + b 3)−1 = 1√
= 1√
.( a−b√3 )
√ (a+b 3) (a+b 3) a−b 3
a b 3 a b

= a2 −3b2 − a2 −3b2 = a2 −3b2 − a2 −3b2 3.
a −b a b

Since a2 −3b ,
2 a2 −3b2 ∈ Q, we have a2 −3b2
− a2 −3b2
3 ∈ F.
√ −1
Then (a + b 3) ∈ F and hence every non-zero element in F has a multiplicative
inverse. Therefore, (F, +, .) is a field.

Theorem 4.8. Every field is an integral domain.


Proof. Let (F, +, .) be a field, thus (F, +, .) is a commutative ring with identity.
Let a, b ∈ F with a 6= 0 and a.b = 0. We will prove that b = 0.
Since F is a field, it follows that a has a multiplicative inverse a−1 in F.
Then a−1 .(a.b) = a−1 .0 = 0 and hence b = 0.
Thus (F, +, .) has no zero divisors and hence (F, +, .) is an integral domain.

Remark 4.9. The converse of Theorem 4.8 is not true in general, for example the
ring (Z, +, .) is an integral domain but it is not a field since 2 ∈ Z has no a multipli-
cative inverse in Z.

Theorem 4.10. Every finite integral domain is a field.


Proof. Let R be a finite integral domain.
Thus R is a commutative ring with identity.
Let n be the number of distinct elements in R, say R = {a1 , a2 , ..., an }, where the ai

Department of Mathematics/ College of Education/ Al-Qadisiyah University 51 of 124


Ring Theory Prof. Dr. Akeel Ramadan Mehdi Al-Yassiri

are the distinct elements of R.


Let a be any nonzero element of R.
0
Consider the set of products R = {a.a1 , a.a2 , ..., a.an }.
0
We will prove that all elements in R are distinct.
Assume that there are i, j such that i 6= j and a.ai = a.a j .
Since R is an integral domain and a 6= 0, we have from Corollary 1.49 that
R satisfies the cancellation laws for multiplication
and hence ai = a j and this is a contradiction.
0
Thus all elements in R are distinct.
0 0
Since R ⊆ R, we have R = R .
0
Since 1 ∈ R, we have 1 ∈ R and so 1 = a.as for some as ∈ R.
Since R is commutative, we have 1 = a.as = as .a and hence a has a multiplicative
inverse as in R. Therefore, R is a field.

Corollary 4.11. The ring (Zn , +n , .n ) is a field if and only if n is a prime number.
Proof. (⇒) Suppose that (Zn , +n , .n ) is a field.
By Theorem 4.8, (Zn , +n , .n ) is an integral domain.
By Theorem 1.37, n is a prime number.
(⇐) Suppose that n is a prime number.
By Theorem 1.37, the ring (Zn , +n , .n ) has no zero divisor.
Since (Zn , +n , .n ) is a commutative ring with identity,
we have (Zn , +n , .n ) is an integral domain.
Since the ring (Zn , +n , .n ) is finite,
we have from Theorem 4.10 that (Zn , +n , .n ) is a field.

Theorem 4.12. Let R be a commutative ring with identity. Then R is a field if and
only if the only ideals in R are (0) and R itself.
Proof. (⇒) Suppose that R is a field.
Let I be any ideal in R. Suppose that I 6= {0}; we have to show that I = R.
Since I 6= {0}, there is 0 6= a ∈ I.
Since R is a field (by hypothesis), we have a has a multiplicative inverse a−1 .
Since I is an ideal in R, we have a.a−1 ∈ I and hence 1 ∈ I.
By Proposition 2.15, I = R.
Hence the only ideals in R are (0) and R itself.
(⇐) Suppose that R be a commutative ring with identity and the only ideals in R are
(0) and R itself.
Let 0 6= a ∈ R. We will prove that a has a multiplicative inverse in R.
Let I = (a) = Ra. By Proposition 2.6, I is an ideal in R.

Department of Mathematics/ College of Education/ Al-Qadisiyah University 52 of 124


Ring Theory Prof. Dr. Akeel Ramadan Mehdi Al-Yassiri

Since 0 6= a ∈ I, we have I 6= 0.
By hypothesis, I = R and hence 1 ∈ I = Ra.
So there exists b ∈ R with ba = ab = 1.
Thus every nonzero element in R has a multiplicative inverse and hence R is a field.

Proposition 4.13. Let F1 and F2 be two fields. If α : F1 → F2 is a ring homomor-


phism, then either α = 0 or α is injective.
Proof. Since ker(α) is an ideal in F1 and F1 is a field,
we have from Theorem 4.12 that either ker(α) = 0 or ker(α) = F1 .
If ker(α) = 0, then α in injective (by Proposition 3.21).
If ker(α) = F1 , then α = 0.
Thus either α = 0 or α is injective.

Exercise 52. Let (R, +, .) be a division ring. Prove that (cent(R), +, .) is a field.

Exercise 53. Consider the set C = R × R of order pairs of real numbers.


Define addition and multiplication on C by
(a, b) + (c, d) = (a + c, b + d)
and
(a, b).(c, d) = (ac − bd, ad + bc).
Prove that (C, +, .) is a field. (This field isomorphic to the field of complex numbers)

Exercise 54. Define two binary operations ∗ and ◦ on the set Z of integers by
a ∗ b = a + b − 1 and a ◦ b. = a + b − ab f or all a, b ∈ Z
Prove that (Z, +, .) is a field.
Exercise 55. Prove that any subring with identity of a field is an integral domain.
Exercise 56. Prove that every field is a simple ring. Is the converse true or not and
why?
Exercise 57. Prove that every field is a principal ideal domain. Is the converse true
or not and why?

Exercise 58. Let R = {a + b 2 | a, b ∈ Z}. Find a non-trivial ideal of R to show that
(R, +, .) is not a field.
Exercise 59. Let F1 and F2 be two fields. Prove that if there is a nontrivial ring
epimorphism between F1 and F2 , then they are isomorphic.

Department of Mathematics/ College of Education/ Al-Qadisiyah University 53 of 124


Ring Theory Prof. Dr. Akeel Ramadan Mehdi Al-Yassiri

4.2 Subfields and Prime Fields


Definition 4.14. Let (F, +, .) be a field, and S a nonempty subset of F. If (S, +, .) is
also a field under the same operations as F, then S is called a subfield of F (or F is
an extension field of the field S).

Examples 4.15. (1) In any field F, F is a subfield of F.


(2) The field of rational numbers (Q, +, .) is a subfield of the field of real numbers
(R, +, .). √
(3) Let F = {a + b 3 | a, b ∈ Q}. Then (F, +, .) is a subfield of the field of real
numbers (R, +, .).
Proof. By Example 4.7, (F, +, .) is a field.
Since F ⊆ R, we have (F, +, .) is a subfield of the field (R, +, .).

Theorem 4.16. A non-empty subset S 6= {0} of a field (F, +, .) is a subfield if and


only if the following two conditions are hold:
(1) for all a, b ∈ S, we have a − b ∈ S;
(2) for all a, b ∈ S with b 6= 0, we have a.b−1 ∈ S.
Proof. (⇒) Suppose that S is a subfield of a field (F, +, .).
Thus (S, +, .) is a subring of the ring (F, +, .) and every non-zero element in S has a
multiplication inverse.
Let a, b ∈ S, thus we have from Theorem 1.28 that a − b ∈ S and a.b ∈ S.
If b 6= 0, then b−1 ∈ S and hence a.b−1 ∈ S.
(⇐). Suppose that the two conditions above are hold.
Since S 6= {0} (by hypothesis), we have there is 0 6= a ∈ S.
By condition (1), a − a ∈ S and hence 0 ∈ S.
By condition (2), a.a−1 ∈ S and hence 1 ∈ S.
Let 0 6= b ∈ S, thus from condition (2) we have 1.b−1 ∈ S and hence b−1 ∈ S.
Then every non-zero element in S has a multiplication inverse.
Let a, b ∈ S. By condition (1), a − b ∈ S.
If b = 0, then a.b = a.0 = 0 ∈ S.
If b 6= 0, then b−1 ∈ S and b = (b−1 )−1 .
So, a.b = a.(b−1 )−1 ∈ S (by condition (2)).
Thus for any a, b ∈ S, we have a − b, a.b ∈ S.
By Theorem 1.28, (S, +, .) is a subring of the ring (F, +, .).
Since (F, +, .) is a commutative ring, we have (S, +, .) is a commutative ring.
Therefore, (S, +, .) is a commutative ring with identity in which every non-zero ele-
ment in S has a multiplication inverse.
Hence (S, +, .) is a field and so (S, +, .) is a subfield of the field (F, +, .).

Department of Mathematics/ College of Education/ Al-Qadisiyah University 54 of 124


Ring Theory Prof. Dr. Akeel Ramadan Mehdi Al-Yassiri


Exercise 60. Let F = {a + b p | a, b ∈ Q}, where p is a prime number. By using
Theorem 4.16, Prove that (F, +, .) is a subfield of the field of real numbers (R, +, .).

Proposition 4.17. Let (S, +, .) be a subfield of the field (F, +, .). Then the fields
(S, +, .) and (F, +, .) have the same identity element.
Proof. Let 1S and 1F be the identity elements of the fields (S, +, .) and (F, +, .),
respectively.
Since 1S ∈ F, we have 1S · 1F = 1S .
Since 1S ∈ S, we have 1S · 1S = 1S .
Thus 1S · 1F = 1S · 1S .
Since 1S 6= 0 and the cancellation laws for multiplication on F are holding,
we have 1F = 1S .

Corollary 4.18. If (F, +, .) is a field of characteristic n ≥ 1, then the characteristic


of any subfield of (F, +, .) is n too.
Proof. Suppose that (F, +, .) is a field of characteristic n ≥ 1.
Let (S, +, .) be a subfield of the field (F, +, .).
By Theorem 1.61, n is the least positive integer such that n1F = 0.
By Proposition 4.17, 1F = 1S
and hence n is the least positive integer such that n1S = 0.
By Theorem 1.61, n is the characteristic of (S, +, .).

Theorem 4.19. Let (F1 , +, .) and (F2 , +, .) be two subfields of the field (F, +, .).
Then (F1 ∩ F1 , +, .) is a subfield of the field (F, +, .).
Proof. By Proposition 4.17, 0 6= 1F ∈ F1 and 0 6= 1F ∈ F2 and hence 0 6= 1F ∈ F1 ∩ F2 .
Thus {0} 6= F1 ∩ F2 ⊆ F
Let a, b ∈ F1 ∩ F2 , thus a, b ∈ F1 and a, b ∈ F2 .
Since (F1 , +, .) and (F2 , +, .) are subfields of the field (F, +, .),
we have from Theorem 4.16 that a − b ∈ F1 and a − b ∈ F2 and hence a − b ∈ F1 ∩ F2 .
Let a, b ∈ F1 ∩ F2 , with b 6= 0.
Thus a, b ∈ F1 and a, b ∈ F2 .
Since (F1 , +, .) and (F2 , +, .) are subfields of the field (F, +, .),
we have from Theorem 4.16 that a.b−1 ∈ F1 and a.b−1 ∈ F2
and hence a.b−1 ∈ F1 ∩ F2 .
By Theorem 4.16, (F1 ∩ F1 , +, .) is a subfield of the field (F, +, .).

Department of Mathematics/ College of Education/ Al-Qadisiyah University 55 of 124


Ring Theory Prof. Dr. Akeel Ramadan Mehdi Al-Yassiri

Definition 4.20. A field (F, +, .) is said to be prime field is it is has no proper sub-
field.

Example 4.21. The field (Q, +, .) is a prime field.


Proof. Assume that (Q, +, .) is not a prime field.
Thus Q has a proper subfield, say F.
By Proposition 4.17, 1 ∈ F.
Let n ∈ Z.
{z... + 1} ∈ F.
If n > 0, then n = n.1 = 1| + 1 +
n times
If n < 0, then n = −(−n).
Since −n ∈ F (as above), we have −(−n) ∈ F and hence n ∈ F.
If n = 0, then n ∈ F and hence Z ⊆ F.
Let q ∈ Q, thus q = n/m for some n, m ∈ Z with m 6= 0.
Since F is a field, we have from Theorem 4.16 that n.m−1 ∈ F.
Since q = n/m = n.m−1 , we have q ∈ F and hence Q ⊆ F.
Thus F = Q and this is a contradiction.
Therefore, the field (Q, +, .) is a prime field.

Example 4.22. The field (Z p , + p , . p ) is a prime field, where p is a prime number.


Proof. Let p be a prime number.
Assume that (Z p , + p , . p ) is not a prime field.
Thus (Z p , + p , . p ) has a proper subfield, say (F, + p , . p ).
Since (F, + p ) is a subgroup of the group (Z p , + p ), we have from Lagrange’s Theo-
rem that the order (number of elements) of F (o(F)) divides the order of Z p .
Since p is a prime number, we have o(F) = p and hence
F = Z p and this is a contradiction.
Therefore, the field (Z p , + p , . p ) is a prime field.

Exercise 61. Prove that every subfield of the field of real numbers (R, +, .) must be
contains the field of rational numbers (Q, +, .).

Exercise 62. Let (F, +, .) be a field and let α : F → F be a ring epimorphism.


Let H = {a ∈ F | α(a) = a}. Show that whether (H, +, .) is a subfield of the field
(F, +, .) or not and why?

Department of Mathematics/ College of Education/ Al-Qadisiyah University 56 of 124


Ring Theory Prof. Dr. Akeel Ramadan Mehdi Al-Yassiri

5 Certain Special Ideals


5.1 Maximal and Minimal ideals
Definition 5.1. Let R be a ring. An ideal I of a ring R is said to be maximal ideal in
R if I satisfies the following two conditions:
(1) I 6= R;
(2) There is no ideal J of R such that I $ J $ R. That is if J is an ideal of R such that
I $ J ⊆ R, then J = R.

Example 5.2. The ideal < 2 >= {0, 2, 4, 6} is the only maximal ideal in the ring
(Z8 , +8 , .8 ).
Proof. All ideals in the ring (Z8 , +8 , .8 ) are:
< 0 >= {0}, < 2 >= {0, 2, 4, 6},
< 4 >= {0, 4} and Z8 .
< 0 >= {0} is not maximal, since there is an ideal < 2 > of
(Z8 , +8 , .8 ) with < 0 >$< 2 >$ Z8 .
< 4 > is not maximal, since there is an ideal < 2 > of
(Z8 , +8 , .8 ) with < 4 >$< 2 >$ Z8 .
< 2 > is a maximal ideal in (Z8 , +8 , .8 ), since there is no ideal J of
(Z8 , +8 , .8 ) with < 2 >$ J $ Z8 .
(Z8 , +8 , .8 ) is not maximal ideal (from the definition of maximal ideal).
Thus the ideal < 2 >= {0, 2, 4, 6} is the only maximal ideal in the ring (Z8 , +8 , .8 ).

Example 5.3. Find all maximal ideals in the ring (Z10 , +10 , .10 ).
Proof. All ideals in the ring (Z10 , +10 , .10 ) are:
< 0 >= {0}, < 2 >= {0, 2, 4, 6, 8},
< 5 >= {0, 5} and Z10 .
< 0 >= {0} is not maximal, since there is an ideal < 2 > of
(Z10 , +10 , .10 ) with < 0 >$< 2 >$ Z10 .
< 5 > is a maximal ideal in (Z10 , +10 , .10 ), since there is no ideal J of
(Z10 , +10 , .10 ) with < 5 >$ J $ Z10 .
< 2 > is a maximal ideal in (Z10 , +10 , .10 ), since there is no ideal J of
(Z10 , +10 , .10 ) with < 2 >$ J $ Z10 .
(Z10 , +10 , .10 ) is not maximal ideal (from the definition of maximal ideal).
Thus the ideals < 2 >= {0, 2, 4, 6, 8} and < 5 >= {0, 5} are all maximal ideals in
the ring (Z10 , +10 , .10 ).

Department of Mathematics/ College of Education/ Al-Qadisiyah University 57 of 124


Ring Theory Prof. Dr. Akeel Ramadan Mehdi Al-Yassiri

Example 5.4. All maximal ideals in the ring (Zn , +n , .n ) are of the form
(< p >, +n , .n ), where p is a prime number that divides n.

Example 5.5. Find all maximal ideals in the ring (Z12 , +12 , .12 ).
Proof. Since 2, 3 are all prime numbers divide 12, we have from Example 5.4 that
< 2 >= {0, 2, 4, 6, 8, 10} and < 3 >= {0, 3, 6, 9} are all maximal ideals in the
ring (Z12 , +12 , .12 ).

Example 5.6. Find all maximal ideals in the ring (Z252 , +252 , .252 ).
Proof. Since 2, 3, 7 are all prime numbers divide 252, we have from Example 5.4 that
< 2 >, < 3 > and < 7 > are all maximal ideals in the ring (Z252 , +252 , .252 ).

Theorem 5.7. Let R be a commutative ring with identity. Then R is a field if and
only if < 0 > is a maximal ideal in R.
Proof. (⇒) Suppose that R is a field.
By Theorem 4.12, < 0 > and R are all ideals in R.
< 0 > is a maximal ideal in R, since there is no ideal J of R with < 0 >$ J $ R.
Also, R is not maximal ideal in R (from the definition of maximal ideal).
Therefore, < 0 > is the only maximal ideal in R.
(⇐) Suppose that < 0 > is a maximal ideal in R.
Assume that R is not a field.
By Theorem 4.12, there is an ideal J in R with < 0 >$ J $ R
and this contradicts the maximality of < 0 >.
Therefore, R is a field.

Example 5.8. The zero ideal is the only maximal ideal in the fields (Q, +, .), (R, +, .),
(C, +, .) and (Z p , + p , . p ), where p is any prime number.

Remark 5.9. Let R be a ring and let I be a proper ideal of R. If a ∈ R and a ∈


/ I, then
we will use the symbol < I, a > to denote the ideal < I ∪ {a} >.

Theorem 5.10. Let R be a ring and let I be a proper ideal of R. Then I is a maximal
ideal of R if and only if R =< I, a >, for all a ∈ R with a ∈
/ I.

Department of Mathematics/ College of Education/ Al-Qadisiyah University 58 of 124


Ring Theory Prof. Dr. Akeel Ramadan Mehdi Al-Yassiri

Proof. (⇒) Suppose that I is a maximal ideal of R.


Let a ∈ R with a ∈/ I. We will prove that R =< I, a >.
Since a ∈< I, a > and a ∈ / I, we have I $< I, a >⊆ R.
By maximality of I, we have R =< I, a >.
(⇐) Suppose that R =< I, a >, for all a ∈ R with a ∈ / I.
We will prove that I is a maximal ideal of R.
Assume that I is not a maximal ideal of R.
Thus there is an ideal J of R such that I $ J $ R.
Hence there is b ∈ J and b ∈/ I.
By hypothesis, R =< I, b >.
Since I $ J and b ∈ J, we have < I, b >⊆ J and hence R ⊆ J.
Thus J = R and this is a contradiction.
Therefore, I is a maximal ideal of R.

Lemma 5.11. Let R be a commutative ring with identity 1 and let I be a proper ideal
of R. If a ∈ R and a ∈
/ I, then < I, a >=< I ∪ Ra >= I + Ra.
Proof. By Exercise 31, we have < I ∪ Ra >= I + Ra.
Since I ⊆< I ∪ Ra > and a ∈< I ∪ Ra >, we have < I, a >⊆< I ∪ Ra >.
Since a ∈< I, a >, we have Ra =< a >⊆< I, a >.
Since I ⊆< I, a >, we have < I ∪ Ra >⊆< I, a > and hence < I, a >=< I ∪ Ra >.
Therefore, < I, a >=< I ∪ Ra >= I + Ra.

Theorem 5.12. Let R be a commutative ring with identity 1. A proper ideal I of R


is a maximal ideal of R if and only if the quotient ring R/I is a field.
Proof. (⇒) Suppose that I is a maximal ideal of R.
Since R is a commutative ring with 1,
we have R/I is a commutative ring with identity 1 + I.
Since I is a proper ideal in R, we have 1 ∈/ I and hence 1 + I 6= I.
Let a + I be a non-zero element of R/I. Then a + I 6= I and hence a ∈ / I.
Since I $< I, a >⊆ R, we have from maximality of I that < I, a >= R.
Since 1 ∈ R, we have 1 ∈< I, a >.
Since < I, a >= I + Ra (by Lemma 5.11), we have 1 ∈ I + Ra.
Thus there are x ∈ I and r ∈ R such that 1 = x + ra.
Now 1 + I = (x + ra) + I
= (x + I) + (ra + I)
= ra + I (since x ∈ I, x + I = I, the zero element of R/I)
= (r + I)(a + I).
This shows that the inverse of a + I exists in R/I and hence R/I is a field.

Department of Mathematics/ College of Education/ Al-Qadisiyah University 59 of 124


Ring Theory Prof. Dr. Akeel Ramadan Mehdi Al-Yassiri

(⇐) Suppose that the quotient ring R/I is a field.


Let I be a proper ideal of R. We will prove that I is a maximal ideal of R.
Suppose there exists an ideal J of R such that I $ J ⊆ R.
Then there is an element a ∈ J with a ∈ / I.
Hence a + I 6= I and thus a + I is a non-zero element of R/I.
Since R/I is a field (by hypothesis), there is an element b + I ∈ R/I
such that (b + I).(a + I) = 1 + I.
Hence ba + I = 1 + I and thus 1 − ba ∈ I and so 1 − ba ∈ J (since I $ J).
Since a ∈ J, we have ba ∈ J and hence 1 ∈ J.
Thus J = R and hence I is a maximal ideal of R.

Theorem 5.13. Let (Z, +, .) be the ring of integers and n > 1. Then the principal
ideal (< n >, +, .) is maximal if and only if n is a prime number.
Proof. (⇒) Suppose that the ideal (< n >, +, .) is a maximal ideal in the ring (Z, +, .).
By Theorem 5.12, (Z/ < n >, +, .) is a field.
Since (Z/ < n >, +, .) ∼ = (Zn , +n , .n ) (from Corollary 3.29),
we have (Zn , +n , .n ) is a field.
By Corollary 4.11, n is a prime number.
(⇐) Suppose that n is a prime number.
By Corollary 4.11, (Zn , +n , .n ) is a field.
Since (Z/ < n >, +, .) ∼ = (Zn , +n , .n ) (from Corollary 3.29),
we have (Z/ < n >, +, .) is a field.
By Theorem 5.12, (< n >, +, .) is maximal in the ring (Z, +, .).

Exercise 63. Prove that the ideal (< 4 >, +, .) is a maximal ideal in the ring of even
integers (Ze , +, .).
Exercise 64. Give an example of a commutative ring (R, +, .) and a maximal ideal
(I, +, .) of (R, +, .) such that (R/I, +, .) is not a field.
0 0
Exercise 65. Let (R, +, .) and (S, + , . ) be two commutative rings with identity and
let f : R → S be a ring epimorphism. Prove that if (I, +, .) is a maximal ideal in
0 0 0 0
the ring (R, +, .), then either ( f (I), + , . ) is a maximal ideal in the ring (S, + , . ) or
f (I) = S.
0 0
Exercise 66. Let (R, +, .) and (S, + , . ) be two commutative rings with identity and
0 0
let f : R → S be a ring epimorphism. Prove that if (I, + , . ) is a maximal ideal in the
0 0
ring (S, + , . ), then ( f −1 (I), +, .) is a maximal ideal in the ring (R, +, .).
0 0
Exercise 67. Let (R, +, .) and (S, + , . ) be two commutative rings with identity. If
f : R → S is a ring epimorphism, show that S is a field if and only if ker( f ) is a
maximal ideal of R.

Department of Mathematics/ College of Education/ Al-Qadisiyah University 60 of 124


Ring Theory Prof. Dr. Akeel Ramadan Mehdi Al-Yassiri

Definition 5.14. Let R be a ring. An ideal I of a ring R is said to be minimal ideal in


R if I satisfies the following two conditions:
(1) I 6=< 0 >;
(2) There is no ideal J of R such that 0 $ J $ I. That is if J is an ideal of R such that
0 ⊆ J $ I, then J = 0.

Example 5.15. The ideal < 4 >= {0, 4} is the only minimal ideal in the ring (Z8 , +8 , .8 ).
Proof. All ideals in the ring (Z8 , +8 , .8 ) are:
< 0 >= {0}, < 2 >= {0, 2, 4, 6},
< 4 >= {0, 4} and Z8 .
< 0 >= {0} is not minimal (from the definition of minimal ideal).
< 4 > is minimal ideal, since there is no ideal J of
(Z8 , +8 , .8 ) with < 0 >$ J $< 4 >.
< 2 > is not minimal, since there is an ideal < 4 > of
(Z8 , +8 , .8 ) with < 0 >$< 4 >$< 2 >.
(Z8 , +8 , .8 ) is not minimal, since there is an ideal < 4 > of
(Z8 , +8 , .8 ) with < 0 >$< 4 >$ Z8 .
Thus the ideal < 4 >= {0, 4} is the only minimal ideal in the ring (Z8 , +8 , .8 ).

Example 5.16. Find all minimal ideals in the ring (Z10 , +10 , .10 ).
Proof. All ideals in the ring (Z10 , +10 , .10 ) are:
< 0 >= {0}, < 2 >= {0, 2, 4, 6, 8},
< 5 >= {0, 5} and Z10 .
< 0 >= {0} is not minimal (from the definition of minimal ideal).
< 5 > is a minimal ideal in (Z10 , +10 , .10 ), since there is no ideal J of
(Z10 , +10 , .10 ) with < 0 >$ J $< 5 >.
< 2 > is a minimal ideal in (Z10 , +10 , .10 ), since there is no ideal J of
(Z10 , +10 , .10 ) with < 0 >$ J $< 2 >.
(Z10 , +10 , .10 ) is not minimal ideal, since there is an ideal < 2 > of
(Z10 , +10 , .10 ) with < 0 >$< 2 >$ Z10 .
Thus the ideals < 2 >= {0, 2, 4, 6, 8} and < 5 >= {0, 5} are all minimal ideals in the
ring (Z10 , +10 , .10 ).

Example 5.17. Find all minimal ideals in the ring (Z12 , +12 , .12 ).
Proof. Exercise.

Department of Mathematics/ College of Education/ Al-Qadisiyah University 61 of 124


Ring Theory Prof. Dr. Akeel Ramadan Mehdi Al-Yassiri

Example 5.18. The ring (Z, +, .) has no minimal ideal.


Proof. Assume that the ring (Z, +, .) has a minimal ideal, say I.
By Theorem 2.37, (Z, +, .) is a principal ideal domain
and hence I =< n > for some n ∈ Z.
Since I is a minimal ideal (by assumption), we have I 6= 0 and hence n 6= 0.
Let J =< 2n >. Since n 6= 0, we have 2n 6= 0 and hence < 0 >$ J.
It is clear that n ∈
/ J, (since if n ∈ J, then n = 2nt for some 0 6= t ∈ Z and hence
t = 1/2 and this is a contradiction.)
Thus n ∈ I and n ∈ / J and hence J $ I.
Then < 0 >$ J $ I and this contradicts the minimality of I.
Hence the ring (Z, +, .) has no minimal ideal.

Theorem 5.19. Let R be a commutative ring with identity. Then R is a field if and
only if R is a minimal ideal in R.
Proof. (⇒) Suppose that R is a field.
By Theorem 4.12, < 0 > and R are all ideals in R.
R is a minimal ideal in R, since there is no ideal J of R with < 0 >$ J $ R.
Also, < 0 > is not minimal ideal in R (from the definition of minimal ideal).
Therefore, R is the only minimal ideal in R.
(⇐) Suppose that R is a minimal ideal in R.
Assume that R is not a field.
By Theorem 4.12, there is an ideal J in R with < 0 >$ J $ R
and this contradicts the minimality of R.
Therefore, R is a field.

Proposition 5.20. Let R be a commutative ring with identity 1 and let I be an ideal
of R. Then I is a minimal ideal in R if and only if I =< a >, for any non-zero element
a ∈ I.
Proof. (⇒) Suppose that I is a minimal ideal in R.
Assume that there is 0 6= a ∈ I such that I 6=< a >.
Thus < 0 >$< a >$ I and this contradicts the minimality of I.
Hence I =< a >, for any non-zero element a ∈ I.
(⇐) Suppose that I =< a >, for any non-zero element a ∈ I.
Assume that I is not minimal ideal in R,
thus there is an ideal J such that < 0 >$ J $ I.
Then there is an element x ∈ J with x 6= 0 and hence x ∈ I.
By hypothesis, I =< x >.

Department of Mathematics/ College of Education/ Al-Qadisiyah University 62 of 124


Ring Theory Prof. Dr. Akeel Ramadan Mehdi Al-Yassiri

Since < x >⊆ J, we have I ⊆ J and hence I = J and this is a contradiction.


Thus I is a minimal ideal in R.

Exercise 68. Find all minimal ideals in the ring (Z pq , + pq , . pq ), where p, q ≥ 2 are
distinct prime numbers.

Exercise 69. Give an example of ideal I of a ring R such that I is not minimal and
not maximal ideal in R.

Exercise 70. Give an example of ideal I of a ring R such that I is a minimal and
maximal ideal in R.

Department of Mathematics/ College of Education/ Al-Qadisiyah University 63 of 124


Ring Theory Prof. Dr. Akeel Ramadan Mehdi Al-Yassiri

5.2 Krull-Zorn’s Theorem and Some Applications


Theorem 5.21. (Krull-Zorn) In a commutative ring with identity, each proper ideal
is contained in a maximal ideal.
Proof. Exercise.

Corollary 5.22. Any non-zero commutative ring with identity contains a maximal
ideal.
Proof. Let (R, +, .) be a non-zero commutative ring with identity.
Thus the zero ideal (< 0 >, +, .) is a proper ideal in R.
By Theorem 5.21(Krull-Zorn Theorem), (< 0 >, +, .) is contained in a maximal
ideal, say I in R.
Thus (R, +, .) contains a maximal ideal.

Corollary 5.23. Let R be a commutative ring with identity 1, and let a ∈ R. Then a
is invertible in R if and only if a ∈
/ I for each maximal ideal I of R.
Proof. (⇒) Suppose that a is invertible in a ring R.
Let I be a maximal ideal in R, thus I 6= R.
By Corollary 2.16, a ∈/ I.
(⇐) Suppose that a ∈ / I for each maximal ideal I of R.
We will prove that a is invertible in R.
Assume that < a >6= R.
Since (< a >, +, .) is a proper ideal in R (by assumption), we have from Theo-
rem 5.21(Krull-Zorn Theorem) that the ideal (< a >, +, .) is contained in a maximal
ideal in R, say J.
Since a ∈< a >, it follows that a ∈ J and this contradicts the hypothesis.
Thus < a >= R and hence 1 ∈< a >.
Then there is r ∈ R such that 1 = ra.
Since R is commutative, we have ar = ra = 1 and hence a is invertible in R.

Department of Mathematics/ College of Education/ Al-Qadisiyah University 64 of 124


Ring Theory Prof. Dr. Akeel Ramadan Mehdi Al-Yassiri

5.3 Local Rings and Jacobson Radical of Rings


Definition 5.24. A ring with identity (R, +, .) is called a local ring if R contains only
one maximal ideal.

Examples 5.25. (1) The following rings are local rings: (R, +, .), (Z5 , +5 , .5 ) and
(Z25 , +25 , .25 ).
(2) For any prime number p and n ∈ Z+ , the ring (Z pn , + pn , . pn ) is a local ring.
(3) Every field is a local ring but the converse is not true in general, for example
(Z4 , +4 , .4 ) is a local ring but it is not a field.
(4) For any two prime numbers p, q, the ring (Z pq , + pq , . pq ) is not local.
(5) The ring (Z, +, .) is not local.

Theorem 5.26. Let (R, +, .) be a commutative ring with identity and let I be a proper
ideal in R. Then R is a local ring with maximal ideal I if and only if every element
x∈/ I is invertible in R.
Proof. (⇒) Suppose that R is a local ring with maximal ideal I.
Let x ∈ R and x ∈ / I.
Since (I, +, .) is the unique maximal ideal in R (by hypothesis),
it follows from Corollary 5.23 that x is invertible in R.
(⇐) First we will prove that I is a maximal ideal in R.
Let J be an ideal of R such that I $ J ⊆ R.
Thus there is an element x ∈ J with x ∈/ I.
By hypothesis, x is invertible in R and hence 1 = x.x−1 ∈ J.
Thus J = R and hence I is a maximal ideal in R.
Assume that (K, +, .) is an another maximal ideal in R.
Thus K * I and I * K.
Hence there is y ∈ K and y ∈ / I.
By hypothesis, y is invertible in R and hence 1 = y.y−1 ∈ K.
Thus K = R and this contradicts the maximality of K.
Then I is the unique maximal ideal in R.
Therefore, R is a local ring with maximal ideal I.

Theorem 5.27. The only idempotent elements in a local ring are 0 and 1.
Proof. Let R be a local ring with the maximal ideal I.
Let e be an idempotent element in R.
Thus e = e2 and hence e(e − 1) = 0.
If e ∈
/ I, then e is invertible (by Theorem 5.26).

Department of Mathematics/ College of Education/ Al-Qadisiyah University 65 of 124


Ring Theory Prof. Dr. Akeel Ramadan Mehdi Al-Yassiri

Since e(e − 1) = 0, we have e − 1 = e−1 e(e − 1) = e−1 0 = 0 and hence e = 1.


If e ∈ I, then 1 − e ∈
/ I.
By Theorem 5.26, 1 − e is invertible.
Since e(e − 1) = 0, we have e = e(e − 1)(e − 1)−1 = 0(e − 1)−1 = 0.
Therefore, 0 and 1 are the only idempotent elements in a local ring R.

Exercise 71. Let R be a commutative ring with identity and let I be a maximal ideal
of R such that every element of the set {1 + x | x ∈ I} is invertible in R. Prove that R
is a local ring.

Exercise 72. Let (R, +, .) be a commutative ring with identity and let
N = {x ∈ R | x noninvertible in R}. Prove that the following conditions are equiva-
lent.
(1) R is a local ring.
(2) There is a proper ideal I of R that contains N.
(3) The set (N, +, .) is an ideal of R.
(4) The ring (R/N, +, .) is a field.

Department of Mathematics/ College of Education/ Al-Qadisiyah University 66 of 124


Ring Theory Prof. Dr. Akeel Ramadan Mehdi Al-Yassiri

Definition 5.28. Let (R, +, .) be a ring with identity. The Jacobson radical (or the
radical) of the ring R is denoted by Rad(R) and defined as the intersection of all
maximal ideals of R.
That is Rad(R) = {I | I is a maximal ideal of R}.
T

Lemma 5.29. Let (R, +, .) be a ring with identity. Then Rad(R) is an ideal of R.
Proof. Since Rad(R) = {I | I is a maximal ideal of R} (from Definition 5.28), we
T

have from Proposition 2.21 that Rad(R) is an ideal of R.

Examples 5.30. Find the Jacobson radical of the following rings:


(1) (R, +, .), (2) (Z20 , +20 , .20 ), (3) (F, +, .) where (F, +, .) is a field,
(4) (Z4 , +4 , .20 ), (5) (Z35 , +35 , .35 ), (6) (Z, +, .).
Solution:
(1) Since (R, +, .) is a field, we have from Theorem 5.7 that < 0 > is a maximal
ideal of R and hence Rad(R) = (0).

(2) Since < 2 > and < 5 > are all maximal ideals of Z20 , we have
Rad(Z20 ) =< 2 > ∩ < 5 >
= {0, 2, 4, 6, 8, 10, 12, 14, 16, 18} ∩ {0, 5, 10, 15} = {0, 10} =< 10 >.

(3) Since (F, +, .) is a field, we have from Theorem 5.7 that < 0 > is a maximal ideal
of F and hence Rad(F) = (0).

(4) Since < 2 > is the only maximal ideal in Z4 , we have Rad(Z4 ) =< 2 >

(5) Since < 5 > and < 7 > are all maximal ideals of Z35 , we have
Rad(Z35 ) =< 5 > ∩ < 7 >=< 0 >.

(6) By Theorem 5.13, the maximal ideals in the ring (Z, +, .) are
(< p >, +, .) for any prime number p > 1.
Thus Rad(Z) = {I | I is a maximal ideal of Z}
T

= {< p >| p is a prime number with p > 1}


T

< a >, where a is a common multiple of all prime numbers.


Since 0 is the only common multiple of all prime numbers, we have Rad(Z) =< 0 >.

Department of Mathematics/ College of Education/ Al-Qadisiyah University 67 of 124


Ring Theory Prof. Dr. Akeel Ramadan Mehdi Al-Yassiri

Theorem 5.31. Let R be a commutative ring with identity and let I be an ideal in R.
Then I ⊆Rad(R) if and only if every element of the coset 1 + I has an inverse in R.
Proof. (⇒) Suppose that I ⊆Rad(R).
Let 1 + a ∈ 1 + I, thus a ∈ I.
By hypothesis, a ∈Rad(R) and hence a ∈ J for all maximal ideals J of R.
Thus 1 + a ∈/ J for all maximal ideals J of R.
By Corollary 5.23, 1 + a is invertible in R.
(⇐) Suppose that every element of the coset 1 + I has an inverse in R.
Assume that I *Rad(R).
Then there is a maximal ideal, say J, such that I * J and
hence there is x ∈ I with x ∈
/ J.
By Theorem 5.10, R =< J, x >.
By Lemma 5.11, < J, x >= J + Rx and hence R = J + Rx.
Since Rx ⊆ I, we have R = J + I and hence 1 = a + b, for some a ∈ J and b ∈ I.
Since a = 1+(−b), we have a ∈ 1+I and hence a has an inverse in R (by hypothesis).
Since a ∈ J, we have 1 = aa−1 ∈ J.
So J = R and this contradicts the maximality of J.
Therefore, I ⊆Rad(R).

Theorem 5.32. Let R be a commutative ring with identity. Then an element a ∈Rad(R)
if and only if 1 + r.a has an inverse in R for each r ∈ R.
Proof. (⇒) Let a ∈Rad(R), thus < a >⊆Rad(R).
By Theorem 5.31, every element of the coset 1+ < a > has an inverse in R.
Since 1+ < a >= {1 + r.a | r ∈ R}, we have 1 + r.a has an inverse in R for each r ∈ R.
(⇐) Suppose that 1 + r.a has an inverse in R for each r ∈ R.
Since 1+ < a >= {1 + r.a | r ∈ R}, we have
every element of the coset 1+ < a > has an inverse in R.
By Theorem 5.31, < a >⊆Rad(R).
Since a ∈< a >, we have a ∈Rad(R).

Corollary 5.33. Let (R, +, .) be a commutative ring with identity and let N be the
set of all noninvertible elements of R. Then the triple (N, +, .) is an ideal in the ring
(R, +, .) if and only if N =Rad(R).
Proof. (⇒) Suppose that (N, +, .) is an ideal in the ring (R, +, .).
By Exercise 72, R is a local ring and the ring (R/N, +, .) is a field.
By Theorem 5.12, (N, +, .) is a maximal ideal in R and hence N =Rad(R).
(⇐) Suppose that N =Rad(R).
By Lemma 5.29, Rad(R) is an ideal in R and hence (N, +, .) is an ideal in the ring
(R, +, .).

Department of Mathematics/ College of Education/ Al-Qadisiyah University 68 of 124


Ring Theory Prof. Dr. Akeel Ramadan Mehdi Al-Yassiri

Exercise 73. Describe the radical of the ring (Zn , +n , .n ).

Exercise 74. Let R be a commutative ring with identity. Prove that an element a ∈ R
is invertible in R if and only if the coset a+Rad(R) is invertible in the quotient ring
(R/Rad(R), +, .).

Exercise 75. Let R be a commutative ring with identity. Prove that the zero element
is the only idempotent in Rad(R).

Definition 5.34. A ring with identity (R, +, .) is said to be semisimple if it has zero
Jacobson radical.

Theorem 5.35. Let (R, +, .) be a commutative ring with identity. Then the quotient
ring (R/Rad(R), +, .) is semisimple.
Proof. Since (R, +, .) be a commutative ring with identity and Rad(R) is an ideal
in R, we have that (R/Rad(R), +, .) is commutative ring with identity (by Proposi-
tion 2.63).
More simply, in this proof, we will denote to Rad(R) by I.
Let a + I ∈Rad(R/I).
We will prove that a + I = I.
By Theorem 5.32, the element (1 + I) + ((r + I).(a + I)) = (1 + r.a) + I is invertible
in the ring R/I, for each r ∈ R.
Thus there is an element b + I ∈ R/I (which depends on both r and a)
such that ((1 + r.a) + I).(b + I) = 1 + I and hence b + r.a.b − 1 ∈ I =Rad(R).
By Theorem 5.32, the element b + r.a.b = 1 + 1.(b + r.a.b − 1) is invertible in the
ring R and hence there is an element c ∈ R such that (b + r.a.b).c = 1.
Thus (1 + r.a).(b.c) = 1 and hence 1 + r.a is invertible in the ring R, for each r ∈ R.
By Theorem 5.32, the element a ∈Rad(R) and hence a+Rad(R)=Rad(R).
Therefore, Rad(R/Rad(R)) = {Rad(R)} and hence the quotient ring (R/Rad(R), +, .)
is semisimple.

Exercise 76. Let R be a commutative ring with identity and let I be an ideal of R
such that the quotient ring (R/I, +, .) is semisimple, prove that Rad(R) ⊆ I.

Department of Mathematics/ College of Education/ Al-Qadisiyah University 69 of 124


Ring Theory Prof. Dr. Akeel Ramadan Mehdi Al-Yassiri

5.4 Nil Ideals


Definition 5.36. Let (R, +, .) be a ring with identity. An ideal (I, +, .) of R is said to
be a nil ideal if each of its elements is nilpotent.

Proposition 5.37. Let (R, +, .) be a commutative ring with identity. If (I, +, .) is a


nil ideal of R, then I ⊆Rad(R).
Proof. Let 1 + a ∈ 1 + I, thus a ∈ I.
Hence there is n ∈ Z+ such that an = 0 and so (−a)n = 0.
Thus 1 − (−a)n = (1 − (−a)).(1 + (−a) + (−a)2 + · · · + (−a)n−1 ) and
hence (1 − (−a)).(1 + (−a) + (−a)2 + · · · + (−a)n−1 ) = 1.
Then 1 + a = 1 − (−a) is invertible in R and
hence every element of the coset 1 + I has an inverse in R.
By Theorem 5.31, I ⊆Rad(R).

Corollary 5.38. If (R, +, .) is a semisimple ring, then (< 0 >, +, .) is the only nil
ideal in R.
Proof. Suppose that (R, +, .) is a semisimple ring, thus Rad(R) =< 0 >.
Let (I, +, .) be any nil ideal in R.
By Proposition 5.37, I ⊆Rad(R) =< 0 >.
Hence I =< 0 >.

Proposition 5.39. Let (R, +, .) be a commutative ring with identity. Then a is a


nilpotent element in R if and only if (< a >, +, .) is a nil ideal in R.
Proof. (⇒) Suppose that a is a nilpotent element in R.
Since R is commutative, we have
from Proposition 2.6 that (< a >, +, .) is an ideal in R.
Let x ∈< a >, thus x = r.a for some r ∈ R.
Since a is a nilpotent element in R, there is n ∈ Z+ such that an = 0
and hence xn = (r.a)n = rn .an = rn .0 = 0.
Thus every element in the ideal (< a >, +, .) is nilpotent
and hence (< a >, +, .) is a nil ideal in R.
(⇐) Suppose that (< a >, +, .) is a nil ideal in R.
Since a ∈< a >, we have from the definition of nil ideal that
a is a nilpotent element in R.

Department of Mathematics/ College of Education/ Al-Qadisiyah University 70 of 124


Ring Theory Prof. Dr. Akeel Ramadan Mehdi Al-Yassiri

Corollary 5.40. Let (R, +, .) be a commutative ring with identity. Then < a >⊆Rad(R),
for any nilpotent element a in R.
Proof. Let a be any nilpotent element in R.
By Proposition 5.39, (< a >, +, .) is a nil ideal in R.
By Proposition 5.37, < a >⊆Rad(R).

Examples 5.41. (1) The zero ideal (< 0 >, +, .) is a nil ideal in any commutative
ring with identity (R, +, .).
(2) The ideals (< 0 >, +4 , .4 ) and (< 2 >, +4 , .4 ) are all nil ideals in the ring (Z4 , +4 , .4 ).
(3) The zero ideal (< 0 >, +6 , .6 ) is the only nil ideal in the ring (Z6 , +6 , .6 ).
(4) The zero ideal (< 0 >, +, .) is the only nil ideal in the ring (Z, +, .).

Exercise 77. Find all nil ideals in the following rings:


(1) (Z9 , +9 , .9 ), (2) (Z30 , +30 , .30 ), (3) (Q, +, .), (4) (Z12 , +12 , .12 ).

Exercise 78. Let (R, +, .) be a commutative ring with identity. Prove that every
nilpotent element in R is noninvertible in R.

Exercise 79. Let (R, +, .) be a commutative ring with identity. Prove that if a is a
nilpotent element in R, then 1 + r.a is invertible in R, for all r ∈ R.

Department of Mathematics/ College of Education/ Al-Qadisiyah University 71 of 124


Ring Theory Prof. Dr. Akeel Ramadan Mehdi Al-Yassiri

5.5 Prime Ideals


Definition 5.42. Let (R, +, .) be a ring with identity. An ideal (I, +, .) of R is said to
be a prime ideal if for all a, b ∈ R with a.b ∈ I, then either a ∈ I or b ∈ I.

Lemma 5.43. (Euclid’s Lemma) Let p be a prime number and let a, b ∈ Z. If p|a.b,
then either p|a or p|b.

Examples 5.44. (1) The ideal (R, +, .) is a prime ideal in the ring (R, +, .).
(2) The ideal (< 2 >, +, .) is a prime ideal in the ring (Z, +, .).
Proof: Let a, b ∈ Z such that a.b ∈< 2 >.
Thus 2 divides a.b.
By Lemma 5.43(Euclid’s Lemma), either 2 divides a or 2 divides b.
By Lemma 2.39, either < a >⊆< 2 > or < b >⊆< 2 >
and hence either a ∈< 2 > or b ∈< 2 >.
Thus (< 2 >, +, .) is a prime ideal in the ring (Z, +, .).
(3) The ideal (< 6 >, +, .) is not a prime ideal in the ring (Z, +, .),
since 2.3 = 6 ∈< 6 > but neither 2 ∈< / 6 > nor 3 ∈< / 6 >.
(4) In the ring (Z4 , +4 , .4 ), the prime ideals are: (Z4 , +4 , .4 ) and (< 2 >, +4 , .4 ).
Proof: All ideals in the ring (Z4 , +4 , .4 ) are
(< 0 >, +4 , .4 ), (Z4 , +4 , .4 ) and (< 2 >, +4 , .4 ).
- (< 0 >, +4 , .4 ) is not prime ideal in Z4 , since 2.4 2 = 0 ∈< 0 >, but 2 ∈< / 0 >.
- (Z4 , +4 , .4 ) is a prime ideal in (Z4 , +4 , .4 ) and this from Example (1) above.
- (< 2 >, +4 , .4 ) is a prime ideal in (Z4 , +4 , .4 ), since
if a, b ∈ Z4 such that a.4 b ∈< 2 >= {0, 2} and b ∈< / 2 >, then either b = 1 or b = 3.
We will prove that a ∈< 2 >.
(i) If b = 1, then a = a.4 1 = a.4 b ∈< 2 >.
(ii) If b = 3, then 3.4 a ∈< 2 >.
Assume that a ∈< / 2 >, thus either a = 1 or a = 3.
(ii − 1) If a = 1, then a.4 b = 1.4 3 = 3 ∈< / 2 > and this is a contradiction.
(ii − 2) If a = 3, then a.4 b = 3.4 3 = 1 ∈< / 2 > and this is a contradiction.
Thus a ∈< 2 > and hence (< 2 >, +4 , .4 ) is a prime ideal in (Z4 , +4 , .4 ).

Theorem 5.45. A commutative ring with identity (R, +, .) is an integral domain if


and only if the zero ideal (< 0 >, +, .) is a prime ideal in R.
Proof. Exercise.

Department of Mathematics/ College of Education/ Al-Qadisiyah University 72 of 124


Ring Theory Prof. Dr. Akeel Ramadan Mehdi Al-Yassiri

Theorem 5.46. Let (I, +, .) be a proper ideal of a commutative ring (R, +, .) with
identity 1. Then I is a prime ideal in R if and only if the quotient ring R/I is an
integral domain.
Proof. (⇒) Suppose that I is a prime ideal in the ring R.
Since R is a commutative ring with 1, we have
the ring R/I is also a commutative ring with
identity element 1 + I and zero element 0 + I = I.
Since I is a proper ideal in R, we have 1 + I 6= I.
Let a + I, b + I ∈ R/I such that (a + I).(b + I) = I.
Thus a.b + I = I and hence a.b ∈ I.
Since I is a prime ideal in R (by hypothesis), we have either a ∈ I or b ∈ I and hence
either a + I = I or b + I = I.
Thus the ring R/I has no zero divisors
and so the quotient ring R/I is an integral domain.
(⇐) Suppose that the quotient ring R/I is an integral domain.
Let a, b ∈ R such that a.b ∈ I.
Then a.b + I = I and hence (a + I).(b + I) = I.
Since R/I has no zero divisors, we have either a + I = I or b + I = I
and hence either a ∈ I or b ∈ I.
Thus I is a prime ideal in R.

Theorem 5.47. The prime ideals in the ring (Z, +, .) are (< 0 >, +, .), (Z, +, .) and
(< p >, +, .) for any prime number p.
Proof. (Z, +, .) is a prime ideal in (Z, +, .) (by the definition of prime ideal).
Since (Z, +, .) is an integral domain, we have
from Theorem 5.45 that (< 0 >, +, .) is a prime ideal in (Z, +, .).
Let (< n >, +, .) be a prime ideal in (Z, +, .) with n > 1.
We will prove that n is a prime number.
By Theorem 5.46, the ring (Z/ < n >, +, .) is an integral domain.
Since (Z/ < n >, +, .) ∼ = (Zn , +n , .n ) (by Corollary 3.29), we have
(Zn , +n , .n ) is an integral domain.
By Theorem 1.37, n is a prime number.

Theorem 5.48. Let R be a commutative ring with identity. Then every maximal
ideal of R is a prime ideal.
Proof. Let I be a maximal ideal in R.
Thus I is a proper ideal of R.
By Theorem 5.12, the quotient ring R/I is a field.

Department of Mathematics/ College of Education/ Al-Qadisiyah University 73 of 124


Ring Theory Prof. Dr. Akeel Ramadan Mehdi Al-Yassiri

By Theorem 4.8, the quotient ring R/I is an integral domain.


By Theorem 5.46, I is a prime ideal in R.

Remarks 5.49. (1) If a ring R without identity element, then Theorem 5.48 does
not remain valid, for example the ideal (< 4 >, +, .) is a maximal ideal in the ring
(Ze , +, .) (from Exercise 63) but it is not prime.
(2) The converse of Theorem 5.48 is not true in general, for example (Z, +, .) and
(< 0 >, +, .) are prime ideals in (Z, +, .) but they are not maximal.

Theorem 5.50. Let (R, +, .) be a principal ideal domain. Then a nontrivial ideal of
(R, +, .) is prime if and only if it is a maximal ideal.
Proof. (⇒) Let (I, +, .) be a nontrivial prime ideal in the PID (R, +, .).
Suppose J is an ideal of R such that I $ J ⊆ R.
We will prove that J = R.
Since R is a PID, we write I =< a > and J =< b > for some a, b ∈ R − {0}.
Since a ∈< a >$< b >, we have a = r.b for some r ∈ R.
Since < a > is a prime ideal, it follows that either r ∈< a > or b ∈< a >.
If b ∈< a >, then < b >⊆< a > and this is a contradiction and hence r ∈< a >.
Then r = t.a for some t ∈ R and hence a = r.b = (t.a).b = (t.b).a.
Thus (1 − t.b).a = 0.
Since a 6= 0 and R is an integral domain (by hypothesis),
we have 1 − t.b = 0 and hence t.b = 1.
Thus 1 ∈< b >= J and so J = R.
Therefore, I is a maximal ideal of R.
(⇐) Let (I, +, .) be a nontrivial maximal ideal in the PID (R, +, .).
Thus R is a commutative ring with identity.
By Theorem 5.48, (I, +, .) is a prime ideal in (R, +, .).

Corollary 5.51. A non-trivial ideal of (Z, +, ) is maximal iff it is prime.


Proof. Let (I, +, ) be a non-trivial ideal of (Z, +, ).
By Theorem 2.37, the ring (Z, +, .) is a principal ideal domain.
By Theorem 5.50, the ideal (I, +, ) is prime if and only if it is a maximal ideal in
(Z, +, ).

Exercise 80. Let A and B be ideals in a commutative ring R with identity. Prove that
if A ∩ B is a prime ideal in R, then either A ⊆ B or B ⊆ A.

Department of Mathematics/ College of Education/ Al-Qadisiyah University 74 of 124


Ring Theory Prof. Dr. Akeel Ramadan Mehdi Al-Yassiri

0 0
Exercise 81. Let (R, +, .) and (S, + , . ) be two commutative rings with identity and
let f : R → S be a ring epimorphism. Prove that if (I, +, .) is a prime ideal in the ring
0 0 0 0
(R, +, .) with ker( f ) ⊆ I, then ( f (I), + , . ) is a prime ideal in the ring (S, + , . ).

0 0
Exercise 82. Let (R, +, .) and (S, + , . ) be two commutative rings with identity and
0 0
let f : R → S be a ring homomorphism. Prove that if (I, + , . ) is a prime ideal in the
0 0
ring (S, + , . ), then ( f −1 (I), +, .) is a prime ideal in the ring (R, +, .).

Department of Mathematics/ College of Education/ Al-Qadisiyah University 75 of 124


Ring Theory Prof. Dr. Akeel Ramadan Mehdi Al-Yassiri

5.6 Primary Ideals


Definition 5.52. Let R be a commutative ring with identity. A proper ideal I of R is
/ I, then an ∈ I for some
said to be a primary ideal if for a, b ∈ R with a.b ∈ I and b ∈
positive integer n.

Lemma 5.53. Let p > 1 be a prime number and let a, b ∈ Z. If p2 | a.b and p2 is not
divide a, then p | b.
Proof. Exercise.

Example 5.54. The ideal I =< p2 > in the ring (Z, +, .) is primary, for any prime
number p.
Proof. Let a, b ∈ Z such that a.b ∈ I =< p2 > and b ∈ / I =< p2 >.
To prove that an ∈ I =< p2 >, for some n ∈ Z+ .
Since a.b ∈ I =< p2 >, we have p2 divides a.b.
Since p divides p2 , we have p divides a.b.
By Lemma 5.43(Euclid’s Lemma), either p divides a or p divides b.
/ p2 > (by hypothesis), we have p2 is not divide b.
Since b ∈<
By Lemma 5.53, p divides a and hence p2 divides a2 .
Thus < a2 >⊆< p2 > and hence a2 ∈< p2 >.
Therefore, there is n = 2 such that an ∈< p2 > and
hence the ideal I =< p2 > in the ring (Z, +, .) is primary.

Remark 5.55. It is clear that every prime ideal of a ring R is a primary ideal, but the
converse is not true in general, for example the ideal (< 4 >, +, .) is a primary ideal
in the ring (Z, +, .) (by Example 5.54) but it is not prime.

Proposition 5.56. The primary ideals in Z are < 0 > and < pn >, for prime numbers
p and positive integers n.
Proof. Exercise.

Example 5.57. The ideal I =< pn11 .pn22 . · · · pnk k > in the ring (Z, +, .) is not primary,
for any distinct prime numbers p1 , p2 , · · · pk and k > 1.
Proof. This follows from Proposition 5.56.

Department of Mathematics/ College of Education/ Al-Qadisiyah University 76 of 124


Ring Theory Prof. Dr. Akeel Ramadan Mehdi Al-Yassiri

Theorem 5.58. Let (R, +, .) be a commutative ring with identity and let (I, +, .) be
a proper ideal in R. Then I is a primary ideal in R if and only if every zero-divisor in
the ring (R/I, +, .) is nilpotent.
Proof. (⇒) Suppose that I is a primary ideal of R.
Let a + I 6= I be a zero-divisor in the ring (R/I, +, .),
then there exists b + I 6= I in (R/I, +, .) such that (a + I).(b + I) = I.
Then a.b ∈ I and b ∈ / I.
Hence a ∈ I for some n ∈ Z+ .
n

So (a + I)n = an + I = I.
Thus a + I is nilpotent in (R/I, +, .).
(⇐) Suppose that every zero-divisor in R/I is nilpotent.
Let a, b ∈ R such that a.b ∈ I and b ∈ / I.
If a ∈ I, then I is a primary ideal in R.
If a ∈
/ I, then a.b + I = I, b + I 6= I and a + I 6= I.
Therefore (a + I).(b + I) = I, b + I 6= I and a + I 6= I.
Then a + I is a zero-divisor and hence (a + I)n = I for some positive integer n.
Thus an ∈ I and hence I is a primary ideal in R.

Exercise 83. Prove that all ideals in the ring (Z4 , +4 , .4 ) are primary.

Exercise 84. By using the definition of primary ideal, prove that the ideal
(< 9 >, +, .) in the ring (Z, +, .) is primary.

Exercise 85. Give an example of a ring contains infinite many of primary ideals and
non primary ideals.

Exercise 86. Let (R, +, .) be a commutative ring with identity. Prove that the zero
ideal (< 0 >, +, .) is a primary ideal in R if and only if every zero-divisor in R is
nilpotent.

Department of Mathematics/ College of Education/ Al-Qadisiyah University 77 of 124


Ring Theory Prof. Dr. Akeel Ramadan Mehdi Al-Yassiri

5.7 Radical of Ideals and Semiprime Ideals


Definition 5.59.
√ Let (I, +, .) be an ideal of the ring R. The radical of the ideal I is
denoted by I and defined as follows:

I = {a ∈ R | an ∈ I for some positive integer n}.

Proposition√5.60. Let (I, +, .) be an ideal of√a commutative ring (R, +, .) with iden-
tity. Then ( I, +, .) is an ideal in R and I ⊆ I.
√ √
n = 0 for any n ∈ Z+ , we have 0 ∈ I and hence φ 6= I ⊆ R.
Proof. Since√ 0 ∈ I and 0
Let a, b ∈ I, thus there are n, m ∈ Z+ such that an , bm ∈ I.
By the Newton’s binomial
 formula,
 we have  
n + m n + m (n+m)!
(a − b)n+m = ∑n+m i=0 an+m−i .(−b)i , where = i!(n+m−i)! .
i i
If 0 ≤ i ≤ m, then m − i ≥ 0 and hence
an+m−i .(−b)i = an .(am−i .(−b)i ).
Since an ∈ I and (I, +, .) is an ideal of R, we have
an .(am−i .(−b)i ) ∈ I and hence an+m−i .(−b)i ∈ I.
If m < i ≤ n + m, then i − m ≥ 0 and hence
an+m−i .(−b)i = an+m−i .((−b)m .(−b)i−m ).
Since (−b)m = (−1)m .(bm ) ∈ I and (I, +, .) is an ideal of R, we have
an+m−i .((−b)m .(−b)i−m ) ∈ I.
Hence an+m−i  .(−b) 
i ∈ I.

n+m
Thus ∑n+m
i=0 an+m−i .(−b)i ∈ I and so (a − b)n+m ∈ I.
i

Therefore,
√ a − b ∈ I.
Let a ∈ I and r ∈ R, thus an ∈ I for some n ∈ Z+ .
Since (r.a)n = rn .an and (I, +, .) is an ideal in R, we have√
rn .an ∈ I and hence (r.a)n ∈ I and this implies
√ that r.a ∈√ I.
Since R is commutative, we have √ a.r ∈ I and hence ( I, +, .) is an ideal in R.
Now, we will prove that I ⊆ I. √
Let a ∈ I, thus a√n ∈ I, where n = 1 and hence a ∈ I.
Therefore, I ⊆ I.

Proposition 5.61. Let (R, +, .) be a commutative


√ ring with identity. If (I, +, .) is a
proper ideal in (R, +, .), then the ideal ( I, +, .) is a proper in (R, +, .).
√ √
Proof. Assume that ( I, +, √ .) is not a proper in (R, +, .), that is I = R.
Since 1 ∈ R, we have 1 ∈ I and hence there is n ∈ Z+ such that 1n ∈ I and so 1 ∈ I.
Thus I = R and this is√a contradiction.
Therefore, the ideal ( I, +, .) is a proper in (R, +, .).

Department of Mathematics/ College of Education/ Al-Qadisiyah University 78 of 124


Ring Theory Prof. Dr. Akeel Ramadan Mehdi Al-Yassiri

m1 m2 mk
Example 5.62. Let I =< n > be an ideal in the ring (Z, √+, .) such that n = p1 .p2 . . . pk
with p1 , p2 , . . . , pk are distinct prime numbers. Then < n > =< p1 .p2 . . . pk >.
Proof. Exercise.


Example 5.63. Find I in the ring (Z, +, .), where
(1) I =< pn >, (2) I =< 5 >, (3) I =< 30 >,
(4) I =< 100 >, (5) I =< 10 >, (6) I =< 72 >.
√ √
Solution:
√ (1)
√ I = < pn > =< p >.
(2) √I = √< 5 > =<√ 5 >.
(3) √I = √< 30 > = √ < 2 ∗ 3 ∗ 5 > =< 2 ∗ 3 ∗ 5 >=< 30 >.
(4) √I = √< 100 > =√ < 22 ∗ 52 > =< 2 ∗ 5 >=< 10 >.
(5) √I = √< 10 > = √< 2 ∗ 5 > =< 2 ∗ 5 >=< 10 >.
(6) I = < 72 > = < 23 ∗ 32 > =< 2 ∗ 3 >=< 6 >.


Example 5.64. Find I, for any ideal I in the ring (Z8 , +8 , .8 ).
Solution: All ideals in the ring (Z8 , +8 , .8 ) are:
(<√0 >, +8 , .8 ), (< 4 >, +8 , .8 ), (< 2 >, +8 , .8 ), and (Z8 , +8 , .8 ).
- Z8 = Z8 (by Definition 5.59). √
- Since
√ < 2 >6 = Z 8 , we have from Proposition 5.61 that < 2 > 6= Z8 .
Thus < 2 > =< 2 >. √
- Since < 4 >6= Z8 , we have from Proposition
2 √ 5.61 that√ < 4 > 6= Z8 .
Since 2 = 4 ∈< 4 >, we have 2 ∈ < 4 > and hence √< 4 > =< 2 >.
- Since < 0 >6= Z8 , we have from Proposition
√ 5.61 that√ < 0 > 6= Z8 .
3
Since 2 = 0 ∈< 0 >, we have 2 ∈ < 0 > and hence < 0 > =< 2 >.

Theorem 5.65. Let √ (I, √


+, .) and (J, +, .) be two ideals in a ring (R, +, .).
(1) If I ⊆ J,
√ then √I ⊆ J. √
(2) If J ⊆ I, √then J ⊆ √ I. √
(3) If I ⊆ J ⊆ I, then J = I.

√ Suppose that I ⊆ J. +
Proof. (1)
n ∈ I.
Let x ∈ I, thus there is n ∈ Z such that x√
Since√I ⊆ J,√we have xn ∈ J and hence x ∈ J.
Thus I ⊆ J. √
(2) Suppose
√ that J ⊆ I.
Let a ∈ J, Z+ such that an ∈ J.
√thus there is nn ∈√
Since J ⊆ I, we have a ∈ √I and hence there is m ∈ Z+ such that (an )m ∈ I.
Thus an.m ∈ I and hence a ∈ I.

Department of Mathematics/ College of Education/ Al-Qadisiyah University 79 of 124


Ring Theory Prof. Dr. Akeel Ramadan Mehdi Al-Yassiri

√ √
Therefore, J ⊆ I. √
(3) Suppose that I ⊆ J ⊆ I. √ √
Since I ⊆ J,
√ we have from (1) above that I
√ ⊆ J.

Since√J⊆ √ I, we have from (2) above that J ⊆ I.
Thus J = I.

Theorem 5.66. Let (R, +, .) be√a commutative ring with identity. If (I, +, .) is a
primary ideal in (R, +, .), then ( I, +, .) is prime in R..
Proof. Suppose that I is a primary
√ ideal of
√ R.
Let a, b ∈ R such that a.b ∈ I with b ∈ / I,
thus there exists a positive integer n such that (a.b)n ∈ I.
Since R is commutative, we have an · bn ∈ I.
Since I is primary and bn ∈ / I, this yields by definition that
n m
(a ) ∈ I for some positive√integer m.
Hence √ anm ∈ I and so a ∈ I.
Thus ( I, +, .) is a prime ideal in R.

Definition 5.67. Let (R, +, .)√be a ring with identity. An ideal (I, +, .) of (R, +, .) is
said to be semiprime in R if I = I.

Example 5.68. The semiprime ideals in the ring (Z, +, .) are (< 0 >, +, .) and
(< p1 .p2 . . . pk >, +, .) for all distinct prime numbers p1 , p2 , . . . , pk .

Proof. Since < 0 > =< 0 >, we have < 0 > is a semiprime ideal in Z.

By Example 5.62, < p1 .p2 . . . pk > =< p1 .p2 . . . pk >,
for any distinct prime numbers p1 , p2 , . . . , pk and for any k ∈ Z+ .
Thus (< p1 .p2 . . . pk >, +, .) is a semiprime ideal in Z.
Let (<√n >, +, .) be any nonzero semiprime ideal in Z.
Thus < n > =< n >.
We will prove that < n >=< q1 .q2 . . . qm >,
where m ∈ Z+ and q1 , q2 , . . . , qm are distinct prime numbers.
Since n ∈ Z+ , we have n = qt11 .qt22 . . . qtmm ,
where m ∈ Z+ and q1 , q2 , q . . . , qm are distinct prime numbers.

Thus < n >= < n > = < qt11 .qt22 . . . qtmm >.
q
By Example 5.62, < qt11 .qt22 . . . qtmm > =< q1 .q2 . . . qm >.
Thus < n >=< q1 .q2 . . . qm >.

Department of Mathematics/ College of Education/ Al-Qadisiyah University 80 of 124


Ring Theory Prof. Dr. Akeel Ramadan Mehdi Al-Yassiri

√ Let (I, √
Exercise 87. +, .) and
√ (J, +, .) be two ideals in a ring (R, +, .).
Prove that I ∩ J = I ∩ J.

√ Let (I, √
Exercise 88. +, .) and (J, +, .) be two ideals in a ring (R, +, .).
Prove that I ∩ J = I.J.

Exercise 89.
p√Let (I, +, .) be an ideal in a ring (R, +, .).

Prove that I = I.

Exercise 90. Let (R, +, .) be a commutative ring with identity. Prove that every
prime ideal in R is semiprime. Give an example to show the converse is not true in
general.

Department of Mathematics/ College of Education/ Al-Qadisiyah University 81 of 124


Ring Theory Prof. Dr. Akeel Ramadan Mehdi Al-Yassiri

6 Polynomial Rings
6.1 Definitions and Some Properties
Definition 6.1. Let (R, +, .) be a ring. The set R[x] is defined on R as follows:
R[x] = {a0 + a1 x + . . . + an xn | ai ∈ R, for all 0 ≤ i ≤ n, n ∈ Z+ ∪ {0}}.
The set R[x] is called the set of polynomials over a ring R with the variable x.
Any element in R[x] is called a polynomial over a ring R with the variable x.

Remark 6.2. Let (R, +, .) be a ring. The elements in R[x] are sometimes written as
sequences of elements in R.
That is, if a0 + a1 x + . . . + an xn is a polynomial over a ring R, then we can write it as
follows:
a0 + a1 x + . . . + an xn = (a0 , a1 , . . . , an , 0, 0, 0, . . .).
Thus, we can write R[x] as follows:
R[x] = {(a0 , a1 , . . . , an , 0, 0, 0, . . .) | ai ∈ R, for all 0 ≤ i ≤ n, n ∈ Z+ ∪ {0}}.

Example 6.3. In the set of polynomials Z[x], we have:


(1) 1 + 2x2 + x5 + 3x7 = (1, 0, 2, 0, 0, 1, 0, 3, 0, 0, 0, . . .).
(2) 1 − 3x3 − 7x8 + x9 = (1, 0, 0, −3, 0, 0, 0, 0, −7, 1, 0, 0, 0, . . .).

Remark 6.4. The sequence (1, 0, 1, 0, 1, 0, . . . , 1, 0, . . .) is not a polynomial in Z[x].


So any polynomial in R[x] is a sequence of elements in R but the converse is not true
in general.

Definition 6.5. Let (R, +, .) be a ring and let f (x) = a0 + a1 x + . . . + an xn be a non-


zero polynomial in R[x] with an 6= 0. The integer n is called the degree of the poly-
nomial polynomial f (x) and denoted by deg( f (x)) and the non-zero coefficient an is
known as the leading coefficient.

Remark 6.6. The zero polynomial has no degree.

Example 6.7. (1) In the set of polynomials Z[x], we have:


- The polynomial f (x) = 1 + 2x2 + x5 + 3x7 of degree 7.
- The polynomial f (x) = 1 − 3x3 − 7x8 + x9 of degree 9.
- The polynomial f (x) = 5 of degree 0.

Department of Mathematics/ College of Education/ Al-Qadisiyah University 82 of 124


Ring Theory Prof. Dr. Akeel Ramadan Mehdi Al-Yassiri

(2) The polynomial f (x) = 2x3 − 3 has degree 3 in Q[x] and Z7 [x],
but f (x) has degree 0 in Z2 [x].
(3) The polynomial g(x) = 9x4 + 7x2 − x + 1 has degree 4 in all of Q[x], Z7 [x] and
Z2 [x].

Definition 6.8. Let (R, +, .) be a ring with identity 1. A nonzero polynomial f (x) =
a0 +a1 x+. . .+an xn is said to be a monic polynomial if the leading coefficient an = 1.
That is f (x) = a0 + a1 x + . . . + xn .

Example 6.9. (1) In the set of polynomials Z[x], we have:


- The polynomial f (x) = 1 + 2x2 + x5 + x7 is monic.
- The polynomial f (x) = 1 − 3x3 − 7x8 + 4x9 is not monic.
- The polynomial f (x) = 5 is not monic.
(2) The polynomial f (x) = 3x3 − 3 is monic in Z2 [x] but it not monic in Z[x].

Definition 6.10. Let (R, +, .) be a ring.


The addition is defined on R[x] as follows:
If f (x), g(x) ∈ R[x] such that f (x) = a0 + a1 x + . . . + an xn
and g(x) = b0 + b1 x + . . . + bm xm , then
f (x) + g(x) = c0 + c1 x + . . . + ct xt , where
ci = ai + bi for all 0 ≤ i ≤ t with t ≤max{n, m}.
max{n+m}
That is, f (x) + g(x) = (∑ni=0 ai xi ) + (∑m i
i=0 bi x ) = ∑i=0 (ai + bi )xi .
The multiplication of f (x) and g(x) is defined by:
f (x).g(x) = c0 + c1 x + . . . + cn+m xn+m , where
ck = ∑i+ j=k ai .b j , for all 0 ≤ k ≤ n + m.
That is, f (x).g(x) = (∑ni=0 ai xi ).(∑mj=0 b j x j ) = ∑n+m k
k=0 (∑i+ j=k ai .b j )x .

Proposition 6.11. If (R, +, .) is a commutative ring with identity, then (R[x], +, .)


is a commutative ring with identity, where addition (+) and multiplication (.) are
defined as in Definition 6.10.
Proof. Exercise.

Example 6.12. Compute f (x) + g(x) and f (x).g(x) for the following:
(1) f (x) = 1 + 5x − 4x2 , g(x) = 2 + 3x in the ring R[x].
(2) f (x) = 1 + x2 + 3x3 , g(x) = 3 + x3 in the ring Z4 [x].
(3) f (x) = 1 + 2x, g(x) = 1 + 3x + 2x3 in the ring Z5 [x].

Department of Mathematics/ College of Education/ Al-Qadisiyah University 83 of 124


Ring Theory Prof. Dr. Akeel Ramadan Mehdi Al-Yassiri

Solution:
(1) f (x) + g(x) = (1 + 5x − 4x2 ) + (2 + 3x) = 3 + 8x − 4x2 .
f (x).g(x) = (1 + 5x − 4x2 ).(2 + 3x)
= c0 + c1 x + c2 x2 + c3 x3 , where
ck = ∑i+ j=k ai .b j , with 0 ≤ k ≤ 3,
a0 = 1, a1 = 5, a2 = −4, a3 = 0 and b0 = 2, b1 = 3, b2 = 0, b3 = 0.
Thus,
c0 = a0 .b0 = 1 ∗ 2 = 2,
c1 = a0 .b1 + a1 .b0 = (1 ∗ 3) + (5 ∗ 2) = 3 + 10 = 13,
c2 = a0 .b2 + a1 .b1 + a2 .b0 = (1 ∗ 0) + (5 ∗ 3) + (−4 ∗ 2) = 0 + 15 − 8 = 7,
c3 = a0 .b3 + a1 .b2 + a2 .b1 + a3 .b0 = (1 ∗ 0) + (5 ∗ 0) + (−4 ∗ 3) + (0 ∗ 2) = 0 + 0 −
12 + 0 = −12.
Hence f (x).g(x) = 2 + 13x + 7x2 − 12x3 .

(2) f (x) + g(x) = (1 + x2 + 3x3 ) + (3 + x3 ) = 4 + x2 + 4x3 = x2 .


f (x).g(x) = (1 + x2 + 3x3 ).(3 + x3 )
= c0 + c1 x + c2 x2 + c3 x3 + c4 x4 + c5 x5 + c6 x6 , where
ck = ∑i+ j=k ai .4 b j , with 0 ≤ k ≤ 6,
a0 = 1, a1 = 0, a2 = 1, a3 = 3, a4 = 0, a5 = 0, a6 = 0
and b0 = 3, b1 = 0, b2 = 0, b3 = 1, b4 = 0, b5 = 0, b6 = 0.
Thus,
c0 = a0 .4 b0 = 1.4 3 = 3,
c1 = (a0 .4 b1 ) +4 (a1 .4 b0 ) = (1.4 0) +4 (0.4 3) = 0,
c2 = (a0 .4 b2 ) +4 (a1 .4 b1 ) +4 (a2 .4 b0 )
= (1.4 0) +4 (0.4 0) +4 (1.4 3)
= 0 +4 0 +4 3 = 3,
c3 = (a0 .4 b3 ) +4 (a1 .4 b2 ) +4 (a2 .4 b1 ) +4 (a3 .4 b0 )
= (1.4 1) +4 (0.4 0) +4 (1.4 0) +4 (3.4 3)
= 1 +4 0 +4 0 +4 9 = 10 = 2,
c4 = (a0 .4 b4 ) +4 (a1 .4 b3 ) +4 (a2 .4 b2 ) +4 (a3 .4 b1 ) +4 (a4 .4 b0 )
= (1.4 0) +4 (0.4 1) +4 (1.4 0) +4 (3.4 0) +4 (0.4 3)
= 0 +4 0 +4 0 +4 0 +4 0 = 0,
c5 = (a0 .4 b5 ) +4 (a1 .4 b4 ) +4 (a2 .4 b3 ) +4 (a3 .4 b2 ) +4 (a4 .4 b1 ) +4 (a5 .4 b0 )
= (1.4 0) +4 (0.4 0) +4 (1.4 1) +4 (3.4 0) +4 (0.4 3) +4 (0.4 3)
= 0 +4 0 +4 1 +4 0 +4 0 +4 0 = 1,
c6 = (a0 .4 b6 ) +4 (a1 .4 b5 ) +4 (a2 .4 b4 ) +4 (a3 .4 b3 ) +4 (a4 .4 b2 ) +4 (a5 .4 b1 ) +4 (a6 .4 b0 )
= (1.4 0) +4 (0.4 0) +4 (1.4 0) +4 (3.4 1) +4 (0.4 0) +4 (0.4 0) +4 (0.4 3)
= 0 +4 0 +4 0 +4 3 +4 0 +4 0 +4 0 = 3.
Hence f (x).g(x) = 3 + 3x2 + 2x3 + x5 + 3x6 .

(3) Exercise.

Department of Mathematics/ College of Education/ Al-Qadisiyah University 84 of 124


Ring Theory Prof. Dr. Akeel Ramadan Mehdi Al-Yassiri

Theorem 6.13. Let R be a ring and let f (x), g(x) ∈ R[x] be nonzero. Then either
f (x) + g(x) = 0 or deg( f (x) + g(x)) ≤ max{deg( f (x)),deg(g(x))}.
Proof. Let f (x), g(x) ∈ R[x] such that
f (x) = a0 + a1 x + . . . + an xn and
g(x) = b0 + b1 x + . . . + bm xm with an 6= 0 and bm 6= 0.
Thus deg( f (x)) = n and deg(g(x)) = m.
(1) If g(x) = − f (x), then f (x) + g(x) = 0.
(2) If g(x) 6= − f (x), then f (x) + g(x) 6= 0 and hence we have the following cases:
- if n > m, then f (x) + g(x) = (a0 + b0 ) + (a1 + b1 )x + (a2 + b2 )x2 + . . . +
(am + bm )xm + am+1 xm+1 + . . . + an xn .
Hence deg( f (x) + g(x)) = n ≤ max{deg( f (x)),deg(g(x))}.
- if n < m, then f (x) + g(x) = (a0 + b0 ) + (a1 + b1 )x + (a2 + b2 )x2 + . . . +
(an + bn )xn + bn+1 xn+1 + . . . + bm xm .
Hence deg( f (x) + g(x)) = m ≤ max{deg( f (x)),deg(g(x))}.
- if n = m, then f (x) + g(x) = (a0 + b0 ) + (a1 + b1 )x + (a2 + b2 )x2 + . . . + (an + bn )xn .
Then deg( f (x) + g(x)) = n = max{deg( f (x)),deg(g(x))}, if an + bn 6= 0 and
deg( f (x) + g(x)) < n = max{deg( f (x)),deg(g(x))}, if an + bn = 0.
Thus deg( f (x) + g(x)) ≤ n = max{deg( f (x)),deg(g(x))}.
Therefore, either f (x) + g(x) = 0 or deg( f (x) + g(x)) ≤ max{deg( f (x)),deg(g(x))}.

Theorem 6.14. If R is an integral domain and f (x), g(x) ∈ R[x] with f (x) 6= 0,
g(x) 6= 0, then deg( f (x).g(x)) = deg( f (x))+deg(g(x)).
Proof. Let f (x) = a0 + a1 x + . . . + an xn and
g(x) = b0 + b1 x + . . . + bm xm with an 6= 0 and bm 6= 0.
Thus f (x).g(x) = c0 + c1 x + . . . + cn+m xn+m with
cn+m = a0 bn+m +a1 bn+m−1 +a2 bn+m−2 +. . .+an−1 bm+1 +an bm +an+1 bm−1 +. . .+
an+m−1 b1 + an+m b0 .
Since an+1 = an+2 = . . . = an+m−1 = an+m = 0 and
bm+1 = bm+2 = . . . = bn+m−1 = bn+m = 0,
we have cn+m = an bm .
Since R is an integral domain and an 6= 0 and bm 6= 0,
we have an bm 6= 0 and hence cn+m 6= 0.
Thus deg( f (x).g(x)) = n + m = deg( f (x))+deg(g(x)).

Remark 6.15. If R is not an integral domain, then Theorem 6.14 is not true in gene-
ral, for example if f (x) = 2x2 and g(x) = 2x2 + 2 ∈ Z4 [x], then f (x).g(x) = 2x2 .
Hence deg( f (x).g(x)) = 2 6= 4 =deg( f (x))+deg(g(x)).

Department of Mathematics/ College of Education/ Al-Qadisiyah University 85 of 124


Ring Theory Prof. Dr. Akeel Ramadan Mehdi Al-Yassiri

Theorem 6.16. If (R, +, .) is an integral domain, then the ring (R[x], +, .) is an inte-
gral domain.
Proof. Since (R, +, .) is commutative ring with identity,
we have from Proposition 6.11 that (R[x], +, .) is a commutative ring with identity.
Let f (x), g(x) ∈ R[x] with f (x) 6= 0 and g(x) 6= 0.
Thus deg( f (x)) ≥ 0 and deg(g(x)) ≥ 0.
To prove that f (x).g(x) 6= 0.
Since (R, +, .) is an integral domain (by hypothesis), we have from Theorem 6.14
that deg( f (x).g(x)) = deg( f (x))+deg(g(x)).
Since deg( f (x))+deg(g(x)) ≥ 0, it follows that deg( f (x).g(x)) ≥ 0
and hence f (x).g(x) 6= 0.
Thus the ring (R[x], +, .) has no zero divisors and hence (R[x], +, .) is an integral
domain.

Proposition 6.17. If (R, +, .) is an integral domain, then the ring (R[x], +, .) is not a
filed.
Proof. Assume that (R[x], +, .) is a filed.
Let f (x) ∈ R[x] such that deg( f (x)) > 0, thus f (x) 6= 0.
By assumption, f (x) has an inverse, say g(x), in R[x].
Thus g(x) 6= 0 (and hence deg(g(x)) ≥ 0) with f (x).g(x) = 1.
Then deg( f (x).g(x)) =deg(1) = 0.
Since R is an integral domain, we have from Theorem 6.14 that
deg( f (x).g(x)) = deg( f (x))+deg(g(x)) and hence
deg( f (x))+deg(g(x)) = 0 and this contradicts deg( f (x)) > 0.
Thus the ring (R[x], +, .) is not a filed.

Theorem 6.18. Let (R, +, .) be a commutative ring with identity. If f (x), g(x) ∈ R[x]
with f (x) 6= 0, g(x) 6= 0 with one of the polynomial f (x) or g(x) has an invertible
leading coefficient, then deg( f (x).g(x)) = deg( f (x))+deg(g(x)).
Proof. The proof is similar to the proof of Theorem 6.14.
Let f (x) = a0 + a1 x + . . . + an xn and
g(x) = b0 + b1 x + . . . + bm xm with an 6= 0 and bm 6= 0.
Thus f (x).g(x) = c0 + c1 x + . . . + cn+m xn+m with
cn+m = a0 bn+m +a1 bn+m−1 +a2 bn+m−2 +. . .+an−1 bm+1 +an bm +an+1 bm−1 +. . .+
an+m−1 b1 + an+m b0 .
Since an+1 = an+2 = . . . = an+m−1 = an+m = 0 and
bm+1 = bm+2 = . . . = bn+m−1 = bn+m = 0,
we have cn+m = an bm .

Department of Mathematics/ College of Education/ Al-Qadisiyah University 86 of 124


Ring Theory Prof. Dr. Akeel Ramadan Mehdi Al-Yassiri

Assume that cn+m = 0, thus an bm = 0.


If an is invertible (by hypothesis), we have bm = 0 and this is a contradiction.
If bm is invertible (by hypothesis), we have an = 0 and this is a contradiction.
Thus cn+m 6= 0 and hence deg( f (x).g(x)) = n + m = deg( f (x))+deg(g(x)).

Exercise 91. Let f (x) = 1 + 2x and g(x) = 4 + x + 4x2 in Z8 [x].


Prove that deg( f (x).g(x)) 6=deg( f (x))+deg(g(x)).

Exercise 92. Prove that if (R, +, .) is an integral domain, then every polynomial
f (x) ∈ R[x] with positive degree is not invertible in R[x].

Department of Mathematics/ College of Education/ Al-Qadisiyah University 87 of 124


Ring Theory Prof. Dr. Akeel Ramadan Mehdi Al-Yassiri

6.2 Division Algorithm, Remainder Theorem and Factorization


Theorem
Theorem 6.19. (Division Algorithm for polynomials) Let (R, +, .) be a commuta-
tive ring with identity and let f (x) and g(x) be two polynomials in R[x], with g(x) 6= 0
and the leading coefficient of g(x) an invertible element. Then there exist unique po-
lynomials q(x), r(x) ∈ R[x] such that f (x) = q(x)g(x) + r(x) where either r(x) = 0 or
deg(r(x)) <deg(g(x)).
Proof. Let f (x), g(x) ∈ R[x] with g(x) 6= 0 and
the leading coefficient of g(x) an invertible element.
If f (x) = 0 or deg( f (x)) < deg(g(x)), then
we can simply take q(x) = 0 and r(x) = f (x).
If deg( f (x)) = deg(g(x)) = 0, then f (x) and g(x) are both elements in R.
So we can take q(x) = f (x)(g(x))−1 and r(x) = 0.
Thus from now on we will assume deg( f (x)) ≥ deg(g(x)).
We will use induction on deg( f (x)) to prove the existence of a suitable q(x) and r(x).
Suppose that the theorem is true for polynomials f (x) of degree less than n.
We must prove that the theorem is true for polynomials of degree n.
Suppose that deg( f (x)) = n and deg(g(x)) = m with n ≥ m.
Let f (x) = a0 + a1 x + . . . + an xn and
g(x) = b0 + b1 x + . . . + bm xm with an 6= 0 and bm 6= 0.
The polynomial f1 (x) = f (x) − ((an b−1 m ).x
n−m ).g(x) ∈ R[x].

Since an − (an b−1 n


m ).bm = 0 is the coefficient of x in the polynomial f 1 (x), we have
the degree of f1 (x) is less than n.
Since the theorem is true for polynomials of degrees less than n,
we have the theorem is true for f1 (x).
Hence there exist polynomials q1 (x) and r1 (x)
such that f1 (x) = q1 (x).g(x)+r1 (x) where either r1 (x) = 0 or deg(r1 (x)) <deg(g(x)).
Since f1 (x) = f (x) − ((an b−1 m ).x
n−m ).g(x), it follows that

f (x) = f1 (x) + ((an b−1


m ).x
n−m ).g(x) and hence

f (x) = (q1 (x).g(x) + r1 (x)) + ((an b−1m ).x


n−m ).g(x)

= (q1 (x) + (an b−1m )x


n−m )g(x) + r (x).
1
−1
Put q(x) = q1 (x) + (an bm )x n−m and r(x) = r1 (x).
Thus f (x) = q(x).g(x) + r(x), where r(x) = 0 or deg(r(x)) <deg(g(x)).
Hence the theorem is true for polynomials of degree n
and so the theorem is true for polynomials of any degree.
For uniqueness of q(x) and r(x),
0 0
suppose that we can find another pair of polynomials q (x), r (x)
0 0
with f (x) = q (x)g(x) + r (x) = q(x)g(x) + r(x),
0 0
where r(x) = 0 or deg(r(x)) <deg(g(x)), r (x) = 0 or deg(r (x)) <deg(g(x)).

Department of Mathematics/ College of Education/ Al-Qadisiyah University 88 of 124


Ring Theory Prof. Dr. Akeel Ramadan Mehdi Al-Yassiri

0 0
Thus r(x) − r (x) = (q (x) − q(x))g(x).
0
Assume that r(x) − r (x) 6= 0.
0
Thus q (x) − q(x) 6= 0 and
0 0
deg(r(x) − r (x)) = deg((q (x) − q(x))g(x))
0
= deg (q (x) − q(x))+ deg(g(x) (by Theorem 6.18).
0 0
Since q (x) − q(x) 6= 0, we have deg(q (x) − q(x)) ≥ 0.
0
Thus deg(r(x) − r (x)) ≥deg(g(x)).
0
Since deg(r(x)) <deg(g(x)) and deg(r (x)) <deg(g(x)),
0
we have deg(r(x) − r (x)) <deg(g(x)) and this is a contradiction.
0 0
Thus r(x) − r (x) = 0 and hence r(x) = r (x).
0 0 0
Since r(x) − r (x) = (q (x) − q(x))g(x), we have (q (x) − q(x))g(x) = 0.
0
Assume that q (x) − q(x) 6= 0.
0
By Theorem 6.18, deg((q (x) − q(x))g(x))
0
= deg (q (x) − q(x))+ deg(g(x)
0
and hence the polynomial (q (x) − q(x))g(x) has a degree.
Thus the zero polynomial 0 has a degree and this is a contradiction.
0 0
Thus q (x) − q(x) = 0 and hence q (x) = q(x).

Remark 6.20. The polynomials q(x) and r(x) in Theorem 6.19 are respectively
known as the quotient and remainder when dividing f (x) by g(x).

Example 6.21. In the ring Q[x], divide f (x) = 2x3 + 3x2 − x + 5 by


g(x) = x2 − x + 1.
Solution:
2x + 5
x2 − x + 1 2x3 + 3x2 − x + 5


− 2x3 + 2x2 − 2x
5x2 − 3x + 5
− 5x2 + 5x − 5
2x
Thus f (x) = q(x)g(x) + r(x), where
q(x) = 2x + 5 and r(x) = 2x.

Example 6.22. In the ring Q[x], divide f (x) = x3 + x2 − 1 by g(x) = x − 1.

Department of Mathematics/ College of Education/ Al-Qadisiyah University 89 of 124


Ring Theory Prof. Dr. Akeel Ramadan Mehdi Al-Yassiri

Solution:
x2 + 2x + 2
x3 + x2

x−1 −1
3
−x +x 2

2x2
− 2x2 + 2x
2x − 1
− 2x + 2
1

Thus f (x) = q(x)g(x) + r(x), where q(x) = x2 + 2x + 2 and r(x) = 1.

Example 6.23. In the ring Q[x], divide f (x) = x3 + x2 − x − 1 by g(x) = x + 1.


Solution:
x2 −1
3 2

x+1 x +x −x−1
− x3 − x2
−x−1
x+1
0

Thus f (x) = q(x)g(x) + r(x), where q(x) = x2 − 1 and r(x) = 0.

Exercise 93. Let (R, +, .) be a commutative ring with identity and let f (x) and g(x)
be two polynomials in R[x], with g(x) 6= 0. Prove that if (R, +, .) is a field or g(x) is
monic, then there exist unique polynomials q(x), r(x) ∈ R[x] such that
f (x) = q(x)g(x) + r(x) where either r(x) = 0 or deg(r(x)) <deg(g(x)).

Exercise 94. In the ring Z4 [x], divide f (x) = x3 + x2 + x + 1 by g(x) = 3x2 + x + 2.

Exercise 95. In the ring Q[x], divide f (x) = 2x3 + 5x2 + 2x + 7 by g(x) = x2 + 2x + 3.

Exercise 96. In the ring Z4 [x], divide f (x) = x2 + 3x + 1 by g(x) = 3x + 1.

Exercise 97. In the ring Z5 [x], divide f (x) = 4x3 + x2 + 1 by g(x) = 2x + 1.

Department of Mathematics/ College of Education/ Al-Qadisiyah University 90 of 124


Ring Theory Prof. Dr. Akeel Ramadan Mehdi Al-Yassiri

Definition 6.24. Let R be a ring and f (x), g(x) ∈ R[x] such that
f (x) = a0 + a1 x + . . . + an xn and g(x) = b0 + b1 x + . . . + bn xn .
We say that f (x) = g(x) if ai = bi for all i = 0, 1, . . . , n.

Definition 6.25. Let R be a ring and f (x), g(x) ∈ R[x] with g(x) 6= 0. We say that
g(x) divides f (x) (or g(x) is a factor of f (x)) if there is some polynomial h(x) ∈ R[x],
such that f (x) = g(x)h(x).

Remark 6.26. Let (R, +, .) be a commutative ring with identity and let f (x) and
g(x) be two polynomials in R[x], with g(x) 6= 0 and the leading coefficient of g(x)
an invertible element. By Theorem 6.19 (Division Algorithm for polynomials), there
exist unique polynomials q(x), r(x) ∈ R[x] such that f (x) = q(x)g(x) + r(x) where
either r(x) = 0 or deg(r(x)) <deg(g(x)).
If r(x) = 0, then f (x) = q(x)g(x) and hence g(x) divides f (x).

Example 6.27. In the ring Q[x], g(x) = 3x + 2 divides f (x) = 6x2 + x − 2.


Since
2x − 1
6x2 + x − 2

3x + 2
− 6x2 − 4x
− 3x − 2
3x + 2
0
we have f (x) = q(x)g(x), where q(x) = 2x − 1 and hence g(x) divides f (x).

Definition 6.28. Let R be a ring and f (x) ∈ R[x] such that f (x) = a0 + a1 x + a2 x2 +
. . . + an xn . If r ∈ R, then we define f (r) as follows:
f (r) = a0 + a1 r + a2 r2 . . . + an rn .

Definition 6.29. Let R be a ring and f (x) ∈ R[x]. An element r ∈ R is said to be a


root of f (x) (or a zero of f (x)) if f (r) = 0.

Examples 6.30. (1) If f (x) = x2 + 1 ∈ R[x], then f (x) has no root in R.


(2) If f (x) = x2 + 1 ∈ C[x], then f (x) has two roots in C, they are i and −i.
(3) If f (x) = x2 + 1 ∈ Z2 [x], then r = 1 is a root of f (x) in Z2 but r = 0 is not a root
of f (x) in Z2 .

Department of Mathematics/ College of Education/ Al-Qadisiyah University 91 of 124


Ring Theory Prof. Dr. Akeel Ramadan Mehdi Al-Yassiri

Theorem 6.31. (Remainder Theorem) Let (R, +, .) be a commutative ring with


identity. If f (x) ∈ R[x] and a ∈ R, then there is a unique polynomial q(x) ∈ R[x] such
that f (x) = (x − a)q(x) + f (a).
Proof. By applying the division algorithm for polynomials (Theorem 6.19) to
f (x) and x − a, we have there exist unique polynomial q(x), r(x) ∈ R[x] such that
f (x) = (x − a)q(x) + r(x), where either r(x) = 0 or deg(r(x)) <deg(x − a) = 1.
Thus either r(x) = 0 or deg(r(x)) = 0.
Hence r(x) is a constant polynomial and so r(x) = r, for some r ∈ R.
Since f (a) = (a − a)q(x) + r = r, we have f (x) = (x − a)q(x) + f (a).

Theorem 6.32. (Factorization Theorem) Let (R, +, .) be a commutative ring with


identity and let f (x) ∈ R[x] and a ∈ R. Then the polynomial f (x) is divisible by x − a
if and only if a is a root of f (x).
Proof. By remainder theorem (Theorem 6.31), there is
a unique polynomial q(x) ∈ R[x] such that f (x) = (x − a)q(x) + f (a).
Thus f (x) = (x − a)q(x) if and only if f (a) = 0.
Hence x − a divides f (x) if and only if a is a root of f (x).

Theorem 6.33. Let (R, +, .) be an integral domain and let f (x) ∈ R[x] be a nonzero
polynomial of degree n. Then f (x) has at most n distinct roots in R.
Proof. We will prove by induction on the degree n of f (x).
(1) If n = 0, then f (x) is a constant polynomial and hence f (x) has no root.
(2) If n = 1, then f (x) = a + bx, for some a, b ∈ R with b 6= 0.
If b is invertible in R, then −b−1 a is the only root of f (x).
If b is noninvertible in R, then f (x) has no root.
Thus f (x) has at most one root in R.
(3) Suppose that the theorem is true for all polynomials of degree n − 1.
We will prove the theorem is true for all polynomials of degree n.
Let f (x) be a polynomial of degree n.
Then either f (x) has no root or it has a root.
If f (x) has no root, then f (x) has zero of roots and hence
the number of roots less than n.
Now, if f (x) has a root, say r, then factorization theorem (Theorem 6.32) implies
that there is a unique polynomial q(x) ∈ R[x]
such that f (x) = (x − r)q(x).
Since f (x) of degree n and R is an integral domain, we have from Theorem 6.14 that
deg( f (x)) =deg(x − r)+deg(q(x)) = 1+deg(q(x)).
Thus deg(q(x)) = n − 1.

Department of Mathematics/ College of Education/ Al-Qadisiyah University 92 of 124


Ring Theory Prof. Dr. Akeel Ramadan Mehdi Al-Yassiri

By hypothesis, q(x) has at most n − 1 distinct roots in R.


Let r1 be another root of f (x), thus r1 6= r.
Hence f (r1 ) = (r1 − r)q(r1 ) = 0.
Since R is an integral domain and r1 − r, q(r1 ) ∈ R with r1 − r 6= 0, we have
q(r1 ) = 0 and hence r1 is a root of q(x).
Thus any root r1 of f (x) with r1 6= r is a root of q(x).
Since q(x) has at most n − 1 distinct roots in R, we have
f (x) has at most n distinct roots in R.

Remark 6.34. If (R, +, .) is not integral domain, then Theorem 6.33 is not true in
general, for example: let f (x) = 2x ∈ Z4 [x], thus deg( f (x)) = 1 but f (x) has two
roots 0, 2 in Z4 . Thus the number of distinct roots of f (x) in Z4 is not equal or less
than deg( f (x)).

Corollary 6.35. Let R be an integral domain and let f (x), g(x) ∈ R[x] be nonzero
polynomials of degrees less than or equal to n. If there exist n + 1 distinct elements
ak ∈ R (k = 1, 2, ..., n + 1) for which f (ak ) = g(ak ), then f (x) = g(x).
Proof. Assume that f (x) 6= g(x).
Thus f (x) − g(x) 6= 0.
Suppose that h(x) = f (x) − g(x), thus h(x) ∈ R[x]
and h(x) has a degree less than or equal to n.
By Theorem 6.33, h(x) has at most n distinct roots in R.
Since f (ak ) = g(ak ), for distinct elements ak ∈ R (k = 1, 2, ..., n + 1) (by hypothesis),
we have h(ak ) = 0, for all k = 1, 2, ..., n + 1.
Hence h(x) has n + 1 distinct roots in R and this is a contradiction.
Therefore, f (x) = g(x).

Lemma 6.36. If p is a prime number, then the group ((Z p )∗ , . p ) is a cyclic group of
order p − 1, where (Z p )∗ is the set of all multiplicative inverse elements in Z p .
Proof. Exercise.

Example 6.37. Let f (x) = x p −x ∈ Z p [x]. Then every element of the ring (Z p , + p , . p )
is a root of f (x).
p
Proof. Since f (0) = 0 − 0 = 0, we have 0 is a root of f (x).
By Lemma 6.36, the group ((Z p )∗ , . p ) is a cyclic group of order p − 1,
where (Z p )∗ is the set of all multiplicative inverse elements in Z p .

Department of Mathematics/ College of Education/ Al-Qadisiyah University 93 of 124


Ring Theory Prof. Dr. Akeel Ramadan Mehdi Al-Yassiri

Let 0 6= a ∈ Z p , thus a ∈ (Z p )∗ and hence (a) p−1 = 1.


So, (a) p = a.
Thus f (a) = (a) p − a = 0 and hence a is a root of f (x) in Z p [x].
Therefore, every element of the ring (Z p , + p , . p ) is a root of f (x).

Example 6.38. Let f (x) = x5 −x ∈ Z5 [x]. Then every element of the ring (Z5 , +5 , .5 )
is a root of f (x).

Theorem 6.39. (The Fundamental Theorem of Algebra) Let (C, +, .) be the field
of complex numbers. If f (x) ∈ C[x] is a polynomial of positive degree, then f (x) has
at least one root in C.

Corollary 6.40. If f (x) ∈ C[x] is a polynomial of degree n > 0, then f (x) can be
expressed in C[x] as a product of n (not necessarily distinct) linear factors.

Examples 6.41. (1) Let f (x) = x2 + 1 ∈ C[x]. Then deg( f (x)) = 2 and
f (x) = (x − i)(x + i).
(2) Let f (x) = x2 − 2ix − 1 ∈ C[x]. Then deg( f (x)) = 2 and f (x) = (x − i)(x − i).
(3) Let f (x) = x3 + (1 − i)x2 − (2 + i)x + 2i ∈ C[x]. Then deg( f (x)) = 3 and
f (x) = (x − 1)(x + 2)(x − i).

Theorem 6.42. If (F, +, .) is a field, then the ring (F[x], +, .) is a principal ideal
domain.
Proof. Let (F, +, .) be a field, thus (F, +, .) is an integral domain.
By Theorem 6.16, the ring (F[x], +, .) is an integral domain.
We will prove that every ideal in (F[x], +, .) is principal.
Let (I, +, .) be any ideal of F[x], thus either I = {0} or I 6= {0}.
If I = {0}, then I =< 0 > and hence it is a principal ideal.
If I 6= {0}, then there is some nonzero polynomial p(x) of lowest degree in I.
Now, we will prove that I =< p(x) >.
Let f (x) ∈ I.
By applying the division algorithm (Theorem 6.19) on f (x) and p(x),
we have that there exist unique polynomials q(x), r(x) ∈ F[x]
such that f (x) = q(x)p(x) + r(x) where either r(x) = 0 or deg(r(x)) <deg(p(x)).
Thus r(x) = f (x) − q(x)p(x) ∈ I.
If deg(r(x)) <deg(p(x)), then

Department of Mathematics/ College of Education/ Al-Qadisiyah University 94 of 124


Ring Theory Prof. Dr. Akeel Ramadan Mehdi Al-Yassiri

we have a contradiction to the lowest degree of p(x) in I.


Thus r(x) = 0 and hence f (x) = q(x)p(x) and this implies that f (x) ∈< p(x) >.
Thus I ⊆< p(x) >.
Since < p(x) >⊆ I, we have I =< p(x) > and hence I is a principal ideal.
Therefore, (F[x], +, .) is a principal ideal domain.

Exercise 98. Let (R, +, .) be a ring with identity. Prove that if (I, +, .) is an ideal of
the ring (R, +, .), then (I[x], +, .) is an ideal of the ring (R[x], +, .).

Exercise 99. Give an example of a ring (R, +, .) and a polynomial f (x) ∈ R[x] such
that f (x) has exactly 19 roots in R.

Exercise 100. Let (F, +, .) be a field. Prove that a nontrivial ideal of the ring (F[x], +, .)
is maximal if and only if it is a prime ideal.

Department of Mathematics/ College of Education/ Al-Qadisiyah University 95 of 124


Ring Theory Prof. Dr. Akeel Ramadan Mehdi Al-Yassiri

6.3 Irreducible Polynomials


Definition 6.43. Let (F, +, .) be a field. A nonconstant polynomial f (x) ∈ F[x] is
said to be irreducible polynomial in F[x] if f (x) cannot be expressed as the product of
two polynomials of positive degree. Otherwise, f (x) is called a reducible polynomial
in F[x].

Example 6.44. (1) If f (x) = x2 + 1 ∈ R[x], then f (x) is an irreducible polynomial in


R[x].
(2) If f (x) = x2 + 1 ∈ C[x], then f (x) = (x − i)(x + i) and hence f (x) is a reducible
polynomial in C[x].
(3) If f (x) = x2 + 1 ∈ Z2 [x], then f (x) = (x + 1)(x + 1) and hence f (x) is a reducible
polynomial in Z2 .

Example 6.45. Let f (x) = x3 + x + 1 ∈ Z2 [x].


3
Since f (0) = 0 +2 0 +2 1 = 1 6= 0, we have 0 is not a root of f (x).
3
Since f (1) = 1 +2 1 +2 1 = 1 6= 0, we have 1 is not a root of f (x).
Thus f (x) has no roots in Z2 .
Hence f (x) = x3 + x + 1 ∈ Z2 [x] is an irreducible polynomial in Z2 [x].

Example 6.46. Let (F, +, .) be a field. Then any linear polynomial f (x) = ax + b
with a 6= 0, is irreducible in F[x].
In other words, every reducible polynomial in F[x] has degree at least 2.
Proof. Assume that f (x) = ax + b is a reducible polynomial in F[x].
Thus there are two polynomials g(x), h(x) ∈ F[x] of positive degrees such that
f (x) = g(x).h(x) and hence deg(ax + b) = deg(g(x).h(x)).
By Theorem 6.14, deg(ax + b) = deg(g(x))+deg(h(x))
and hence deg(g(x))+deg(h(x)) = 1.
Since g(x), h(x) of positive degrees, we have deg(g(x))+deg(h(x)) ≥ 2
and hence 1 ≥ 2 and this is a contradiction.
Therefore, the linear polynomial f (x) = ax + b with a 6= 0, is irreducible in F[x].

Example 6.47. The polynomial f (x) = x2 − 2 is irreducible in Q[x].


Proof. Assume that f (x) = x2 − 2 is a reducible polynomial in Q[x].
Thus there are two polynomials g(x) = ax + b, h(x) = cx + d ∈ Q[x] such that
f (x) = g(x).h(x), where a, b, c, d ∈ Q with a 6= 0, c 6= 0.
Thus x2 − 2 = (ax + b).(cx + d) = (ac)x2 + (ad + bc)x + bd and hence

Department of Mathematics/ College of Education/ Al-Qadisiyah University 96 of 124


Ring Theory Prof. Dr. Akeel Ramadan Mehdi Al-Yassiri

ac = 1, bd = −2, ad + bc = 0.
Thus c = 1/a, d = −2/b.
By replacing c and d in the relation ad + bc = 0, we have
0 = (−2a/b) + (b/a) = (−2a2 + b2 )/a.b and hence
−2a2 + b2 = 0 and so 2a2 = b2 . √
Thus b2 /a2 = 2 and hence√ (b/a) 2 = 2 and this implies that b/a = 2.

Since b/a ∈ Q, we have 2 ∈ Q and this is a contradiction.


Therefore, the polynomial f (x) = x2 − 2 is irreducible in Q[x].

Theorem 6.48. If (F, +, .) is a field and 0 6= f (x) ∈ F[x], then the following state-
ments are equivalent:
(1) f (x) is an irreducible polynomial in F[x].
(2) The principal proper ideal (< f (x) >, +, .) is a maximal (prime) ideal of (F[x], +, .).
(3) The quotient ring (F[x]/ < f (x) >, +, .) is a field.
Proof. (1) ⇒ (2). Suppose that f (x) is an irreducible polynomial in F[x].
We will prove that (< f (x) >, +, .) is a maximal ideal of (F[x], +, .).
Let I be an ideal of F[x] such that < f (x) >$ I ⊆ F[x].
Since (F, +, .) is a field (by hypothesis), we have from Theorem 6.42 that
(F[x], +, .) is a principal ideal domain.
Hence there is g(x) ∈ F[x] such that I =< g(x) >.
Thus < f (x) >$< g(x) >⊆ F[x] and hence f (x) ∈< g(x) >.
So f (x) = h(x).g(x) for some h(x) ∈ F[x].
Since f (x) is an irreducible polynomial in F[x] (by hypothesis), we have
either h(x) or g(x) is of degree zero.
If h(x) is of degree zero, then h(x) = a for some nonzero a ∈ F
and hence f (x) = a.g(x).
Since (F, +, .) is a field, we have a is invertible and hence
g(x) = a−1 f (x).
Thus g(x) ∈< f (x) > and hence < g(x) >⊆< f (x) > and this is a contradiction.
Therefore, g(x) is of degree zero and hence g(x) = b, for some nonzero b ∈ F.
Thus g(x) is invertible in F[x] and hence < g(x) >= F[x].
Therefore, the principal ideal (< f (x) >, +, .) is a maximal ideal of (F[x], +, .).
(2) ⇒ (1). Suppose that the principal ideal (< f (x) >, +, .) is a maximal ideal of
(F[x], +, .), thus < f (x) >6= F[x].
Assume that f (x) is a reducible polynomial in F[x].
Thus there are two polynomials of positive degree h(x), g(x) ∈ F[x] such that
f (x) = h(x).g(x).
By Theorem 6.14, deg( f (x)) = deg(h(x))+ deg(g(x)).
Since deg(h(x)) ≥ 1 and deg(g(x)) ≥ 1, we have
deg( f (x)) > deg(g(x)) and deg( f (x)) > deg(h(x)).

Department of Mathematics/ College of Education/ Al-Qadisiyah University 97 of 124


Ring Theory Prof. Dr. Akeel Ramadan Mehdi Al-Yassiri

Since (< f (x) >, +, .) is a maximal ideal of (F[x], +, .), we have


(< f (x) >, +, .) is a prime ideal of (F[x], +, .)
Since f (x) = h(x).g(x), we have h(x).g(x) ∈< f (x) >.
Thus either h(x) ∈< f (x) > or g(x) ∈< f (x) >.
If h(x) ∈< f (x) >, then there is p(x) ∈ F[x] such that h(x) = p(x). f (x).
Hence deg(h(x)) ≥ deg( f (x)) and this is a contradiction.
If g(x) ∈< f (x) >, then there is q(x) ∈ F[x] such that g(x) = q(x). f (x).
Hence deg(g(x)) ≥ deg( f (x)) and this is a contradiction.
Therefore, f (x) is an irreducible polynomial in F[x].
(2) ⇔ (3) By Theorem 5.12.

Theorem 6.49. (Unique Factorization Theorem) Let (F, +, .) be a field. Then each
polynomial f (x) ∈ F[x] of positive degree is the product of a nonzero element of F
and irreducible monic polynomials in F[x]. Apart from the order of the factors, this
factorization is unique.
Proof. Exercise.

Example 6.50. Consider the polynomial f (x) = 9x2 + 1 ∈ C[x]. Then


f (x) = 9x2 + 1 = 9(x2 + 1/9)
= 9(x + (1/3)i)(x − (1/3)i).

Corollary 6.51. If f (x) ∈ R[x] is of positive degree, then f (x) can be factored into
linear and irreducible quadratic factors.
Proof. Exercise.

Exercise 101. Let (F, +, .) be a field and f (x) ∈ F[x] be a polynomial of degree 2 or
3. Prove that f (x) is reducible in F[x] if and only if f (x) has a root in F.

Department of Mathematics/ College of Education/ Al-Qadisiyah University 98 of 124


Ring Theory Prof. Dr. Akeel Ramadan Mehdi Al-Yassiri

7 Extension Fields
7.1 Extension Fields and Kronecker Theorem
0 0
Definition 7.1. Let (F, +, .) and (F , +, .) be two fields. A field (F , +, .) is said to
0
be an extension field of a field (F, +, .) if (F, +, .) is a subfield of (F , +, .).

Examples 7.2. (1) The field of real numbers (R, +, .) is an extension field of the
field of rational numbers (Q, +, .)
(2) The field of complex numbers (C, +, .) is an extension field of the fields (Q, +, .)
and (R, +, .).

(3) Let F = {a + b p | a, b ∈ Q}, where p is a prime number. Then (F, +, .) is an
extension field of the field (Q, +, .) and the field (R, +, .) is an extension field of the
field (F, +, .).

Theorem 7.3. (Kronecker Theorem) Let (F, +, .) be a field. If f (x) is an irredu-


cible polynomial in F[x], then there is an extension field of (F, +, .) in which f (x)
has a root.
Proof. Let f (x) be an irreducible polynomial in F[x].
Let I =< f (x) >,
that is (I, +, .) is a nontrivial principal ideal of (F[x], +, .) generated by f (x).
By Theorem 6.48, the quotient ring (F[x]/I, +, .) is a field.
We will prove that the field (F[x]/I, +, .) is a required field.
That is, we will prove that the field (F[x]/I, +, .) is
an extension field of (F, +, .) and it has a root of f (x).
Let natI : F[x] → F[x]/I be the natural ring epimorphism.
Define α : F → F[x]/I to be the restriction of natI on F.
That is α = natI ◦ iF : F → F[x]/I, where
iF : F → F[x] is the inclusion ring homomorphism.
By Proposition 4.13, either α = 0 or α is injective.
Since α(1) = (natI ◦ iF )(1) = natI (iF (1)) = natI (1) = 1 + I 6= I,
we have α 6= 0 and hence α is injective.
0 0
Define α : F → α(F) by α (a) = α(a), for all a ∈ F.
0
It is clear that α is a ring isomorphism (Why?) and hence (α(F), +, .) is a field.
Since α(F) ⊆ F[x]/I, we have (α(F), +, .) is a subfield of the field (F[x]/I, +, .).
Since F ∼ = α(F), we have (F, +, .) is a subfield of the field (F[x]/I, +, .) and hence
the field (F[x]/I, +, .) is an extension field of (F, +, .).
Now, we will prove that the field (F[x]/I, +, .) has a root of f (x).
Let f (x) = a0 + a1 x + . . . + an xn , where ai ∈ F.

Department of Mathematics/ College of Education/ Al-Qadisiyah University 99 of 124


Ring Theory Prof. Dr. Akeel Ramadan Mehdi Al-Yassiri

Let λ = x + I, thus λ ∈ F[x]/I.


Since f (λ ) ∈ F[x]/I, we have
f (λ ) = f (λ ) + I = (a0 + a1 λ + . . . + an λ n ) + I
= (a0 + I) + (a1 λ + I) + . . . + (an λ n + I)
= (a0 + I) + (a1 (x + I) + I) + . . . + (an (x + I)n + I)
= (a0 + I) + (a1 x + I) + . . . + (an xn + I) = (a0 + a1 x + . . . + an xn ) + I
= f (x) + I = I (since f (x) ∈ I =< f (x) >).
Therefore, λ is a root of f (x) in the field F[x]/I.

Corollary 7.4. Let (F, +, .) be a field. If the polynomial f (x) ∈ F[x] is of positive
degree, then there is an extension field of (F, +, .) in which f (x) has a root.
Proof. By Theorem 6.49, f (x) = a.q1 (x).q2 (x) . . . qm (x) for some positive integer m,
where 0 6= a ∈ F and qi (x) ∈ F[x] are irreducible monic.
By Theorem 7.3 (Kronecker Theorem), there is an extension field of (F, +, .) in
which q1 (x) has a root (say λ ).
Thus q1 (λ ) = 0 and hence f (λ ) = 0.
Thus there is an extension field of (F, +, .) in which f (x) has a root.

Theorem 7.5. Let (F, +, .) be a field and let f (x) ∈ F[x] be an irreducible polynomial
of positive degree n. If λ is a root of f (x) in the field (F[x]/ < f (x) >, +, .), then
F[x]/ < f (x) >= {b0 + b1 λ + . . . + bn−1 λ n−1 | bi ∈ F and f (λ ) = 0}.

Example 7.6. Let f (x) = x2 + 1 ∈ R[x]. By using Kronecker Theorem, find an ex-
tension field E of the field (R, +, .) such that f (x) has a root in E.
Solution: Since the root of f (x) = x2 + 1 are i, −i ∈ / R,
we have from Exercise 101 that f (x) is an irreducible in R[x].
By Theorem 7.3 (Kronecker Theorem), we have E = R[x]/ < f (x) > is an extension
field of R with E has a root of f (x), say λ .
By Theorem 7.5, we have E = {b0 + b1 λ | b0 , b1 ∈ R and λ 2 + 1 = 0}.
= {b0 + b1 λ | b0 , b1 ∈ R and λ 2 = −1}.
0 0
Thus (E, + , . ) is an extension field of (R, +, .) such that f (x) = x2 + 1 has a root in
0 0
E, where + and . are usual operations for polynomials.
That is, for all a0 + a1 λ , b0 + b1 λ ∈ E, we have:
0
(a0 + a1 λ ) + (b0 + b1 λ ) = (a0 + b0 ) + (a1 + b1 )λ and
0
(a0 + a1 λ ). (b0 + b1 λ ) = c0 + c1 λ + c2 λ 2 , where
c0 = a0 .b0 , c1 = a0 .b1 + a1 .b0 , c2 = a0 .b2 + a1 .b1 + a2 .b0 = a1 .b1 .
0
Hence (a0 + a1 λ ). (b0 + b1 λ ) = a0 .b0 + (a0 .b1 + a1 .b0 )λ + a1 .b1 λ 2
= a0 .b0 + (a0 .b1 + a1 .b0 )λ − a1 .b1 = (a0 .b0 − a1 .b1 ) + (a0 .b1 + a1 .b0 )λ .

Department of Mathematics/ College of Education/ Al-Qadisiyah University 100 of 124


Ring Theory Prof. Dr. Akeel Ramadan Mehdi Al-Yassiri

0 0
Define α : (E, + , . ) → (C, +, .) by α(a + bλ ) = a + bi, for all a + bλ ∈ E.
It is clear that α is a ring isomorphism (Exercise).
0 0
Thus (E, + , . ) ∼= (C, +, .).

Example 7.7. Let f (x) = x2 − 2 ∈ Q[x]. By using Kronecker Theorem, find an


extension field E of the field (Q, +, .) such that f (x) has a root in E.
√ √
Solution: Since the root of f (x) = x2 − 2 are 2, − 2 ∈ / Q,
we have from Exercise 101 that f (x) is an irreducible in Q[x].
By Theorem 7.3 (Kronecker Theorem), we have E = Q[x]/ < f (x) > is an extension
field of Q with E has a root of f (x), say λ .
By Theorem 7.5, we have E = {b0√ + b1 λ | b0 , b1 ∈ Q and λ 2 − 2 = 0}.
= {b0 + b1 √ λ | b0 , b1 ∈ Q and λ = 2}
= {b0 + b1 2 | b0 , b1 ∈ Q}.
0 0
Thus (E, + , . ) is an extension field of (Q, +, .) such that f (x) = x2 − 2 has a root in
0 0
E, where + and . are usual √ operations √ for polynomials.
√ all 0(a0 + a1 √2), (b0 + b1 2) ∈ E, we have:
That is, for √
(a0 + a1 √2) + (b0 + b√1 2) = (a0 +√b0 ) + (a1√+ b1 ) 2 and
0
(a0 + a1 2). (b0 + b1 2) = c0 + c1 2 + c2 ( 2)2 , where
c0 = a0 .b0 , c1 √= a0 .b1 + a1 .b√0 , c2 = a0 .b2 + a1 .b1 + a2 .b0 √
= a1 .b1 .
0
Hence (a0 + a1 2). (b0 + b1 2) = a√0 .b0 + (a0 .b1 + a1 .b0 ) 2 + 2a1 .b1
= (a0 .b0 + 2a1 .b1 ) + (a0 .b1 + a1 .b0 ) 2.

Example 7.8. Let f (x) = x2 + x + 1 ∈ Z2 [x]. By using Kronecker Theorem, find an


extension field E of the field (Z2 , +, .) such that f (x) has a root in E.
Solution: Since f (0) = 1 and f (1) = 1, we have f (x) has no root in Z2 .
By Exercise 101, we have f (x) is an irreducible in Z2 [x].
By Theorem 7.3 (Kronecker Theorem), we have E = Z2 [x]/ < f (x) > is an extension
field of Z2 with E has a root of f (x), say λ .
By Theorem 7.5, we have E = {b0 + b1 λ | b0 , b1 ∈ Z2 and λ 2 + λ + 1 = 0}.
= {b0 + b1 λ | b0 , b1 ∈ Z2 and λ 2 = −λ − 1}
= {0, 1, λ , 1 + λ | λ 2 = 1 + λ } and λ , 1 + λ are the roots of f (x) in E.
Thus (E, +, .) is an extension field of (Z2 , +2 , .2 ) such that f (x) = x2 + x + 1 has a
root in E, where + and . are usual operations for polynomials.
That is, (+) and (.) are defined as the following two tables:

+ 0 1 λ 1+λ . 0 1 λ 1+λ
0 0 1 λ 1+λ 0 0 0 0 0
1 1 0 1+λ λ 1 0 1 λ 1+λ
λ λ 1+λ 0 1 λ 0 λ 1+λ 1
1+λ 1+λ λ 1 0 1+λ 0 1+λ 1 λ

Department of Mathematics/ College of Education/ Al-Qadisiyah University 101 of 124


Ring Theory Prof. Dr. Akeel Ramadan Mehdi Al-Yassiri

Definition 7.9. A field (F, +, .) is said to be Galois field and denoted by GF(pn ) if
F is a finite field contains pn elements, where p is a prime number and n ∈ Z+ .

Example 7.10. An extension field E = {0, 1, λ , 1 + λ | λ 2 = 1 + λ } of the field


(Z2 , +, .) in the Example 7.13 is a Galois field (GF(22 ))

Lemma 7.11. Let p be a prime number and let f (x) ∈ Z p [x] be an irreducible po-
lynomial of positive degree n. Then the finite field Z p [x]/ < f (x) >= {b0 + b1 λ +
. . . + bn−1 λ n−1 | bi ∈ Z p and f (λ ) = 0} is a Galois field with pn elements.

Theorem 7.12. Let p be a prime number. Then any two fields having pn elements
are isomorphic.

Example 7.13. Give an example of a field contains 8 elements (or find GF(23 )).
Solution: Let f (x) = x3 + x + 1 ∈ Z2 [x].
Since f (0) = 1 and f (1) = 1, we have f (x) has no root in Z2 .
By Exercise 101, we have f (x) is an irreducible in Z2 [x].
By Theorem 7.3 (Kronecker Theorem), we have E = Z2 [x]/ < f (x) > is an extension
field of Z2 with E has a root of f (x), say λ .
By Theorem 7.5, we have
E = {b0 + b1 λ + b2 λ 2 | b0 , b1 , b2 ∈ Z2 and λ 3 + λ + 1 = 0}.
= {b0 + b1 λ + b2 λ 2 | b0 , b1 , b2 ∈ Z2 and λ 3 = −λ − 1}
= {0, 1, λ , λ 2 , 1 + λ , λ + λ 2 , 1 + λ 2 , 1 + λ + λ 2 | λ 3 = 1 + λ },
and λ , λ 2 , λ + λ 2 are the roots of f (x) in E.
Thus (E, +, .) is an extension field of (Z2 , +2 , .2 ),
where + and . are usual operations for polynomials.
That is, (+) and (.) are defined as the following two tables:

+ 0 1 λ 1+λ λ2 1+λ2 λ +λ2 1+λ +λ2


0 0 1 λ 1+λ λ2 1+λ2 λ +λ2 1+λ +λ2
1 1 0 1+λ λ 1+λ2 λ2 1+λ +λ2 λ +λ2
λ λ 1+λ 0 1 λ +λ2 1+λ +λ2 λ2 1+λ2
1+λ 1+λ λ 1 0 1+λ +λ2 λ +λ2 1+λ2 λ2
λ2 λ2 1+λ2 λ +λ2 1+λ +λ2 0 1 λ 1+λ
1+λ2 1+λ2 λ2 1+λ +λ2 λ +λ2 1 0 1+λ λ
λ +λ2 λ +λ2 1+λ +λ2 λ2 1+λ2 λ 1+λ 0 1
1+λ +λ2 1+λ +λ2 λ +λ2 1+λ2 λ2 1+λ λ 1 0

Department of Mathematics/ College of Education/ Al-Qadisiyah University 102 of 124


Ring Theory Prof. Dr. Akeel Ramadan Mehdi Al-Yassiri

. 0 1 λ 1+λ λ2 1+λ2 λ +λ2 1+λ +λ2


0 0 0 0 0 0 0 0 0
1 0 1 λ 1+λ λ2 1+λ2 λ +λ2 1+λ +λ2
λ 0 λ λ2 λ +λ2 1+λ 1 1+λ +λ2 1+λ2
1+λ 0 1+λ λ +λ2 1+λ2 1+λ +λ2 λ2 1 λ
λ2 0 λ2 1+λ 1+λ +λ2 λ +λ2 λ 1+λ2 1
1+λ2 0 1+λ2 1 λ2 λ 1+λ +λ2 1+λ λ +λ2
λ +λ2 0 λ +λ2 1+λ +λ2 1 1+λ2 1+λ λ λ2
1+λ +λ2 0 1+λ +λ2 1+λ2 λ 1 λ +λ2 λ2 1+λ

Department of Mathematics/ College of Education/ Al-Qadisiyah University 103 of 124


Ring Theory Prof. Dr. Akeel Ramadan Mehdi Al-Yassiri

7.2 Splitting Extension Fields


Definition 7.14. Let (F, +, .) be a field and let f (x) ∈ F[x] of degree n. An extension
field (E, +, .) of a field (F, +, .) is said to be a splitting field (or splitting extension
field) for f (x) over F if the following two conditions are hold:
(1) f (x) can be factored completely into linear factors in E[x].
That is: there are λ , λ1 , λ2 , . . . , λn ∈ E such that
f (x) = λ (x − λ1 )(x − λ2 ) . . . (x − λn ).
(2) There is no field (K, +, .) with F ⊆ K $ E such that f (x) can be factored com-
pletely into linear factors in K[x].
That is: (E, +, .) is the smallest extension field of (F, +, .) such that f (x) can be
factored completely into linear factors in E[x].

Theorem 7.15. Let (F, +, .) be a field. If f (x) ∈ F[x] is a polynomial of positive


degree, then there exists an extension field (E, +, .) of a field (F, +, .) in which f (x)
factors completely into linear polynomials.
That is: there exists an extension field (E, +, .) of a field (F, +, .) contains all roots
of f (x).
Proof. We will prove by induction on the degree of f (x).
Let n =deg( f (x)).
If n = 1, then f (x) = a + bx with a, b ∈ F and b 6= 0.
Thus F is itself the required field.
Assume that n > 1 and that the theorem is true
for all polynomials of degree less than n.
By Theorem 6.49 (Unique Factorization Theorem), f (x) must have
an irreducible polynomial factor, say g(x) ∈ F[x].
By Theorem 7.3(Kronecker Theorem), there is an extension field
(F1 , +, .) (F1 = F[x]/ < g(x) >) of (F, +, .) in which g(x) has a root λ1 .
Thus λ1 ∈ F1 is a root of f (x)
By Theorem 6.32 (Factorization Theorem), there is
f1 (x) ∈ F1 [x] such that f (x) = (x − λ1 ) f1 (x).
Since deg( f1 (x)) = n − 1, there exists, by our induction hypothesis,
an extension field (E, +, .) of a field (F1 , +, .) in which
f1 (x) = λ (x − λ2 )(x − λ3 ) . . . (x − λn ), with λ , λ2 , λ3 , . . . , λn ∈ E, λ 6= 0.
Thus f (x) = λ (x − λ1 )(x − λ2 ) . . . (x − λn ), with λ , λ1 , λ2 , . . . , λn ∈ E, λ 6= 0.
Therefore, there exists an extension field (E, +, .) of a field (F, +, .) in which
f (x) factors completely into linear polynomials.
Remark 7.16. We can find splitting extension field for any polynomial f (x) ∈ F[x]
of positive degree, where (F, +, .) is a filed, by applying Kronecker Theorem one or
two times or more.

Department of Mathematics/ College of Education/ Al-Qadisiyah University 104 of 124


Ring Theory Prof. Dr. Akeel Ramadan Mehdi Al-Yassiri

Example 7.17. Let f (x) = x2 − 2 ∈ Q[x]. Find the splitting extension field for f (x)
over Q
Solution: The solution is similar to√Example √ 7.7 as following:
2
Since the root of f (x) = x − 2 are 2, − 2 ∈ / Q,
we have from Exercise 101 that f (x) is an irreducible in Q[x].
By Theorem 7.3 (Kronecker Theorem), we have E = Q[x]/ < f (x) > is an extension
field of Q with E has a root of f (x), say λ .
By Theorem 7.5, we have E = {b0√ + b1 λ | b0 , b1 ∈ Q and λ 2 − 2 = 0}.
= {b0 + b1 √ λ | b0 , b1 ∈ Q and λ = 2}
= {b0 + b1 2 | b0 , b1 ∈ Q}. √ √
Since the roots of the polynomial f (x) = x2 − 2 are 2, − 2 ∈ E, we have
0 0
(E, + , . ) is the splitting extension field for f (x) over Q,
0 0
where + and . are usual √ operations for √ polynomials.
√ all 0(a0 + a1 √2), (b0 + b1 2) ∈ E, we have:
That is, for √
(a0 + a1 √2) + (b0 + b√1 2) = (a0 +√b0 ) + (a1√+ b1 ) 2 and
0
(a0 + a1 2). (b0 + b1 2) = c0 + c1 2 + c2 ( 2)2 , where
c0 = a0 .b0 , c1 √ = a0 .b1 + a1 .b√0 , c2 = a0 .b2 + a1 .b1 + a2 .b0 √
= a1 .b1 .
0
Hence (a0 + a1 2). (b0 + b1 2) = a√0 .b0 + (a0 .b1 + a1 .b0 ) 2 + 2a1 .b1
= (a0 .b0 + 2a1 .b1 ) + (a0 .b1 + a1 .b0 ) 2.

Example 7.18. Let f (x) = x2 + x + 1 ∈ Z2 [x]. Find the splitting extension field for
f (x) over Z2
Solution: The solution is similar to Example 7.7 as following:
Since f (0) = 1 and f (1) = 1, we have f (x) has no root in Z2 .
By Exercise 101, we have f (x) is an irreducible in Z2 [x].
By Theorem 7.3 (Kronecker Theorem), we have E = Z2 [x]/ < f (x) > is an extension
field of Z2 with E has a root of f (x), say λ .
By Theorem 7.5, we have E = {b0 + b1 λ | b0 , b1 ∈ Z2 and λ 2 + λ + 1 = 0}.
= {b0 + b1 λ | b0 , b1 ∈ Z2 and λ 2 = −λ − 1}
= {0, 1, λ , 1 + λ | λ 2 = 1 + λ }.
Since the roots of the polynomial f (x) = x2 + x + 1 are λ , 1 + λ ∈ E, we have
0 0
(E, + , . ) is the splitting extension field for f (x) over Z2 ,
where + and . are usual operations for polynomials.
That is, (+) and (.) are defined as the following two tables:

+ 0 1 λ 1+λ . 0 1 λ 1+λ
0 0 1 λ 1+λ 0 0 0 0 0
1 1 0 1+λ λ 1 0 1 λ 1+λ
λ λ 1+λ 0 1 λ 0 λ 1+λ 1
1+λ 1+λ λ 1 0 1+λ 0 1+λ 1 λ

Department of Mathematics/ College of Education/ Al-Qadisiyah University 105 of 124


Ring Theory Prof. Dr. Akeel Ramadan Mehdi Al-Yassiri

Example 7.19. Let f (x) = x4 − 5x2 + 6 = (x2 − 2)(x2 − 3) ∈ Q[x]. Find the splitting
extension field for f (x) over Q
√ √
Solution: Since the root of x2 − 2 are 2, − 2 ∈ / Q,
2
we have from Exercise 101 that x − 2 is an irreducible in Q[x].
By Theorem 7.3 (Kronecker Theorem), we have F1 = Q[x]/ < x2 − 2 > is an exten-
sion field of Q with F1 has a root of x2 − 2, say λ .
By Theorem 7.5, we have F1 = {b0√+ b1 λ | b0 , b1 ∈ Q and λ 2 − 2 = 0}.
= {b0 + b1 √ λ | b0 , b1 ∈ Q and λ = 2}
= {b0 + b1 2 | b0 , b1 ∈ Q}. √ √
Since the roots of the polynomial x 2 − 2 are 2, − 2 ∈ F , we have the factorization
√ √ 1
2
f (x) = (x − 2)(x + 2)(x − 3) in F1 [x].
Now, we will prove that the polynomial x2 − 3 ∈ Q[x] is an irreducible in F1 [x].
Assume that the polynomial x2 − 3 ∈ Q[x] is a reducible √ in F1 [x].
2
By Exercise√101, x − 3 has a root in F1 , say a + b 2, where a, b ∈ Q.
2 2 2

Thus (a + b 2) − 3 = 0 and hence a + 2b − 3 + 2ab 2 = 0.
So, a2 + 2b2 − 3 = 0 and 2ab = 0.
Since a, b ∈ Q and (Q, +, .) is an integral p domain, we have either a = 0 or b = 0.
2
If a = 0, then b = 3/2 and hence b √ = 3/2 ∈ Q and this is a contradiction.
2
If b = 0, then a = 3 and hence a = 3 ∈ Q and this is a contradiction.
Thus the polynomial x2 − 3 is an irreducible polynomial in F1 [x].
Hence f (x) does not factor completely into linear polynomials in F1 [x].
By applying Theorem 7.3 (Kronecker Theorem) again on the irreducible polynomial
x2 − 3 ∈ F1 [x], we have E = F1 [x]/ < x2 − 3 > is an extension field of F1 and hence
E is an extension field of Q with E has a root of x2 − 3, say α.
Thus E =√ {c0 + c1 α | c√ 0 , c1√∈ F1 and α 2 − 3 = 0}.
= {(a + b√ 2) +√ (c + d √2) 3 | a, b, c, d ∈ Q}
= {a + b 2 + c 3 + d 6 | a, b, c, d ∈ Q}
Since the
√ √ roots
√ of√ the polynomial f (x) = x4 − 5x2 + 6 = (x2 − 2)(x2 − 3) are
2, − 2, 3, − 3 ∈ E, we have
0 0
(E, + , . ) is the splitting extension field for f (x) over Q,
0 0
where + and . are usual √operations √ for √ polynomials. √ √ √
That is, for all √ (a1 + b1√ 2 + c1 √3 + d1 6), (a√ 2 + b2 √2 + c2 3√+ d2 6) ∈ E, we
0
have: (a1√+ b1 2 + c1 √3 + d1 6) + √ (a2 + b2 2 + c2 3 + d2 6) = (a1 + a2 ) +
(b1 + b2 )√ 2 + (c√1 + c2 ) √3 + (d1 + d2 )√ 6 and √ √
0
(a1 + b1 2 + c1 3 + d1 6). (a2 + b2 2 + c2 3 + √ 2 6)
d √
= (a1 a2 + 2b1 b2 + 3c1 c2 + 6d√ 1 d2 ) + (b1 a2 + a1 b2 ) 2 + (a1 c2 + c1 a2 ) 3
+(a1 d2 + d1 a2 + b1 c2 + c1 b2 ) 6.

Exercise 102. Let f (x) = x4 −(p+q)x2 + pq = (x2 − p)(x2 −q) ∈ Q[x], where p, q ≥
2 are distinct prime numbers. Find the splitting extension field for f (x) over Q

Department of Mathematics/ College of Education/ Al-Qadisiyah University 106 of 124


Ring Theory Prof. Dr. Akeel Ramadan Mehdi Al-Yassiri

8 Boolean Rings
8.1 Definition, Examples and Some Properties of Boolean Rings
Definition 8.1. A ring R with identity is said to be Boolean ring if every element in
R is idempotent, that is a = a2 , for all a ∈ R.

Examples 8.2. (1) The ring (Z2 , +2 , .2 ) is a Boolean ring,


since 0.2 0 = 0 and 1.2 1 = 1.
(2) The ring ({0, 3}, +6 , .6 ) is a Boolean ring, since 3 it’s identity and 0.6 0 = 0 and
3.6 3 = 3.
(3) Let X be a nonempty set and P(X) the power set of X,
that is, P(X) = {A | A ⊆ X}. Define A4B = (A − B) (B − A) for all subsets A
S

and B of X. Then the ring (P(X), 4, ) is a Boolean ring, since X it’s identity and
T

A ∩ A = A for every A ∈ P(X).


(4) The ring (Z6 , +6 , .6 ) is not a Boolean ring, since for example 2.6 2 = 4 6= 2.
(5) The ring (Z, +, .) is not a Boolean ring, since for example 2.2 = 4 6= 2.

Theorem 8.3. Every Boolean ring (R, +, .) is commutative of characteristic 2.


Proof. See Theorem 1.24.
Let a ∈ R. Then a + a = (a + a)2 = (a + a)(a + a)
= a2 + a2 + a2 + a2 = a + a + a + a and hence a + a = 0.
That is 2a = 0 and hence the characteristic of R is 2.
Let a, b ∈ R. Then
a + b = (a + b)2 = (a + b)(a + b) = a2 + ab + ba + b2 = a + ab + ba + b.
Hence ab + ba = 0 = ab + ab (from above).
By cancellation law of addition, we have ab = ba.
Therefore, R is a commutative ring.

Theorem 8.4. Let R be a Boolean ring. Then:


(1) xn = x for any positive integer n.
(2) 0 is the only nilpotent element in R.
Proof. (1) We will prove by induction on n.
By hypothesis, xn = x is true for n = 1, 2.
Suppose that xk = x for some k ≥ 2 (induction hypothesis).
Then we have xk+1 = x.xk = x.x = x2 = x.
Thus, we conclude that xn = x for any positive integer n.

Department of Mathematics/ College of Education/ Al-Qadisiyah University 107 of 124


Ring Theory Prof. Dr. Akeel Ramadan Mehdi Al-Yassiri

(2) Assume that R has a nonzero nilpotent element, say x.


Thus there is a positive integer n such that xn = 0.
It follows from part (1) we have that x = xn = 0 and this is a contradiction.
Thus the only nilpotent element in R is 0.

Theorem 8.5. Let R be a Boolean ring. A proper ideal I of R is prime if and only if
I is a maximal ideal.
Proof. (⇒) Suppose that I is a prime ideal in R.
Let J be an ideal of R such that I $ J ⊆ R.
We will prove that J = R.
Since I $ J, there is an element a ∈ J with a ∈
/ I.
2
Thus a(1 − a) = a − a = a − a = 0 ∈ I.
Hence either a ∈ I or 1 − a ∈ I.
Since a ∈
/ I, we have 1 − a ∈ I $ J.
Since a ∈ J, we have 1 ∈ J and hence J = R.
Therefore, I is a maximal ideal of R.
(⇐) Suppose that I is a maximal ideal in R.
By Theorem 8.3, R is a commutative ring.
By Theorem 5.48, I is a prime ideal in R.

Theorem 8.6. A Boolean ring (R, +, .) is a field if and only if (R, +, .) ∼


= (Z2 , +2 , .2 ).
Proof. Let (R, +, .) be a Boolean ring.
(⇒) Suppose that (R, +, .) is a field.
Let 0 6= a ∈ R, thus we have:
a = a.1 = a.(a.a−1 ) = a2 .a−1 = a.a−1 = 1.
Hence R = {0, 1}.
Since any two-element field is isomorphic to (Z2 , +2 , .2 ), we have (R, +, .) ∼
= (Z2 , +2 , .2 ).

(⇐) Suppose that (R, +, .) = (Z2 , +2 , .2 ).
By Corollary 4.11, (Z2 , +2 , .2 ) is a field and hence (R, +, .) is a field.

Lemma 8.7. Let (R, +, .) be a Boolean ring. For each nonzero element a ∈ R, there
exists a ring epimorphism f : (R, +, .) → (Z2 , +2 , .2 ) such that f (a) = 1.
Proof. Exercise.

Department of Mathematics/ College of Education/ Al-Qadisiyah University 108 of 124


Ring Theory Prof. Dr. Akeel Ramadan Mehdi Al-Yassiri

Corollary 8.8. Every Boolean ring (R, +, .) is a semisimple ring.


Proof. We will prove that Rad(R) =< 0 >.
Assume that Rad(R) 6=< 0 >.
Thus there is a ∈ Rad(R) with a 6= 0.
By Lemma 8.7, there exists a ring epimorphism
f : (R, +, .) → (Z2 , +2 , .2 ) such that f (a) = 1.
Hence (ker( f ), +, .) is proper ideal of a ring (R, +, .).
By Theorem 8.3, (R, +, .) is a commutative ring.
By Theorem 5.21 (Krull-Zorn), we have
(ker( f ), +, .) is contained in a maximal ideal (I, +, .) of a ring (R, +, .).
Since f is a ring epimorphism (by hypothesis), we have
from Theorem 3.14(3) that f (1) = 1.
Since f (1 − a) = f (1) −2 f (a) = 1 −2 1 = 0, we have 1 − a ∈ker( f ) ⊆ I.
Since a ∈ Rad(R) ⊆ I, we have 1 = a + (1 − a) ∈ I.
Hence I = R and this is a contradiction.
Therefore, Rad(R) =< 0 > and hence (R, +, .) is a semisimple ring.

Exercise 103. Let R be a Boolean ring. Prove that the zero ideal (< 0 >, +, .) is the
only nil ideal in R.

Exercise 104. Let (I, +, .) be a proper ideal of a ring (R, +, .). Prove that if (R, +, .)
is a Boolean ring, then the quotient ring (R/I, +, .) is a Boolean ring.

Exercise 105. Prove that a proper ideal (I, +, .) of the Boolean ring (R, +, .) is maxi-
mal ideal if and only if (R/I, +, .) ∼
= (Z2 , +2 , .2 ).

Department of Mathematics/ College of Education/ Al-Qadisiyah University 109 of 124


Ring Theory Prof. Dr. Akeel Ramadan Mehdi Al-Yassiri

9 Basic Concepts in Module Theory


9.1 Definition and Examples of Modules
Definition 9.1. Let R be a ring. A left R-module or a left module over R is a set M
together with
(1) a binary operation + on M under which M is an abelian group, and
(2) a mapping R × M → M (is called a module multiplication) denoted by rm, for all
r ∈ R and for all m ∈ M which satisfies
(a) (r + s)m = rm + sm, for all r, s ∈ R, m ∈ M,
(b) (rs)m = r(sm), for all r, s ∈ R, m ∈ M, and
(c) r(m + n) = rm + rn, for all r ∈ R, m, n ∈ M.
If the ring R has an identity element 1R and
(d) 1R .m = m, for all m ∈ M, then M is said to be a unitary left R-module.

Remark 9.2. The descriptor "left" in the above definition indicates that the ring ele-
ments appear on the left. A right R-modules can be defined analogously as follows.

Definition 9.3. Let R be a ring. A right R-module or a right module over R is a set
M together with
(1) a binary operation + on M under which M is an abelian group, and
(2) a mapping M × R → M (is called a module multiplication) denoted by mr, for all
r ∈ R and for all m ∈ M which satisfies
(a) m(r + s) = mr + ms, for all r, s ∈ R, m ∈ M,
(b) m(rs) = (mr)s, for all r, s ∈ R, m ∈ M, and
(c) (m + n)r = mr + nr, for all r ∈ R, m, n ∈ M.
If the ring R has an identity element 1R and
(d) m.1R = m, for all m ∈ M, then M is said to be a unitary left R-module.

The notation R M (resp. MR ) denotes to left (resp. right) R-module M.

Exercise 106. If the ring R is commutative, then a module M is left R-module if and
only if it is a right R-module.

Lemma 9.4. Let R be a ring with 1, let M be a left R-module and let r, s ∈ R, m, n ∈
M. Then:
(1) r 0M = 0M ;
(2) 0R m = 0M ;

Department of Mathematics/ College of Education/ Al-Qadisiyah University 110 of 124


Ring Theory Prof. Dr. Akeel Ramadan Mehdi Al-Yassiri

(3) (−1)m = −m;


(4) −(r m) = (−r)m = r(−m);
(5) (r − s)m = rm − sm;
(6) r(m − n) = rm − rn.
Proof. (1) r 0M = r(0M + 0M ) = r 0M + r 0M .
Thus r 0M + (−(r 0M )) = r 0M + r 0M + (−(r 0M )).
Hence 0M = r 0M .
(2) 0R m = (0R + 0R )m = 0R m + 0R m.
Thus 0R m + (−(0R m)) = 0R m + 0R m + (−(0R m)).
Hence 0M = 0R m.
(3) 0M = 0R m (by (2))
= (1 + (−1))m = m + (−1)m.
Thus (−1)m = −m.
(4) −(r m) = (−1)(r m) (by (3))
= ((−1)r)m = (−r)m.
Also, r(−m) = r((−1)m) (by (3))
= (r(−1))m = (−r)m.
Thus −(r m) = (−r)m = r(−m).
(5) (r − s)m = (r + (−s))m
= r m + (−s)m = r m + (−(s m)) (by (4))
= r m − s m.
(6) r(m − n) = r(m + (−n))
= r m + r(−n) = r m + (−(rn)) (by (4))
= r m − r n.

Example 9.5. Modules over a field F and vector spaces over F are the same.
Proof. By the definitions of modules and vector spaces over a field F. (Exercise)

Example 9.6. Every left ideal (I, +, .) of a ring (R, +, .) is a left R-module.
Proof. Let (I, +, .) be a left ideal of a ring (R, +, .).
Thus (I, +) is an abelian group (why?).
Define • : R × I → I by •(r, a) = r.a, for all r ∈ R and for all a ∈ I.
Since (R, +, .) is a ring, we have (r + s)a = ra + sa, (rs)a = r(sa) and
r(a + b) = ra + rb for all r, s ∈ R, a, b ∈ I.

Exercise 107. Every right ideal (I, +, .) of a ring (R, +, .) is a right R-module.

Department of Mathematics/ College of Education/ Al-Qadisiyah University 111 of 124


Ring Theory Prof. Dr. Akeel Ramadan Mehdi Al-Yassiri

Example 9.7. Every ring (R, +, .) is a left and right R-module.


Proof. Since (R, +, .) is an ideal of (R, +, .) it follows that
(R, +, .) is a left and right ideal of (R, +, .).
By Example 9.6, R is a left R-module.
By Exercise 107, R is a right R-module.
Thus (R, +, .) is a left and right R-module.

Example 9.8. Every abelian group is Z-module.


Proof. Let M be any abelian group, let a ∈ M and let n ∈ Z.
Define a multiplication na : Z × M → M as follows:
If n > 0, then na = a + a + · · · + a (n times);
if n = 0, then na = 0;
if n < 0, then na = −((−n)a) = −a − a − · · · − a (| n | times).
Let n, m ∈ Z and let a ∈ M, thus
(n + m)a = a + a + · · · + a (n + m times)
= (a + a + · · · + a) + (a + a + · · · + a) = na + ma.
| {z } | {z }
n times m times
Also, (nm)a = a + a + · · · + a (nm times)
= (a + a + · · · + a) + (a + a + · · · + a) + · · · + (a + a + · · · + a)
| {z } | {z } | {z }
m times m times m times
| {z }
n times
= ma + · · · + ma} = n(ma).
| + ma{z
n times
Let n ∈ Z and let a, b ∈ M, thus
n(a + b) = (a + b) + (a + b) + · · · + (a + b) (n times)
=a| +a+ {z· · · + b} = na + nb.
{z· · · + a} + |b + b +
n times n times
Thus the multiplication na : Z × M → M is a module multiplication.
Hence M is a left Z-module.
Since (Z, +, .) is a commutative ring, we have
from Exercise 106 that M is a Z-module.
Since 1Z .a = a for all a ∈ M it follows that M is a unitary Z-module.

Exercise 108. Let R be a ring with 1 and let n ∈ Z+ .


Define Rn = {(a1 , a2 , · · · , an ) | ai ∈ R, for all i}. Prove that Rn is a left R-module by
defining componentwise addition and multiplication by elements of R as follows:
(a1 , a2 , · · · , an ) + (b1 , b2 , · · · , bn ) = (a1 + b1 , a2 + b2 , · · · , an + bn ) and
r(a1 , · · · , an ) = (ra1 , · · · , ran ), for all r ∈ R and (a1 , a2 , · · · , an ), (b1 , b2 , · · · , bn ) ∈ Rn .

Department of Mathematics/ College of Education/ Al-Qadisiyah University 112 of 124


Ring Theory Prof. Dr. Akeel Ramadan Mehdi Al-Yassiri

Exercise 109. If Mn (R) is the set of n × n matrices over a ring R, then Mn (R) is an
additive abelian group under matrix addition.
(1) Prove that Mn (R) is a left R-module by defining a module multiplication on
Mn (R) as follows: for all (ai j ) ∈ Mn (R) and r ∈ R, then r(ai j ) = (rai j ).
(2) Prove that Mn (R) is a right R-module by defining a module multiplication on
Mn (R) as follows: for all (ai j ) ∈ Mn (R) and r ∈ R, then (ai j )r = (ai j r).

Department of Mathematics/ College of Education/ Al-Qadisiyah University 113 of 124


Ring Theory Prof. Dr. Akeel Ramadan Mehdi Al-Yassiri

9.2 Submodules
Definition 9.9. Let R be a ring and let M be a left R-module. A non-empty subset
N of M is said to be a left R-submodule of M if N is a left R-module with the same
addition and module multiplication on M. We will use N ,→ M to denote that N is a
submodule of M.

Proposition 9.10. Let R be a ring, let M be a left R-module and let N be a nonempty
subset of M. Then the following statements are equivalent:
(1) N is a left submodule of M;
(2) For all a, b ∈ N and for all r ∈ R we have that:
(i) a − b ∈ N and
(ii) r • a ∈ N, where • is the module multiplication defined on M;
(3) r • a − s • b ∈ N, for all a, b ∈ N and for all r, s ∈ R.
Proof. Exercise.

Example 9.11. Every left R-module M contains at least two submodules (trivial
submodules): M ,→ M and 0 ,→ M.

Example 9.12. Let R be a ring with 1R . Then the left submodules of R as a left
R-module are exactly the left ideals of a ring (R, +, .).
Proof. Let (I, +, .) be a left ideal of a ring (R, +, .).
By Example 9.6, I is a left R-module with
the same addition and module multiplication on R R.
Since I is a subset of R it follows that I is a left submodule of a left R-module R R.
Hence every left ideal of a ring (R, +, .) is a submodule of R as a left R-module.
Suppose that N is a left submodule of R as a left R-module.
By Proposition 9.10 we have that a − b ∈ N and r • a ∈ N, for all a, b ∈ N
and for all r ∈ R, where • is the module multiplication defined on R R.
Thus N is a left ideal of a ring R.
Therefore, the left submodules of R as a left R-module are exactly the left ideals of a
ring (R, +, .).

Example 9.13. Let F be a field and let M be a left F-module. Then the left submo-
dules of a left F-module M are exactly the subspaces of an F-vector space M.
Proof. Since F is a field it follows from the definitions of submodules and subspaces
that there is no difference between subspaces and submodules of a left F-module M.

Department of Mathematics/ College of Education/ Al-Qadisiyah University 114 of 124


Ring Theory Prof. Dr. Akeel Ramadan Mehdi Al-Yassiri

Example 9.14. Let M be a left Z-module. Then the left submodules of a left Z-
module M are exactly the subgroups of an abelian group M.

Exercise 110. Find all left submodules of the following left R-modules:
(1) Z Z; (2) F F, where F is a field; (3) R R;
(4) Z p as a left Z p -module, where p is a prime number; (5) Z6 as a left Z6 -module;
(6) Zn as a left Zn -module; (7) Zn as a left Z-module.

Proposition 9.15. Let M be a left R-module and let N1 and N2 be two left submo-
dules of M. Define N1 ∩ N2 = {x ∈ M | x ∈ N1 and x ∈ N2 } and
N1 + N2 = {y ∈ M | y = a + b with a ∈ N1 and b ∈ N2 }.
Then N1 ∩ N2 and N1 + N2 are left submodules of M.
Proof. (a) We will prove that N1 ∩ N2 is a submodules of M.
Since N1 and N2 are subgroups of an abelian group M it follows that
N1 and N2 are contain 0 and hence N1 ∩ N2 is a nonempty subset of M.
Let a, b ∈ N1 ∩ N2 and let r ∈ R.
Thus a, b ∈ N1 and a, b ∈ N2 .
Since N1 ,→ M and N2 ,→ M it follows from Proposition 9.10 that
a − b , ra ∈ N1 and a − b , ra ∈ N2 and this implies that a − b , ra ∈ N1 ∩ N2 .
By Proposition 9.10, N1 ∩ N2 is a left submodule of M.
(b) We will prove that N1 + N2 is a submodule of M.
Since N1 and N2 are subgroups of an abelian group M
it follows that N1 and N2 are contain 0 and hence 0 = 0 + 0 ∈ N1 + N2 .
Thus N1 + N2 is a nonempty subset of M.
Let x, y ∈ N1 + N2 and let r ∈ R.
Thus x = a1 + b1 and y = a2 + b2 , where a1 , a2 ∈ N1 and b1 , b2 ∈ N2 .
Thus x − y = a1 + b1 − a2 − b2 = a1 − a2 + b1 − b2 and rx = r(a1 + b1 ) = ra1 + rb1 .
Since N1 ,→ M and N2 ,→ M it follows
from Proposition 9.10 that a1 − a2 , ra1 ∈ N1 and b1 − b2 , rb1 ∈ N2
and this implies that x − y, rx ∈ N1 + N2 .
By Proposition 9.10, N1 + N2 is a submodule of M.

Proposition 9.16. Let M be a left R-module and let {Ni }i∈I be a family of left sub-
modules of M. Define Ni = {x ∈ M | x ∈ Ni for all i ∈ I}.
T
T i∈I
Then Ni is a left submodule of M.
i∈I
Proof. Exercise.

Department of Mathematics/ College of Education/ Al-Qadisiyah University 115 of 124


Ring Theory Prof. Dr. Akeel Ramadan Mehdi Al-Yassiri

Theorem 9.17. (Modular Law) If M is a left R-module and if A, B,C are left sub-
modules of M with B ,→ C, then
(A + B) ∩C = (A ∩C) + (B ∩C) = (A ∩C) + B.
Proof. (1) We will prove that (A + B) ∩C = (A ∩C) + B.
Let x ∈ (A + B) ∩C, thus x ∈ A + B and x ∈ C and
hence x = a + b, where a ∈ A and b ∈ B.
Since B ,→ C it follows that x, b ∈ C and hence x − b ∈ C.
Since a ∈ A and a = x − b it follows that a ∈ A ∩C and hence a + b ∈ (A ∩C) + B.
Thus x ∈ (A ∩C) + B and this implies that (A + B) ∩C ⊆ (A ∩C) + B.
Since A ∩C ⊆ C it follows that (A ∩C) + B ⊆ C + B.
Since A ∩C ⊆ A it follows that (A ∩C) + B ⊆ A + B and
hence (A ∩C) + B ⊆ (A + B) ∩ (C + B) = (A + B) ∩C.
Therefore, (A + B) ∩C = (A ∩C) + B.
(2) We will prove that (A ∩C) + (B ∩C) = (A ∩C) + B.
Since B ,→ C it follows that B ∩C = B and hence (A ∩C) + (B ∩C) = (A ∩C) + B.
From (1) and (2) we get that (A + B) ∩C = (A ∩C) + (B ∩C) = (A ∩C) + B.

Remarks 9.18. (1) In Theorem 9.17, if we remove the condition B ,→ C, then we


already have (A ∩C) + (B ∩C) ,→ (A + B) ∩C.
Proof. Since A ,→ A + B it follows that A ∩C ,→ (A + B) ∩C.
Also, since B ,→ A + B it follows that B ∩C ,→ (A + B) ∩C and
hence (A ∩C) + (B ∩C) ,→ (A + B) ∩C.
(2) The reverse inclusion in (1) above does not necessarily hold, for example:
let M = R2 as a left R-module.
Let A = {(0, y) ∈ R2 | y ∈ R}, B = {(x, 0) ∈ R2 | x ∈ R}
and let C = {(x, x) ∈ R2 | x ∈ R}.
Then (A ∩C) + (B ∩C) 6= (A + B) ∩C.

Exercise 111. Let M be a left R-module and let A, B,C are left submodules of M
such that A ⊆ B, A + C = B + C, A ∩ C = B ∩ C. By using Modular Law, prove that
A = B.

Exercise 112. Is the union of any two submodules of a left R-module M is a submo-
dule of M?

Exercise 113. Let I be a right ideal of a ring R and let M be a left R-module. Define
annM (I) = {m ∈ M | am = 0, for all a in I}. Prove that annM (I) is a left submodule?

Department of Mathematics/ College of Education/ Al-Qadisiyah University 116 of 124


Ring Theory Prof. Dr. Akeel Ramadan Mehdi Al-Yassiri

9.3 Quotient Modules


Proposition 9.19. Let R be a ring, let M be a left R-module and let N be a left
submodule of M. The (additive, abelian) quotient group M/N can be made into a left
R-module by defining a module multiplication • : R × M/N → M/N by r • (x + N) =
(rx) + N, for all r ∈ R, x + N ∈ M/N.
Proof. Since M is an abelian group under + the quotient group M/N is defined and
is an abelian group.
To see that the multiplication • is well defined,
suppose x + N = y + N, i.e., x − y ∈ N.
Since N is a left R-submodule, r(x − y) ∈ N.
Thus rx − ry ∈ N ⇒ rx + N = ry + N.
Thus • is a well defined.
Let x + N, y + N ∈ M/N and let r, r1 , r2 ∈ R.
Thus r • ((x + N) + (y + N)) = r • ((x + y) + N) = r(x + y) + N = (rx + ry) + N =
(rx + N) + (ry + N) = r • (x + N) + r • (y + N) and
(r1 + r2 ) • (x + N) = (r1 + r2 )x + N = (r1 x + r2 x) + N = (r1 x + N) + (r2 x + N) =
r1 • (x + N) + r2 • (x + N).
Also, (r1 r2 ) • (x + N) = (r1 r2 )x + N = r1 (r2 x) + N = r1 • (r2 x + N) = r1 • (r2 • (x +
N)).
Thus M/N is a left R-module.

Definition 9.20. The left R-module M/N is defined in Proposition 9.19 is called
quotient (or factor) module.

Proposition 9.21. Let N be a left R-submodule of a left R-module M. Then A is


a left R-submodule of a left R-module M/N if and only if there is a unique left
R-submodule B of M such that N ⊆ B and A = B/N.
Proof. Exercise.

Example 9.22. Find all submodules of Z20 / < 4 > as left Z-module.
Solution: All submodules of the left Z-module Z20 are:
< 0 >, Z20 , < 2 >, < 4 >, < 5 >, < 10 >.
Since the submodules which contain the submodule < 4 > are Z20 , < 2 >, < 4 >,
we have from Proposition 9.21 that the submodules of Z20 / < 4 > as left Z-module
are Z20 / < 4 >, < 4 > / < 4 >= 0, < 2 > / < 4 >.

Department of Mathematics/ College of Education/ Al-Qadisiyah University 117 of 124


Ring Theory Prof. Dr. Akeel Ramadan Mehdi Al-Yassiri

9.4 Homomorphisms of Modules and Isomorphism Theorems


Definition 9.23. Let N and M be left R-modules.
(1) A function f : N → M is said to be a left R-module homomorphism (or just left
R-homomorphism) if for all a, b ∈ N and r ∈ R, then
f (a + b) = f (a) + f (b) and f (ra) = r f (a).
(2) A left R-module homomorphism is a monomorphism if it is injective and is an
epimorphism if it is surjective.
(3) A left R-module homomorphism is an isomorphism if it is both injective and
surjective. The modules N and M are said to be isomorphic, denoted N ∼ = M, if there
is some left R-module isomorphism ϕ : N → M.
(4) If f : N → M is a left R-module homomorphism.
The kernel of f is denoted by ker( f ) and defined as ker( f ) = {a ∈ N | f (a) = 0}.
The image of f is denoted by im( f ) and defined as
im( f ) = f (N) = {b ∈ M | b = f (a) for some a ∈ N}.

Proposition 9.24. If f : M → N is a left R-homomorphism, then f (0M ) = 0N and


f (−x) = − f (x) for each x ∈ M.
Proof. f (0M ) = f (0M + 0M ) = f (0M ) + f (0M ) ⇒ 0N = f (0M ).
(Exercise) f (−x) = − f (x) for each x ∈ M.

Proposition 9.25. Let f : M → N be a left R-homomorphism.


(1) If B is a left submodule of N, then f −1 (B) is a left submodule of M.
(2) If A is a left submodule of M, then f (A) is a left submodule of N.
Proof. (1) Let x, y ∈ f −1 (B) and let r ∈ R.
Thus x, y ∈ M such that f (x), f (y) ∈ B and hence
f (x + y) = f (x) + f (y) ∈ B and f (rx) = r f (x) ∈ B so x + y, rx ∈ f −1 (B).
Thus f −1 (B) is a left submodule of M.
(2) (Exercise).

Corollary 9.26. Let f : M → N be a left R-homomorphism. Then ker( f ) is a left


submodule of M and im( f ) is a left submodule of N .
Proof. Since ker( f ) = {x ∈ M | f (x) = 0} = f −1 (0)
and since 0 ,→ N it follows from Proposition 9.25(1) that
ker( f ) is a left submodule of M.
Also, since im( f ) = {y ∈ N | y = f (x) for some x ∈ M} = f (M)
and since M ,→ M it follows Proposition 9.25(2) that
im( f ) is a left submodule of N.

Department of Mathematics/ College of Education/ Al-Qadisiyah University 118 of 124


Ring Theory Prof. Dr. Akeel Ramadan Mehdi Al-Yassiri

Example 9.27. For any modules the zero map 0 : A → B given by 0(a) = 0B for all
a ∈ A is a left R-homomorphism (Exercise).

Example 9.28. Let M be a left R-module and let N be a left submodule of M. The
inclusion mapping iN : N → M defined by iN (x) = x, for all x ∈ N is a left R-
monomorphism.
Proof. Let x, y ∈ N and let r ∈ R.
Thus iN (x + y) = x + y = iN (x) + iN (y) and iN (rx) = rx = riN (x).
Hence inclusion mapping iN is a left R-homomorphism.
Since iN is an injective mapping ⇒ iN is a left R-monomorphism.

Example 9.29. Let M be a left R-module. The identity mapping IM : M → M defined


by IM (x) = x for all x ∈ M is a left R-isomorphism.
Proof. (Exercise).

Example 9.30. Let M be a left R-module and let N be a left submodule of M. Then
the natural mapping π : M → M/N defined by π(x) = x + N for all x ∈ M is a left
R-epimorphism with kernel N.
Proof. Let x, y ∈ M and let r ∈ R.
Thus π(x + y) = (x + y) + N = (x + N) + (y + N) = π(x) + π(y) and π(rx) =
rx + N = r(x + N) = rπ(x).
Hence π : M → M/N is a left R-homomorphism.
Since π is a surjective mapping ⇒ π is an R-epimorphism.
Also, ker(π) = {x ∈ M | π(x) = 0} = {x ∈ M | x + N = N}
= {x ∈ M | x ∈ N} = N.

Example 9.31. Let f : Z → Z defined by f (x) = 2x, for all x ∈ Z. Then f is a left
Z-module homomorphism but it is not ring homomorphism, since f (1) = 2 6= 1.

Exercise 114. Let V1 and V2 be vector spaces over a field F and let f : V1 → V2
be a function. Prove that f is a left F-homomorphism if and only if it is linear
transformation over F.

 
x 0
Exercise 115. Let R be a ring. Define f : R → M2×2 (R) by f (x) = for all
0 x
x ∈ R. Prove that f is a left R-monomorphism.

Department of Mathematics/ College of Education/ Al-Qadisiyah University 119 of 124


Ring Theory Prof. Dr. Akeel Ramadan Mehdi Al-Yassiri

Proposition 9.32. Let α : N → M be a left R-homomorphism. Then α is R-monomorphism


if and only if ker(α) = {0N }.
Proof. (⇒) Suppose that α is R-monomorphism.
Let x ∈ ker(α), thus α(x) = 0M .
Since α(0N ) = 0M , we have α(0N ) = α(x).
Since α is an R-monomorphism ⇒ x = 0N ⇒ ker(α) = {0N }.
(⇐) Suppose that ker(α) = {0N }.
Let x, y ∈ N such that α(x) = α(y)
⇒ α(x) − α(y) = 0M ⇒ α(x − y) = 0M ⇒ x − y ∈ ker(α).
Since ker(α) = {0N } ⇒ x − y = 0N ⇒ x = y ⇒ α is an R-monomorphism.

Proposition 9.33. Let f : N → M and g : M → K be left R-homomorphisms. Then


g ◦ f : N → K is a left R-homomorphism.
Proof. Let a, b ∈ N and let r ∈ R,
thus (g ◦ f )(a + b) = g( f (a + b)) = g( f (a) + f (b))
= g( f (a)) + g( f (b)) = (g ◦ f )(a) + (g ◦ f )(b).
Also, (g ◦ f )(ra) = g( f (ra)) = g(r f (a)) = rg( f (a)) = r((g ◦ f )(a)).
Thus g ◦ f : N → K is a left R-homomorphism.

Theorem 9.34. (First Isomorphism Theorem for Modules) If f : M → N is a left


R-homomorphism, then M/ker( f ) ∼
= im( f ).
Proof. Define ϕ : M/ker( f ) → im( f ) by ϕ(x + ker( f )) = f (x), for all x ∈ M.
If x + ker( f ) = y + ker( f ), then
x − y ∈ ker( f ) ⇒ f (x − y) = 0N ⇒ f (x) − f (y) = 0N ⇒ f (x) = f (y)
⇒ ϕ(x + ker( f )) = ϕ(y + ker( f )).
Thus ϕ is well-defined.
Let x + ker( f ), y + ker( f ) ∈ M/ker( f ) and let r ∈ R, thus
ϕ((x + ker( f )) + (y + ker( f ))) = ϕ((x + y) + ker( f )) = f (x + y) = f (x) + f (y) =
ϕ(x + ker( f )) + ϕ(y + ker( f )).
Also, ϕ(r(x + ker( f ))) = ϕ(rx + ker( f )) = f (rx) = r f (x) = rϕ(x + ker( f )).
Hence ϕ is a well-defined left R-homomorphism.
Let y ∈ im( f ) ⇒ ∃x ∈ M 3 f (x) = y.
Since x + ker( f ) ∈ M/ker( f ) and ϕ(x + ker( f )) = f (x) = y,
thus ϕ is a surjective mapping and hence ϕ is an R-epimorphism.
Let x + ker( f ) ∈ ker(ϕ), thus
ϕ(x + ker( f )) = 0N ⇒ f (x) = 0N ⇒ x ∈ ker( f ) ⇒ x + ker( f ) = ker( f ) = 0(M/ker( f ))
⇒ ker(ϕ) = 0(M/ker( f )) .
By Proposition 9.32, ϕ is an R-monomorphism.
Therefore, ϕ is an R-isomorphism and hence M/ker( f ) ∼ = im( f ).

Department of Mathematics/ College of Education/ Al-Qadisiyah University 120 of 124


Ring Theory Prof. Dr. Akeel Ramadan Mehdi Al-Yassiri

Corollary 9.35. If f : M → N is a left R-epimorphism, then M/ker( f ) ∼


= N.

Theorem 9.36. (Second Isomorphism Theorem for Modules) If M1 and M2 are


left submodules of a left R-module M such that M1 ⊆ M2 , then M2 /M1 is a left
submodule of M/M1 and (M/M1 )/(M2 /M1 ) ∼= M/M2 .
Proof. It is clear that M2 /M1 is a left submodule of M/M1 (H.W.).
Define ϕ : M/M1 → M/M2 by ϕ(x + M1 ) = x + M2 , for all x + M1 ∈ M/M1 .
Suppose that x + M1 = y + M1 , where x, y ∈ M. Thus x − y ∈ M1 .
Since M1 ⊆ M2 ⇒ x − y ∈ M2 ⇒ x + M2 = y + M2
⇒ ϕ(x + M1 ) = ϕ(y + M1 ).
Hence ϕ is a well-defined mapping.
Let x + M1 , y + M1 ∈ M/M1 and let r ∈ R.
Thus ϕ((x + M1 ) + (y + M1 )) = ϕ((x + y) + M1 )
= (x + y) + M2 = (x + M2 ) + (y + M2 ) = ϕ(x + M1 ) + ϕ(y + M1 ).
Also, ϕ(r(x + M1 )) = ϕ(rx + M1 ) = rx + M2 = r(x + M2 ) = rϕ(x + M1 ).
Thus ϕ is a left R-homomorphism.
Let x + M2 ∈ M/M2 , thus x ∈ M ⇒ x + M1 ∈ M/M1 and ϕ(x + M1 ) = x + M2 .
Thus ϕ is a left R-epimorphism.
By Corollary 9.35, (M/M1 )/ker(ϕ) ∼ = M/M2 .
Since ker(ϕ) = {x + M1 ∈ M/M1 | ϕ(x + M1 )
= 0(M/M2 ) = M2 } = {x + M1 ∈ M/M1 | x + M2 = M2 }
= {x + M1 ∈ M/M1 | x ∈ M2 } = M2 /M1 .
Thus (M/M1 )/(M2 /M1 ) ∼ = M/M2 .

Example 9.37. Since < 4 >,→< 2 >,→ Z Z,


we have from Theorem 9.36 that Z/ < 2 >∼
= (Z/ < 4 >)/(< 2 > / < 4 >).

Theorem 9.38. (Third Isomorphism Theorem for Modules) If M1 and M2 are left
submodules of a left R-module M, then M1 /(M1 ∩ M2 ) ∼
= (M1 + M2 )/M2 .
Proof. Define ϕ : M1 → (M1 + M2 )/M2 by ϕ(x) = x + M2 , for all x ∈ M1 .
We can prove that ϕ is a left R-epimorphism (H.W.).
By Corollary 9.35, M1 /ker(ϕ) ∼ = (M1 + M2 )/M2 .
Since ker(ϕ) = {x ∈ M1 | ϕ(x) = 0((M1 +M2 )/M2 ) = M2 }
= {x ∈ M1 | x + M2 = M2 } = {x ∈ M1 | x ∈ M2 } = M1 ∩ M2 ,
thus M1 /(M1 ∩ M2 ) ∼
= (M1 + M2 )/M2 .

Exercise 116. Prove that if M = M1 ⊕ M2 , then M/M2 ∼


= M1 .

Department of Mathematics/ College of Education/ Al-Qadisiyah University 121 of 124


Ring Theory Prof. Dr. Akeel Ramadan Mehdi Al-Yassiri

9.5 Simple Modules


Definition 9.39. A left R-module M is said to be simple if M 6= 0 and the only left
R-submodules of M are 0 and M.

Examples 9.40. (1) Let F be a filed. Then F is a simple F-module.


(2) Z is not a simple Z-module.
L
(3) Let F be a filed. Then F F is not a simple F-module.
(4) Z4 is not a simple Z-module.

Example 9.41. Z/pZ (or Z p ) is a simple Z-module for any prime number p ≥ 2.
Proof. Assume that there is a prime number p ≥ 2
such that Z/pZ is not a simple Z-module.
Thus there is a submodule N of Z/pZ such that N 6= 0 and N 6= Z/pZ.
By Proposition 9.21, N = L/pZ with L is a submodule of Z as Z-module and pZ ⊆ L.
Since N 6= 0, we have L 6= pZ and hence L $ pZ.
Since N 6= Z/pZ, we have L 6= Z and hence pZ $ L $ Z
and this implies that pZ is not a maximal ideal of a ring Z.
Since p is a prime number, we have
pZ is a maximal ideal of a ring Z and this is a contradiction.
Thus Z/pZ (or Z p ) is a simple Z-module for any prime number p ≥ 2.

Proposition 9.42. A left R-module M is simple if and only if M =< a >, for all
nonzero element a ∈ M.
Proof. (⇒) Suppose that M is a simple module, thus M 6= 0.
Let 0 6= a ∈ M and let N =< a >, thus N is a nonzero submodule of M.
Since M is a simple left R-module, we have N = M and hence M =< a >.
(⇐) Suppose that M =< a >, for all nonzero element a ∈ M.
Assume that M is not simple,
we have there is a submodule N of M such that 0 $ N $ M
and hence there is 0 6= a ∈ N.
By hypothesis, M =< a > and thus M ⊆ N.
Hence N = M and this is a contradiction.
Thus M is simple left R-module.

Definition 9.43. Let M be a left R-module and let m ∈ M. The annihilator of m is


denoted by annR (m) and defined as follows: annR (m) = {r ∈ R | rm = 0}.

Department of Mathematics/ College of Education/ Al-Qadisiyah University 122 of 124


Ring Theory Prof. Dr. Akeel Ramadan Mehdi Al-Yassiri

Lemma 9.44. Let M be a left R-module and let m ∈ M. Then annR (m) is a left ideal
of a ring R.
Proof. Exercise.

Proposition 9.45. A left R-module M is cyclic if and only if M ∼


= R/annR (m) for
some m ∈ M.
Proof. (⇒) Suppose that a left R-module M is cyclic,
thus there is m ∈ M such that M =< m >.
Define f : R → M by f (r) = rm for every r ∈ R.
It is clear that f is a left R-epimorphism (H.W.).
By Corollary 9.35, R/ker( f ) ∼ = M.
Since ker( f ) = {r ∈ R | f (r) = 0} = {r ∈ R | rm = 0} = annR (m), we have
M∼ = R/annR (m).
(⇐) Suppose that M ∼ = R/annR (m) for some m ∈ M.
Since R/annR (m) is a cyclic left R-module generated by 1 + annR (m),
we have M is a cyclic left R-module.

Proposition 9.46. If R is a ring, then a cyclic left R-module M =< a > is simple if
and only if annR (a) is a maximal left ideal of R.
Proof. Let M =< a > be a cyclic left R-module generated by a.
(⇒) Suppose that M is a simple left R-module.
From the proof of Proposition 9.45 we have that M ∼ = R/annR (a).
Since M is simple left R-module, R/annR (a) is a simple left R-module and hence
R/annR (a) and 0 = {ann(a)}
are the only left R-submodules of a left R-module R/annR (a).
Assume that annR (a) is not maximal left ideal of a ring R.
Thus there is a left ideal J of R such that annR (a) $ J $ R.
Hence there is a left R-submodule J of a left R-module R such that annR (a) $ J $ R R.
By Proposition 9.21, J/annR (a) is a non trivial submodule
of a left R-module R/annR (a) and this is a contradiction.
Thus annR (a) is a maximal left ideal of a ring R.
(⇐) Suppose that annR (a) is a maximal left ideal of R.
Assume that M is not simple left R-module, thus R/annR (a) is not simple left R-
module and hence there is a left R-submodule B of a left R-module R/annR (a)
such that 0 $ B $ R/annR (a).
By Proposition 9.21, there is a unique left submodule J of R R such that
annR (a) $ J $ R and B = J/annR (a) and this contradicts the maximality of the ideal
annR (a) in R. Thus M is a simple left R-module.

Department of Mathematics/ College of Education/ Al-Qadisiyah University 123 of 124


Ring Theory Prof. Dr. Akeel Ramadan Mehdi Al-Yassiri

References
[1] M. R. Adhikari and A. Adhikari, Basic Modern Algebra with Applications,
Springer, New Delhi, 2014.
[2] M. F. Atiyah and I. G. Macdonald, Introduction to Commutative Algebra,
Addison-Wesley Publishing Company, Inc., 1969.
[3] R. B. Ash, Abstract Algebra (The Basic Graduate Year), 2000.
[4] F. W. Anderson and K. R. Fuller, Rings and Categories of Modules, Springer-
Verlag, New York, 1974.
[5] P. E. Bland, Rings and Their Modules, Walter de Gruyter and Co., Berlin,
2011.
[6] D. M. Burton, Introduction to Modern Abstract Algebra, Addison-Wesley,
London, 1967.
[7] P. A. Grillet, Abstract Algebra, 2nd edition, GTM 242, Springer, 2007.
[8] F. Kasch, Modules and Rings, Academic Press, New York, 1982.
[9] J. Lambek, A module is flat if and only if its character module is injective,
Canad. Math. Bull., 7(1964), 237-243.
[10] B. Stenström, Rings of Quotients, Sepringer-Verlage, New York, 1975.
[11] R. Wisbauer, Foundations of Module and Ring Theory, Gordon and Breach,
1991.

Department of Mathematics/ College of Education/ Al-Qadisiyah University 124 of 124

You might also like